Dataset Preview
Viewer
The full dataset viewer is not available (click to read why). Only showing a preview of the rows.
The dataset generation failed
Error code:   DatasetGenerationError
Exception:    DatasetGenerationError
Message:      An error occurred while generating the dataset
Traceback:    Traceback (most recent call last):
                File "/src/services/worker/.venv/lib/python3.9/site-packages/datasets/builder.py", line 2011, in _prepare_split_single
                  writer.write_table(table)
                File "/src/services/worker/.venv/lib/python3.9/site-packages/datasets/arrow_writer.py", line 585, in write_table
                  pa_table = table_cast(pa_table, self._schema)
                File "/src/services/worker/.venv/lib/python3.9/site-packages/datasets/table.py", line 2302, in table_cast
                  return cast_table_to_schema(table, schema)
                File "/src/services/worker/.venv/lib/python3.9/site-packages/datasets/table.py", line 2261, in cast_table_to_schema
                  arrays = [cast_array_to_feature(table[name], feature) for name, feature in features.items()]
                File "/src/services/worker/.venv/lib/python3.9/site-packages/datasets/table.py", line 2261, in <listcomp>
                  arrays = [cast_array_to_feature(table[name], feature) for name, feature in features.items()]
                File "/src/services/worker/.venv/lib/python3.9/site-packages/datasets/table.py", line 1802, in wrapper
                  return pa.chunked_array([func(chunk, *args, **kwargs) for chunk in array.chunks])
                File "/src/services/worker/.venv/lib/python3.9/site-packages/datasets/table.py", line 1802, in <listcomp>
                  return pa.chunked_array([func(chunk, *args, **kwargs) for chunk in array.chunks])
                File "/src/services/worker/.venv/lib/python3.9/site-packages/datasets/table.py", line 2116, in cast_array_to_feature
                  return array_cast(
                File "/src/services/worker/.venv/lib/python3.9/site-packages/datasets/table.py", line 1804, in wrapper
                  return func(array, *args, **kwargs)
                File "/src/services/worker/.venv/lib/python3.9/site-packages/datasets/table.py", line 1963, in array_cast
                  return array.cast(pa_type)
                File "pyarrow/array.pxi", line 997, in pyarrow.lib.Array.cast
                File "/src/services/worker/.venv/lib/python3.9/site-packages/pyarrow/compute.py", line 404, in cast
                  return call_function("cast", [arr], options, memory_pool)
                File "pyarrow/_compute.pyx", line 590, in pyarrow._compute.call_function
                File "pyarrow/_compute.pyx", line 385, in pyarrow._compute.Function.call
                File "pyarrow/error.pxi", line 154, in pyarrow.lib.pyarrow_internal_check_status
                File "pyarrow/error.pxi", line 91, in pyarrow.lib.check_status
              pyarrow.lib.ArrowInvalid: Failed to parse string: 'All matters in the universe have fundamental properties called spin, besides their mass and charge. Spin is an intrinsic form of angular momentum carried by particles. Despite the fact that quantum mechanics is needed for a full treatment of spin, we can still study the physics of spin using the usual classical formalism. In this problem, we are investigating the influence of magnetic field on spin using its classical analogue.
              
              The classical torque equation of spin is given by
              
              $$
              \boldsymbol{\tau}=\frac{d \boldsymbol{L}}{d t}=\boldsymbol{\mu} \times \boldsymbol{B}
              $$
              
              In this case, the angular momentum $\boldsymbol{L}$ represents the "intrinsic" spin of the particles, $\boldsymbol{\mu}$ is the magnetic moment of the particles, and $\boldsymbol{B}$ is magnetic field. The spin of a particle is associated with a magnetic moment via the equation
              
              $$
              \boldsymbol{\mu}=-\gamma \boldsymbol{L}
              $$
              
              where $\gamma$ is the gyromagnetic ratio.
              
              In this problem, the term "frequency" means angular frequency (rad/s), which is a scalar quantity. All bold letters represent vectors; otherwise they represent scalars.
              
              Part A. Larmor precession
              Context question:
              1.  Prove that the magnitude of magnetic moment $\mu$ is always constant under the influence of a magnetic field $\boldsymbol{B}$. For a special case of stationary (constant) magnetic field, also show that the angle between $\boldsymbol{\mu}$ and $\boldsymbol{B}$ is constant.
              
              (Hint: You can use properties of vector products.)
              Context answer:
              \boxed{证明题}
              
              
              Context question:
              2.  A uniform magnetic field $\boldsymbol{B}$ exists and it makes an angle $\phi$ with a particle's magnetic moment $\boldsymbol{\mu}$. Due to the torque by the magnetic field, the magnetic moment $\boldsymbol{\mu}$ rotates around the field $\boldsymbol{B}$, which is also known as Larmor precession. Determine the Larmor precession frequency $\omega_{0}$ of the magnetic moment with respect to $\boldsymbol{B}=B_{0} \boldsymbol{k}$.
              Context answer:
              \boxed{$\omega_{0}=\gamma B_{0}$}
              
              
              Extra Supplementary Reading Materials:
              
              Part B. Rotating frame
              
              In this section, we choose a rotating frame $S^{\prime}$ as our frame of reference. The rotating frame $S^{\prime}=\left(x^{\prime}, y^{\prime}, z^{\prime}\right)$ rotates with an angular velocity $\omega \boldsymbol{k}$ as seen by an observer in the laboratory frame $S=(x, y, z)$, where the axes $x^{\prime}, y^{\prime}, z^{\prime}$ intersect with $x, y, z$ at time $t=0$. Any vector $\boldsymbol{A}=A_{x} \boldsymbol{i}+A_{y} \boldsymbol{j}+A_{z} \boldsymbol{k}$ in a lab frame can be written as $\boldsymbol{A}=A_{x}{ }^{\prime} \boldsymbol{i}^{\prime}+A_{y}{ }^{\prime} \boldsymbol{j}^{\prime}+A_{z}{ }^{\prime} \boldsymbol{k}^{\prime}$ in the rotating frame $S^{\prime}$. The time derivative of the vector becomes
              
              $$
              \frac{d \boldsymbol{A}}{d t}=\left(\frac{d A_{x}{ }^{\prime}}{d t} \boldsymbol{i}^{\prime}+\frac{d A_{y}{ }^{\prime}}{d t} \boldsymbol{j}^{\prime}+\frac{d A_{z}{ }^{\prime}}{d t} \boldsymbol{k}^{\prime}\right)+\left(A_{x}{ }^{\prime} \frac{d \boldsymbol{i}^{\prime}}{d t}+A_{y}{ }^{\prime} \frac{d \boldsymbol{j}^{\prime}}{d t}+A_{z}{ }^{\prime} \frac{d \boldsymbol{k}^{\prime}}{d t}\right)
              $$
              
              
              
              $$
              \left(\frac{d \boldsymbol{A}}{d t}\right)_{l a b}=\left(\frac{d \boldsymbol{A}}{d t}\right)_{r o t}+(\omega \mathbf{k} \times \boldsymbol{A})
              $$
              
              where $\left(\frac{d \boldsymbol{A}}{d t}\right)_{l a b}$ is the time derivative of vector $\boldsymbol{A}$ seen by an observer in the lab frame, and $\left(\frac{d A}{d t}\right)_{\text {rot }}$ is the time derivative seen by an observer in the rotating frame. For all the following problems in this part, the answers are referred to the rotating frame $S^{\prime}$.
              Context question:
              1.  Show that the time evolution of the magnetic moment follows the equation
              
              $$
              \left(\frac{d \boldsymbol{\mu}}{d t}\right)_{r o t}=-\gamma \boldsymbol{\mu} \times \boldsymbol{B}_{e f f}
              $$
              
              where $\boldsymbol{B}_{\text {eff }}=\boldsymbol{B}-\frac{\omega}{\gamma} \boldsymbol{k}^{\prime}$ is the effective magnetic field.
              Context answer:
              \boxed{证明题}
              
              
              Context question:
              2.  For $\boldsymbol{B}=B_{0} \boldsymbol{k}$, what is the new precession frequency $\Delta$ in terms of $\omega_{0}$ and $\omega$ ?
              Context answer:
              \boxed{$\Delta =\gamma B_{0}-\omega$}
              
              
              Context question:
              3.  Now, let us consider the case of a time-varying magnetic field. Besides a constant magnetic field, we also apply a rotating magnetic field $\boldsymbol{b}(t)=b(\cos \omega t \boldsymbol{i}+\sin \omega t \boldsymbol{j})$, so $\boldsymbol{B}=B_{0} \boldsymbol{k}+\boldsymbol{b}(t)$. Show that the new Larmor precession frequency of the magnetic moment is
              
              $$
              \Omega=\gamma \sqrt{\left(B_{0}-\frac{\omega}{\gamma}\right)^{2}+b^{2}}
              $$
              Context answer:
              \boxed{证明题}
              
              
              Context question:
              4.  Instead of applying the field $\boldsymbol{b}(t)=b(\cos \omega t \boldsymbol{i}+\sin \omega t \boldsymbol{j})$, now we apply $\boldsymbol{b}(t)=b(\cos \omega t \boldsymbol{i}-\sin \omega t \boldsymbol{j})$, which rotates in the opposite direction and hence $\boldsymbol{B}=B_{0} \boldsymbol{k}+b(\cos \omega t \boldsymbol{i}-\sin \omega t \boldsymbol{j})$. What is the effective magnetic field $\boldsymbol{B}_{\text {eff }}$ for this case (in terms of the unit vectors $\boldsymbol{i}^{\prime}, \boldsymbol{j}^{\prime}, \boldsymbol{k}^{\prime}$ )? What is its time average, $\overline{\boldsymbol{B}_{\text {eff }}}$ (recall that $\overline{\cos 2 \pi t / T}=\overline{\sin 2 \pi t / T}=0$ )?
              Context answer:
              \boxed{$\mathbf{B}_{\mathrm{eff}}=\left(B_{0}-\frac{\omega}{\gamma}\right) \mathbf{k}^{\prime}+b\left(\cos 2 \omega t \mathbf{i}^{\prime}-\sin 2 \omega t \mathbf{j}^{\prime}\right)$ , $\overline{\mathbf{B}_{\mathrm{eff}}}=\left(B_{0}-\frac{\omega}{\gamma}\right) \mathbf{k}^{\prime}$}
              
              
              Extra Supplementary Reading Materials:
              
              Part C. Rabi oscillation  
              
              For an ensemble of $N$ particles under the influence of a large magnetic field, the spin can have two quantum states: "up" and "down". Consequently, the total population of spin up $N_{\uparrow}$ and down $N_{\downarrow}$ obeys the equation
              
              $$
              N_{\uparrow}+N_{\downarrow}=N
              $$
              
              The difference of spin up population and spin down population yields the macroscopic magnetization along the $z$ axis:
              
              $$
              M=\left(N_{\uparrow}-N_{\downarrow}\right) \mu=N \mu_{z} .
              $$
              
              In a real experiment, two magnetic fields are usually applied, a large bias field $B_{0} \boldsymbol{k}$ and an oscillating field with amplitude $2 b$ perpendicular to the bias field $\left(b \ll B_{0}\right)$. Initially, only the large bias is applied, causing all the particles lie in the spin up states ( $\boldsymbol{\mu}$ is oriented in the $z$-direction at $t=0$ ). Then, the oscillating field is turned on, where its frequency $\omega$ is chosen to be in resonance with the Larmor precession frequency $\omega_{0}$, i.e. $\omega=\omega_{0}$. In other words, the total field after time $t=0$ is given by
              
              $$
              \boldsymbol{B}(t)=B_{0} \boldsymbol{k}+2 b \cos \omega_{0} t \boldsymbol{i} .
              $$
              Context question:
              1.  In the rotating frame $S^{\prime}$, show that the effective field can be approximated by
              
              $$
              \boldsymbol{B}_{\text {eff }} \approx b \boldsymbol{i}^{\prime},
              $$
              
              which is commonly known as rotating wave approximation. What is the precession frequency $\Omega$ in frame $S^{\prime}$ ?
              Context answer:
              \boxed{$\Omega=\gamma b$}
              
              
              Context question:
              2.  Determine the angle $\alpha$ that $\boldsymbol{\mu}$ makes with $\boldsymbol{B}_{\text {eff }}$. Also, prove that the magnetization varies with time as
              
              $$
              M(t)=N \mu(\cos \Omega t) .
              $$
              Context answer:
              \boxed{证明题}
              
              
              Context question:
              3.  Under the application of magnetic field described above, determine the fractional population of each spin up $P_{\uparrow}=N_{\uparrow} / N$ and spin down $P_{\downarrow}=N_{\downarrow} / N$ as a function of time. Plot $P_{\uparrow}(t)$ and $P_{\downarrow}(t)$ on the same graph vs. time $t$. The alternating spin up and spin down population as a function of time is called Rabi oscillation.
              Context answer:
              \boxed{$P_{\downarrow}=\sin ^{2} \frac{\Omega t}{2}$ , $P_{\uparrow}=\cos ^{2} \frac{\Omega t}{2}$}
              
              
              Extra Supplementary Reading Materials:
              
              Part D. Measurement incompatibility  
              
              Spin is in fact a vector quantity; but due to its quantum properties, we cannot measure each of its components simultaneously (i.e. we can know both $|\boldsymbol{\mu}|$ and $\mu_{z}$ as in above problems; but not all $|\boldsymbol{\mu}|, \mu_{x}, \mu_{y}$, and $\mu_{z}$ simultaneously). In this problem, we will do a calculation based on the Heisenberg uncertainty principle (using the relation $\Delta p_{q} \Delta q \geq \hbar$ ) to show how these measurements are incompatible with each other.
              Context question:
              1.  Let us consider an oven source of silver atoms, which has a small opening. The atoms stream out of the opening along $-y$ direction (see Figure below) and experience a spatial varying field $\boldsymbol{B}_{1}$. The field $\boldsymbol{B}_{1}$ has strong bias field component in the $z$ direction, where the atoms with different magnetic moment $\mu_{z}= \pm \gamma \hbar$ are split in the $z$ direction. At a distance $D$ from the oven source, a screen $S C_{1}$ is put to allow only spin up atoms to pass (blocking spin down atoms). Thus, at the instant after passing the screen, the atoms are prepared in spin up states. After the screen, the atoms enter a region of nonhomogenous field $\boldsymbol{B}_{2}$ where the atoms feel a force
              
              $$
              F_{x}=\mu_{x} C
              $$
              
              The field $\boldsymbol{B}_{2}$ has strong bias field component in the $x$ direction, where the atoms have magnetic moment $\mu_{x}= \pm \gamma \hbar$.
              
              
              
              <img_4533>
              
              In order to determine $\mu_{x}$ by observing the splitting in $x$ direction, show that the following condition must be fulfilled:
              
              $$
              \frac{1}{\hbar}\left|\mu_{x}\right| \Delta x C t \gg 1
              $$
              
              where $t$ is the duration after leaving the screen $S C_{1}$ and $\Delta x$ is the opening width on $S C_{1}$.
              Context answer:
              \boxed{证明题}
              ' as a scalar of type double
              
              The above exception was the direct cause of the following exception:
              
              Traceback (most recent call last):
                File "/src/services/worker/src/worker/job_runners/config/parquet_and_info.py", line 1324, in compute_config_parquet_and_info_response
                  parquet_operations = convert_to_parquet(builder)
                File "/src/services/worker/src/worker/job_runners/config/parquet_and_info.py", line 938, in convert_to_parquet
                  builder.download_and_prepare(
                File "/src/services/worker/.venv/lib/python3.9/site-packages/datasets/builder.py", line 1027, in download_and_prepare
                  self._download_and_prepare(
                File "/src/services/worker/.venv/lib/python3.9/site-packages/datasets/builder.py", line 1122, in _download_and_prepare
                  self._prepare_split(split_generator, **prepare_split_kwargs)
                File "/src/services/worker/.venv/lib/python3.9/site-packages/datasets/builder.py", line 1882, in _prepare_split
                  for job_id, done, content in self._prepare_split_single(
                File "/src/services/worker/.venv/lib/python3.9/site-packages/datasets/builder.py", line 2038, in _prepare_split_single
                  raise DatasetGenerationError("An error occurred while generating the dataset") from e
              datasets.exceptions.DatasetGenerationError: An error occurred while generating the dataset

Need help to make the dataset viewer work? Open a discussion for direct support.

id
int64
subfield
string
context
null
question
string
solution
string
final_answer
string
is_multiple_answer
bool
unit
null
answer_type
string
error
null
0
Trigonometric Functions
null
在 $\triangle A B C$ 中, $\sin A=\frac{\sqrt{2}}{2}$, 求 $\cos B+\sqrt{2} \cos C$ 的取值范围.
['根据题意, $A=\\frac{\\pi}{4}$ 或 $A=\\frac{3 \\pi}{4}$. 情形一: $A=\\frac{\\pi}{4}$. 此时\n\n$$\n\\begin{aligned}\n\\cos B+\\sqrt{2} \\cos C & =\\cos \\left(\\frac{3 \\pi}{4}-C\\right)+2 \\sqrt{2} \\cos C \\\\\n& =\\frac{\\sqrt{2}}{2} \\sin C+\\frac{\\sqrt{2}}{2} \\cos C \\\\\n& =\\sin \\left(C+\\frac{\\pi}{4}\\right),\n\\end{aligned}\n$$\n\n其中 $C \\in\\left(0, \\frac{3 \\pi}{4}\\right)$, 从而题中代数式的取值范围是 $(0,1]$. 情形二: $A=\\frac{3 \\pi}{4}$. 此时\n\n$$\n\\begin{aligned}\n\\cos B+\\sqrt{2} \\cos C & =\\cos \\left(\\frac{\\pi}{4}-C\\right)+2 \\sqrt{2} \\cos C \\\\\n& =\\frac{\\sqrt{2}}{2} \\sin C+\\frac{3 \\sqrt{2}}{2} \\cos C \\\\\n& =\\sqrt{5} \\sin (C+\\arctan 3),\n\\end{aligned}\n$$\n\n其中 $C \\in\\left(0, \\frac{\\pi}{4}\\right)$, 从而题中代数式的取值范围是 $(0, \\sqrt{5}]$. 综上所述, 所求代数式的取值范围是 $(0,1] \\cup(2, \\sqrt{5}]$.']
['$(0,1] \\cup (2, \\sqrt{5}]$']
false
null
Interval
null
1
Elementary Functions
null
对正整数 $n$ 及实数 $x(0 \leqslant x<n)$, 定义 $$ f(n, x)=(1-\{x\}) \cdot\left(\begin{array}{c} {[x]} \\ n \end{array}\right)+\{x\} \cdot\left(\begin{array}{c} {[x]+1} \\ n \end{array}\right), $$ 其中 $[x]$ 表示不超过实数 $x$ 的最大整数, $\{x\}=x-[x]$. 若整数 $m, n \geqslant 2$ 满足 $$ f\left(m, \frac{1}{n}\right)+f\left(m, \frac{2}{n}\right)+\cdots+f\left(m, \frac{m n-1}{n}\right)=123 $$ 求 $f\left(n, \frac{1}{m}\right)+f\left(n, \frac{2}{m}\right)+\cdots+f\left(n, \frac{m n-1}{m}\right)$ 的值.
['对 $k=0,1, \\cdots, m-1$, 有\n\n$\\sum_{i=1}^{n-1} f\\left(m, k+\\frac{i}{n}\\right)=\\left(\\begin{array}{c}m \\\\ k\\end{array}\\right) \\cdot \\sum_{i=1}^{n=1}\\left(1-\\frac{i}{n}\\right)+\\left(\\begin{array}{c}m \\\\ k+1\\end{array}\\right) \\cdot \\sum_{i=1}^{n-1} \\frac{i}{n}=\\frac{n-1}{2} \\cdot\\left(\\begin{array}{c}m \\\\ k\\end{array}\\right)+\\left(\\begin{array}{c}m \\\\ k+1\\end{array}\\right)$,\n\n于是\n\n$$\n\\begin{aligned}\ng(m, n) & =f\\left(m, \\frac{1}{n}\\right)+f\\left(m, \\frac{2}{n}\\right)+\\cdots+f\\left(m, \\frac{m n-1}{n}\\right) \\\\\n& =\\sum_{j=1}^{m-1}\\left(\\begin{array}{c}\nm \\\\\nj\n\\end{array}\\right)+\\sum_{k=0}^{m-1} \\sum_{i=1}^{n-1} f\\left(m, k+\\frac{i}{n}\\right) \\\\\n& =2^{m}-2+\\frac{n-1}{2} \\cdot\\left(\\sum_{k=0}^{m-1}\\left(\\begin{array}{c}\nm \\\\\nk\n\\end{array}\\right)+\\sum_{k=0}^{m-1}\\left(\\begin{array}{c}\nm \\\\\nk+1\n\\end{array}\\right)\\right) \\\\\n& =2^{m}-2+\\frac{n-1}{2} \\cdot 2^{m}-1+2^{m}-1 \\\\\n& =\\left(2^{m}-1\\right) n-1,\n\\end{aligned}\n$$\n\n根据题意,有\n\n$$\n\\left(2^{m}-1\\right) n-1=123 \\Longleftrightarrow\\left(2^{m}-1\\right) n=2^{2} \\cdot 31,\n$$\n\n于是 $m=5, n=4$, 从而\n\n$$\n\\begin{aligned}\nf\\left(n, \\frac{1}{m}\\right)+f\\left(n, \\frac{2}{m}\\right)+\\cdots+f\\left(n, \\frac{m n-1}{m}\\right) & =g(n, m) \\\\\n& =\\left(2^{n}-1\\right) m-1 \\\\\n& =\\left(2^{4}-1\\right) \\cdot 5-1 \\\\\n& =74 .\n\\end{aligned}\n$$']
['$74$']
false
null
Numerical
null
2
Plane Geometry
null
在平面直角坐标系中, 点 $A, B, C$ 在双曲线 $x y=1$ 上, 满足 $\triangle A B C$ 为等腰直角三角形. 求 $\triangle A B C$ 的面积的最小值.
['设 $A\\left(a, \\frac{1}{a}\\right)$, 平移坐标系使 $A$ 为原点, 此时双曲线方程为\n\n$$\nH^{\\prime}:(x+a)\\left(x+\\frac{1}{a}\\right)=1 \\Longleftrightarrow x y+\\frac{1}{a} x+a y=0,\n$$\n\n设 $B^{\\prime}(\\theta: r), C^{\\prime}\\left(\\theta+\\frac{\\pi}{2}: r\\right)$, 其中 $r>0$, 则 $\\triangle A B C$ 的面积 $S=\\frac{1}{2} r^{2}$. 根据题意, 有\n\n$$\n\\left\\{\\begin{array}{l}\nr^{2} \\sin \\theta \\cos \\theta+\\frac{r \\cos \\theta}{a}+a r \\sin \\theta=0, \\\\\n-r^{2} \\sin \\theta \\cos \\theta-\\frac{r \\sin \\theta}{a}+a r \\cos \\theta=0,\n\\end{array}\\right.\n$$\n\n\n\n于是\n\n$$\nr=\\frac{-a \\sin \\theta-\\frac{1}{a} \\cos \\theta}{\\sin \\theta \\cos \\theta}=\\frac{a \\cos \\theta-\\frac{1}{a} \\sin \\theta}{\\sin \\theta \\cos \\theta},\n$$\n\n从而可得\n\n$$\na^{2}=\\frac{\\sin \\theta-\\cos \\theta}{\\sin \\theta+\\cos \\theta}\n$$\n\n因此\n\n$$\nr^{2}=\\frac{a^{2} \\sin ^{2} \\theta+2 \\sin \\theta \\cos \\theta+\\frac{1}{a^{2}} \\cos ^{2} \\theta}{\\sin ^{2} \\theta \\cos ^{2} \\theta}=\\frac{a^{2} \\cos ^{2} \\theta-2 \\sin \\theta \\cos \\theta+\\frac{1}{a^{2}} \\sin ^{2} \\theta}{\\sin ^{2} \\theta \\cos ^{2} \\theta},\n$$\n\n进而\n\n$$\n\\begin{aligned}\nS & =\\frac{1}{2} r^{2} \\\\\n& =\\frac{a^{2}+\\frac{1}{a^{2}}}{2 \\sin ^{2} \\theta \\cos ^{2} \\theta} \\\\\n& =\\frac{2}{\\sin ^{2} 2 \\theta \\cos 2 \\theta} \\\\\n& =\\frac{2 \\sqrt{2}}{\\sqrt{\\left(1-\\cos ^{2} 2 \\theta\\right)^{2} \\cdot 2 \\cos ^{2} 2 \\theta}} \\\\\n& \\geqslant \\frac{2 \\sqrt{2}}{\\sqrt{\\left(\\frac{2}{3}\\right)^{3}}}=3 \\sqrt{3},\n\\end{aligned}\n$$\n\n等号当 $\\cos ^{2} 2 \\theta=\\frac{1}{3}$ 时取得, 因此所求面积的最小值为 $3 \\sqrt{3}$.']
['$3 \\sqrt{3}$']
false
null
Numerical
null
3
Plane Geometry
null
如图, 在等腰 $\triangle A B C$ 中, $A B=B C, I$ 为内心, $M$ 为 $B I$ 的中点, $P$ 为边 $A C$ 上一点, 满足 $A P=3 P C, P I$ 延长线上一点 $H$ 满足 $M H \perp P H, Q$ 为 $\triangle A B C$ 的外接圆上劣弧 $A B$ 的中点. 证明: $B H \perp Q H$. <img_4057>
['取 $A C$ 的中点 $N$, 链接 $IN, BQ, QC$, 如下图. \n\n<img_4177>\n\n由 $A P=3 P C$, 可知 $P$ 为 $N C$ 的中点.易知 $B, I, N$ 共线, $\\angle I N C=90^{\\circ}$.由 $I$ 为 $\\triangle A B C$ 的内心, 可知 $C I$ 经过点 $Q$, 且\n\n$$\n\\angle Q I B=\\angle I B C+\\angle I C B=\\angle A B I+\\angle A C Q=\\angle A B I+\\angle A B Q=\\angle Q B I,\n$$\n\n又 $M$ 为 $B I$ 的中点, 所以 $Q M \\perp B I$, 进而 $Q M \\| C N$.\n\n考虑 $\\triangle H M Q$ 和 $\\triangle H I B$. 由于 $M H \\perp P H$, 故\n\n$$\n\\angle H M Q=90^{\\circ}-\\angle H M I=\\angle H I B .\n$$\n\n又 $\\angle I H M=\\angle I N P=90^{\\circ}$, 故 $\\frac{H M}{H I}=\\frac{N P}{N I}$, 于是\n\n$$\n\\frac{H M}{H I}=\\frac{N P}{N I}=\\frac{1}{2} \\cdot \\frac{N C}{N I}=\\frac{1}{2} \\cdot \\frac{M Q}{M I}=\\frac{M Q}{I B}\n$$\n\n所以 $\\triangle H M Q \\sim \\triangle H I B$, 得 $\\angle H Q M=\\angle H B I$. 从而 $H, M, B, Q$ 四点共圆, 于是有\n\n$$\n\\angle B H Q=\\angle B M Q=90^{\\circ} \\text {, }\n$$\n\n即 $B H \\perp Q H$.']
null
false
null
null
null
3
Plane Geometry
null
如图, 在等腰 $\triangle A B C$ 中, $A B=B C, I$ 为内心, $M$ 为 $B I$ 的中点, $P$ 为边 $A C$ 上一点, 满足 $A P=3 P C, P I$ 延长线上一点 $H$ 满足 $M H \perp P H, Q$ 为 $\triangle A B C$ 的外接圆上劣弧 $A B$ 的中点. 证明: $B H \perp Q H$. ![](https://cdn.mathpix.com/cropped/2023_12_20_0aa071b68ff410b45441g-1.jpg?height=537&width=508&top_left_y=871&top_left_x=768)
['取 $A C$ 的中点 $N$, 链接 $IN, BQ, QC$, 如下图. \n\n![](https://cdn.mathpix.com/cropped/2023_12_20_e190815bc9cade260ebeg-1.jpg?height=528&width=508&top_left_y=1872&top_left_x=771)\n\n由 $A P=3 P C$, 可知 $P$ 为 $N C$ 的中点.易知 $B, I, N$ 共线, $\\angle I N C=90^{\\circ}$.由 $I$ 为 $\\triangle A B C$ 的内心, 可知 $C I$ 经过点 $Q$, 且\n\n$$\n\\angle Q I B=\\angle I B C+\\angle I C B=\\angle A B I+\\angle A C Q=\\angle A B I+\\angle A B Q=\\angle Q B I,\n$$\n\n又 $M$ 为 $B I$ 的中点, 所以 $Q M \\perp B I$, 进而 $Q M \\| C N$.\n\n考虑 $\\triangle H M Q$ 和 $\\triangle H I B$. 由于 $M H \\perp P H$, 故\n\n$$\n\\angle H M Q=90^{\\circ}-\\angle H M I=\\angle H I B .\n$$\n\n又 $\\angle I H M=\\angle I N P=90^{\\circ}$, 故 $\\frac{H M}{H I}=\\frac{N P}{N I}$, 于是\n\n$$\n\\frac{H M}{H I}=\\frac{N P}{N I}=\\frac{1}{2} \\cdot \\frac{N C}{N I}=\\frac{1}{2} \\cdot \\frac{M Q}{M I}=\\frac{M Q}{I B}\n$$\n\n所以 $\\triangle H M Q \\sim \\triangle H I B$, 得 $\\angle H Q M=\\angle H B I$. 从而 $H, M, B, Q$ 四点共圆, 于是有\n\n$$\n\\angle B H Q=\\angle B M Q=90^{\\circ} \\text {, }\n$$\n\n即 $B H \\perp Q H$.']
['证明题']
false
null
Need_human_evaluate
null
4
Sequence
null
给定整数 $n \geqslant 3$, 设 $a_{1}, a_{2}, \cdots, a_{2 n}, b_{1}, b_{2}, \cdots, b_{2 n}$ 是 $4 n$ 个非负实数, 满足 $$ a_{1}+a_{2}+\cdots+a_{2 n}=b_{1}+b_{2}+\cdots+b_{2 n}>0 $$ 且对任意 $i=1,2, \cdots, 2 n$, 有 $a_{i} a_{i+2} \geqslant b_{i}+b_{i+1}$ (这里 $a_{2 n+1}=a_{1}, a_{2 n+2}=a_{2}$, $b_{2 n+1}=b_{1}$ ), 求 $a_{1}+a_{2}+\cdots+a_{2 n}$ 的最小值.
['在两组数之和相等的情况下由条件 $a_{i} a_{i+2} \\geqslant b_{i}+b_{i+1}$ 可以猜想两种情形: $2 \\cdot 2=2+2$以及 $0 \\cdot 0=0+0$. 记\n\n$$\n\\sum_{k=1}^{2 n} a_{k}=\\sum_{k=1}^{2 n} b_{k}=S, \\quad \\sum_{k=1}^{n} a_{2 k-1}=T\n$$\n\n不妨设 $T \\leqslant \\frac{1}{2} S$. 情形一 $n=3$. 取 $a_{i}=b_{i}=2(i=1,2, \\cdots, 2 n)$, 可得 $S=12$. 而\n\n$$\nT^{2}-3\\left(a_{1} a_{3}+a_{3} a_{5}+a_{5} a_{1}\\right)=\\frac{1}{2}\\left(\\left(a_{1}-a_{3}\\right)^{2}+\\left(a_{3}-a_{5}\\right)^{2}+\\left(a_{5}-a_{1}\\right)^{2}\\right) \\geqslant 0\n$$\n\n于是\n\n$$\n\\frac{S^{2}}{4} \\geqslant T^{2} \\geqslant 3\\left(a_{1} a_{3}+a_{3} a_{5}+a_{5} a_{1}\\right) \\geqslant 3 \\sum_{k=1}^{6} b_{k}=3 S \\Longrightarrow S \\geqslant 12\n$$\n\n因此 $S$ 的最小值为 12 . 情形二 $n \\geqslant 4$. 此时若 $n$ 为偶数, 则\n\n$$\n\\sum_{k=1}^{n} a_{2 k-1} a_{2 k+1} \\leqslant\\left(a_{1}+a_{5}+\\cdots+a_{2 n-3}\\right)\\left(a_{3}+a_{7}+\\cdots+a_{2 n-1}\\right) \\leqslant \\frac{T^{2}}{4}\n$$\n\n若 $n$ 为奇数, 则\n\n$$\n\\begin{aligned}\n\\sum_{k=1}^{n} a_{2 k-1} a_{2 k+1} & \\leqslant\\left(\\sum_{k=1}^{n-1} a_{2 k-1} a_{2 k+1}\\right)+a_{2 n-1} a_{3} \\\\\n& \\leqslant\\left(a_{1}+a_{5}+\\cdots+a_{2 n-1}\\right)\\left(a_{3}+a_{7}+\\cdots+a_{2 n-3}\\right) \\\\\n& \\leqslant \\frac{T^{2}}{4},\n\\end{aligned}\n$$\n\n\n\n于是\n\n$$\nS \\leqslant \\sum_{k=1}^{n} a_{2 k-1} a_{2 k+1} \\leqslant \\frac{T^{2}}{4} \\leqslant \\frac{S^{2}}{16} \\Longrightarrow S \\geqslant 16\n$$\n\n取\n\n| $k$ | 1 | 2 | 3 | 4 | 5 | 6 | $\\cdots$ | $2 n$ |\n| :---: | :---: | :---: | :---: | :---: | :---: | :---: | :---: | :---: |\n| $a_{k}$ | 4 | 4 | 4 | 4 | 0 | 0 | $\\cdots$ | 0 |\n| $b_{k}$ | 0 | 16 | 0 | 0 | 0 | 0 | $\\cdots$ | 0 |\n\n可得 $S=16$. 因此 $S$ 的最小值为 16 .']
['$12 , 16$']
true
null
Numerical
null
5
Sequence
null
设 $a_{1}=1, a_{2}=2, a_{n}=2 a_{n-1}+a_{n-2}, n=3,4, \cdots$. 证明: 对整数 $n \geqslant 5, a_{n}$ 必有一个模 4 余 1 的素因子.
['记 $\\alpha=1+\\sqrt{2}, \\beta=1-\\sqrt{2}$, 则\n\n$$\na_{n}=\\frac{\\alpha^{n}-\\beta^{n}}{\\alpha-\\beta}=\\frac{1}{2 \\sqrt{2}}\\left(\\alpha^{n}-\\beta^{n}\\right)\n$$\n\n构造\n\n$$\nb_{n}=\\frac{\\alpha^{n}+\\beta^{n}}{\\alpha+\\beta}=\\frac{1}{2}\\left(\\alpha^{n}+\\beta^{n}\\right)\n$$\n\n则\n\n$$\nb_{n}=2 b_{n-1}+b_{n-2}, n \\in \\mathbb{Z}\n$$\n\n且 $b_{1}=1, b_{2}=3$. 易知 $b_{n} \\in \\mathbb{Z}$ 且\n\n$$\nb_{n}^{2}-2 a_{n}^{2}=\\left(\\frac{\\alpha^{n}+\\beta^{n}}{2}\\right)^{2}-\\left(\\frac{\\alpha^{n}-\\beta^{n}}{2}\\right)^{2}=(\\alpha \\beta)^{n}=(-1)^{n}, \\quad n \\geqslant 1\n$$\n\n给出引理: 若正整数 $m, n$ 满足 $m \\mid n$, 则 $a_{m} \\mid a_{n}$. 引理的证明: 设 $n=k m$, 其中 $k \\in \\mathbb{N}^{*}$, 则\n\n$$\n\\begin{aligned}\n\\frac{a_{n}}{a_{m}} & =\\frac{\\alpha^{k m}-\\beta^{k m}}{\\alpha^{m}-\\beta^{m}} \\\\\n& =\\sum_{i=0}^{k}\\left(\\alpha^{m}\\right)^{k-i}\\left(\\beta^{m}\\right)^{i} \\\\\n& =\\frac{1}{2} \\sum_{i=0}^{k}\\left(\\left(\\alpha^{m}\\right)^{k-i}\\left(\\beta^{m}\\right)^{i}+\\left(\\alpha^{m}\\right)^{i}\\left(\\beta^{m}\\right)^{k-i}\\right) \\\\\n& =\\frac{1}{2} \\sum_{i=1}^{k}\\left((\\alpha \\beta)^{m i}\\left(\\alpha^{m(k-2 i)}+\\beta^{m(k-2 i)}\\right)\\right) \\\\\n& =\\frac{1}{2} \\sum_{i=1}^{k}\\left((-1)^{m i} \\cdot 2 b_{m(k-2 i)}\\right) \\in \\mathbb{Z},\n\\end{aligned}\n$$\n\n\n\n于是 $a_{m} \\mid a_{n}$. 情形一 $n$ 的最大奇数约数为 1 . 此时 $n=2^{l}(l \\geqslant 3)$, 由于\n\n$$\na_{8}=408=24 \\cdot 17, \\quad 17 \\equiv 1 \\quad(\\bmod 4)\n$$\n\n于是由 $a_{8} \\mid a_{n}$ 可得 $a_{n}$ 有素因子 17 . 情形二 $n$ 的最大奇数约数 $m$ 不小于 3 . 此时有 $a_{m} \\mid a_{n}$, 只需要考虑 $a_{m}$ 有一个模 4 余 1 的素因子. 由 $a_{1}$ 为奇数推得 $a_{m}$ 为不小于 5 的奇数, 因此 $a_{m}$ 有素因子 $p$. 从而\n\n$$\nb_{m}^{2}-2 a_{m}^{2}=(-1)^{m} \\Longrightarrow b_{m}^{2} \\equiv-1 \\quad(\\bmod p) \\Longrightarrow b_{n}^{p-1} \\equiv(-1)^{\\frac{p-1}{2}} \\quad(\\bmod p)\n$$\n\n又 $\\left(p, b_{m}\\right)=1$, 由费马小定理可得 $b_{m}^{p-1} \\equiv 1(\\bmod p)$, 从而\n\n$$\n(-1)^{\\frac{p-1}{2}} \\equiv 1 \\quad(\\bmod p) \\Longrightarrow(-1)^{\\frac{p-1}{2}}=1 \\Longrightarrow p \\equiv 1 \\quad(\\bmod 4)\n$$\n\n综上所述,命题得证.']
null
false
null
null
null
6
Combinatorics
null
给定凸 20 边形 $P$. 用 $P$ 的 17 条在内部不相交的对角线将 $P$ 分割成 18 个三角形. 所得图形称为 $P$ 的一个三角剖分图. 对 $P$ 的任意一个三角剖分图 $T, P$ 的 20 条边以及添加的 17 条对角线均称为 $T$ 的边. $T$ 的任意 10 条两两无公共端点的边的集合称为 $T$的一个完美匹配. 当 $T$ 取遍 $P$ 的所有三角剖分图时, 求 $T$ 的完美匹配个数的最大值.
['将 20 边形换成 $2 n$ 边形,考虑一般的问题.\n\n对凸 $2 n$ 边形 $P$ 的一条对角线, 若其两侧各有奇数个 $P$ 的顶点, 称其为奇弦, 否则称为偶弦. 首先注意下述基本事实:\n\n引理一: 对 $P$ 的任意三角剖分图 $T, T$ 的完美匹配不含奇弦.\n\n引理一的证明: 如果完美匹配中有一条奇弦 $e_{1}$, 因为 $T$ 的一个完美匹配给出了 $P$ 的顶点集的一个配对划分, 而 $e_{1}$ 两侧各有奇数个顶点, 故该完美匹配中必有 $T$ 的另一条边 $e_{1}$, 端点分别在 $e_{1}$ 的两侧, 又 $P$ 是凸多边形, 故 $e_{1}$ 与 $e$ 在 $P$ 的内部相交, 这与 $T$是三角剖分图矛盾. 记 $f(T)$ 为 $T$ 的完美匹配的个数. 设 $F_{1}=1, F_{2}=2$, 对 $k \\geqslant 2$, $F_{k+2}=F_{k+1}+F_{k}$ 是 Fibonacci 数列. 下面对 $n$ 归纳证明:\n\n引理二: 若 $T$ 是凸 $2 n$ 边形的任意一个三角剖分图, 则 $f(T) \\leqslant F_{n}$.\n\n引理二的证明: 设 $P=A_{1} A_{2} \\cdots A_{2 n}$ 是凸 $2 n$ 边形. 从 $P$ 的 $2 n$ 条边中选 $n$ 条边构成完美匹配,恰有两种方法, $A_{1} A_{2}, A_{1} A_{4}, \\cdots, A_{2 n-1} A_{2 n}$ 或 $A_{2} A_{3}, A_{4} A_{5}, \\cdots, A_{2 n-2} A_{2 n-1}, A_{2 n} A_{1}$.当 $n=2$ 时, 凸四边形 $P$ 的三角剖分图 $T$ 没有偶弦, 因此 $T$ 的完美匹配只能用 $P$ 的边, 故 $f(T)=2=F_{2}$. 当 $n=3$ 时, 凸六边形 $P$ 的三角剖分图 $T$ 至多有一条偶弦. 若 $T$ 没有偶弦, 同上可知 $f(T)=2$. 若 $T$ 含有偶弦, 不妨设是 $A_{1} A_{4}$, 选用 $A_{1} A_{4}$ 的完美匹配是唯一的, 另两条边只能是 $A_{2} A_{3}, A_{3} A_{6}$, 此时 $f(T)=3$. 总之 $f(T) \\leqslant 3=F_{3}$.结论在 $n=2,3$ 时成立.\n\n假设 $n \\geqslant 4$, 且结论在小于 $n$ 时均成立. 考虑凸 $2 n$ 边形 $P=A_{1} A_{2} \\cdots A_{2 n}$ 的一个三角剖分图 $T$. 若 $T$ 没有偶弦, 则同上可知 $f(T)=2$. 对于偶弦 $e$, 记 $e$ 两侧中 $P$ 的顶点个数的较小值为 $w(e)$, 若 $T$ 含有偶弦, 取其中一条偶弦 $e$ 使 $w(e)$ 达到最小. 设 $w(e)=2 k$, 不妨设 $e$ 为 $A_{2 n} A_{2 k+1}$, 则每个 $A_{i}(i=1,2, \\cdots, 2 k)$ 不能引出偶弦.\n\n事实上, 假设 $A_{i} A_{j}$ 是偶弦, 若 $j \\in\\{2 k+2,2 k+3, \\cdots, 2 n-1\\}$, 则 $A_{i} A_{j}$ 与 $e$ 在 $P$ 的内部相交, 矛盾. 若 $j \\in\\{1,2, \\cdots, 2 k+1,2 n\\}$, 则 $w\\left(A_{i} A_{j}\\right)<2 k$, 与 $w(e)$ 的最小性矛盾. 又完美匹配中没有奇弦, 故 $A_{1}, A_{2}, \\cdots, A_{2 k}$ 只能与其相邻顶点配对, 特别地, $A_{1}$只能与 $A_{2}$ 或 $A_{2 n}$ 配对. 下面分两种情况.\n\n情形一选用边 $A_{1} A_{2}$. 则必须选用边 $A_{3} A_{4}, \\cdots, A_{2 k-1} A_{2 k}$. 注意到 $A_{2 n} A_{2 k+1}$ 的两侧分别有 $2 k, 2 n-2 k-2$ 个顶点, $2 n-2 k-2 \\geqslant w\\left(A_{2 n} A_{2 k+1}\\right)=2 k$, 而 $n \\geqslant 4$, 因此 $2 n-2 k \\geqslant 6$. 在凸 $2 n-2 k$ 边形 $P_{1}=A_{2 k+1} A_{2 k+2} \\cdots A_{2 n}$ 上, $T$ 的边给出了 $P_{1}$ 的三角剖分图 $T_{1}$, 在 $T$ 中再选取 $n-k$ 条边 $e_{1}, e_{2}, \\cdots, e_{n-k}$, 与 $A_{1} A_{2}, A_{3} A_{4}, \\cdots, A_{2 k-1} A_{2 k}$一起构成 $T$ 的完美匹配, 当且仅当 $e_{1}, e_{2}, \\cdots, e_{n-k}$ 是 $T_{1}$ 的完美匹配. 故情形一中的 $T$的完美匹配个数等于 $f\\left(T_{1}\\right)$.\n\n情形二选用边 $A_{1} A_{2 n}$. 则必须选用边 $A_{2} A_{3}, \\cdots, A_{2 k} A_{2 k+1}$. 在凸 $2 n-2 k-2$ 边形 $P_{2}=A_{2 k+2} A_{2 k+3} \\cdots A_{2 n-1}$ 中构造如下的三角剖分图 $T_{2}$ : 对 $2 k+2 \\leqslant i<j \\leqslant 2 n-1$,若线段 $A_{i} A_{j}$ 是 $T$ 的边, 则也将其作为 $T_{2}$ 的边, 由于这些边在内部互不相交, 因此可再适当地添加一些 $P_{2}$ 的对角线, 得到一个 $P_{2}$ 的三角剖分图 $T_{2}$, 它包含了 $T$ 的所有在顶点 $A_{2 k+2}, A_{2 k+3}, \\cdots, A_{2 n-1}$ 之间的边. 因此每个包含边 $A_{2 n} A_{1}, A_{2} A_{3}, \\cdots, A_{2 k} A_{2 k+1}$的 $T$ 的完美匹配, 其余的边必定是 $T_{2}$ 的完美匹配. 故情形二中的 $T$ 的完美匹配个数不超过 $f\\left(T_{2}\\right)$.\n\n由归纳假设得\n\n$$\nf\\left(T_{1}\\right) \\leqslant F_{n-k}, \\quad f\\left(T_{2}\\right) \\leqslant F_{n-k-1},\n$$\n\n结合上面两种情形以及 $k \\geqslant 1$, 有\n\n$$\nf(T) \\leqslant f\\left(T_{1}\\right)+f\\left(T_{2}\\right) \\leqslant F_{n-k}+F_{n-k-1}=F_{n-k+1} \\leqslant F_{n}\n$$\n\n下面说明等号可以取得. 考虑凸 $2 n$ 边形 $A_{1} A_{2} \\cdots A_{2 n}$ 的三角剖分图 $\\Delta_{n}$ : 添加对角线 $A_{2} A_{2 n}, A_{2 n} A_{3}, A_{3} A_{2 n-1}, A_{2 n-1} A_{4}, A_{4} A_{2 n-2}, \\cdots, A_{n+3} A_{n}, A_{n} A_{n+2}$. 重复前面的论证过程, 有\n\n$$\nf\\left(\\Delta_{2}\\right)=2, f\\left(\\Delta_{3}\\right)=3\n$$\n\n对 $\\Delta_{n}, n \\geqslant 4$, 考虑偶弦 $A_{n} A_{3}$. 情形一, 用 $A_{1} A_{2}$, 由于在凸 $2 n-2$ 边形 $A_{3} A_{4} \\cdots A_{2 \\pi}$中的三角剖分图恰是 $\\Delta_{n-1}$, 此时有 $f\\left(\\Delta_{n-1}\\right)$ 个 $T$ 的完美匹配. 情形二, 用 $A_{1} A_{2 n}$, 由于在凸 $2 n-4$ 边形 $A_{4} A_{5} \\cdots A_{2 n-1}$ 中 $T$ 的边恰构成三角剖分图 $\\Delta_{n-2}$, 不用添加任何对角线, 故这一情形下 $T$ 的完美匹配个数恰为 $f\\left(\\Delta_{n-2}\\right)$. 从而对 $n \\geqslant 4$, 有\n\n$$\nf\\left(\\Delta_{n}\\right)=f\\left(\\Delta_{n-1}\\right)+f\\left(\\Delta_{n-2}\\right) .\n$$\n\n\n\n由数学归纳法即得 $f\\left(\\Delta_{n}\\right)=F_{n}$. 结论得证. 因此, 对凸 20 边形 $P, f(T)$ 的最大值等于 $F_{10}=89$.']
['$89$']
false
null
Numerical
null
7
Plane Geometry
null
在椭圆中, $A$ 为长轴的一个端点, $B$ 为短轴的一个端点, $F_{1}, F_{2}$ 为两个焦点. 若 $$ \overline{A F_{1}} \cdot \overrightarrow{A F_{2}}+\overrightarrow{B F_{1}} \cdot \overrightarrow{B F_{2}}=0 $$ 求 $\tan \angle A B F_{1} \cdot \tan \angle A B F_{2}$ 的值.
['设 $A$ 为右顶点, $B$ 为上顶点, $F_{1}, F_{2}$ 分别为左、右焦点, $O$ 为 $F_{1} F_{2}$ 的中点.\n\n<img_4141>\n\n根据极化恒等式,有\n\n$$\n\\overline{A F_{1}} \\cdot \\overrightarrow{A F_{2}}+\\overrightarrow{B F_{1}} \\cdot \\overrightarrow{B F_{2}}=0 \\Longleftrightarrow\\left(A O^{2}-\\frac{1}{4} F_{1} F_{2}^{2}\\right)+\\left(B O^{2}-\\frac{1}{4} F_{1} F_{2}^{2}\\right)=0,\n$$\n\n因此 $O A^{2}+O B^{2}=2 O F^{2}$, 又 $O B^{2}+O F^{2}=O A^{2}$, 因此\n\n$$\nO A: O B: O F=\\sqrt{3}: 1: \\sqrt{2}\n$$\n\n于是\n\n$$\n\\begin{aligned}\n\\tan \\angle A B F_{1} \\cdot \\tan \\angle A B F_{2} & =\\tan \\left(\\angle A B O+\\angle O B F_{1}\\right) \\cdot \\tan \\left(\\angle A B O-\\angle O B F_{1}\\right) \\\\\n& =\\frac{\\sqrt{3}+\\sqrt{2}}{1-\\sqrt{3} \\cdot \\sqrt{2}} \\cdot \\frac{\\sqrt{3}-\\sqrt{2}}{1+\\sqrt{3} \\cdot \\sqrt{2}} \\\\\n& =-\\frac{1}{5}\n\\end{aligned}\n$$']
['$-\\frac{1}{5}$']
false
null
Numerical
null
8
Algebra
null
设正实数 $a, b, c$ 满足 $$ a^{2}+4 b^{2}+9 c^{2}=4 b+12 c-2 $$ 求 $\frac{1}{a}+\frac{2}{b}+\frac{3}{c}$ 的最小值.
['根据题意, 有\n\n$$\na^{2}+(2 b-1)^{2}+(3 c-2)^{2}=3\n$$\n\n于是\n\n$$\n\\begin{aligned}\n\\frac{1}{a}+\\frac{2}{b}+\\frac{3}{c} & \\geqslant \\frac{(1+2+3)^{2}}{a+2 b+3 c} \\\\\n& =\\frac{36}{a+(2 b-1)+(3 c-2)+3} \\\\\n& \\geqslant \\frac{36}{\\sqrt{3} \\cdot \\sqrt{a^{2}+(2 b-1)^{2}+(3 c-2)^{2}}+3} \\\\\n& =6,\n\\end{aligned}\n$$\n\n等号当 $a=b=c=1$ 时取得, 因此所求最小值为 6 .']
['$6$']
false
null
Numerical
null
9
Sequence
null
设数列 $a_{n}$ 的通项公式为 $$ a_{n}=\frac{1}{\sqrt{5}}\left(\left(\frac{1+\sqrt{5}}{2}\right)^{n}-\left(\frac{1-\sqrt{5}}{2}\right)^{n}\right), n=1,2, \cdots $$ 证明: 存在无穷多个正整数 $m$, 使得 $a_{m+4} a_{m}-1$ 是完全平方数.
['根据求数列通项的特征根法, 有\n\n$$\na_{n+2}=a_{n+1}+a_{n}, n \\in \\mathbb{N}^{*}\n$$\n\n且 $a_{1}=a_{2}=1$. 观察数列 $\\left\\{a_{n}\\right\\}$ 的通项\n\n| $n$ | 1 | 2 | 3 | 4 | 5 | 6 | 7 | 8 | 9 | 10 |\n| :---: | :---: | :---: | :---: | :---: | :---: | :---: | :---: | :---: | :---: | :---: |\n| $a_{n}$ | 1 | 1 | 2 | 3 | 5 | 8 | 13 | 21 | 34 | 55 |\n\n尝试证明\n\n$$\na_{k+2}^{2}-a_{k} a_{k+4}=(-1)^{k}\n$$\n\n注意到 $a_{3}^{2}-a_{1} a_{5}=-1$, 只需要证明\n\n$$\na_{k+4}^{2}-a_{k+2} a_{k+6}=(-1)\\left(a_{k+2}^{2}-a_{k} a_{k+4}\\right)\n$$\n\n也即\n\n$$\n\\frac{a_{k+6}+a_{k+2}}{a_{k+4}}=\\frac{a_{k+4}+a_{k}}{a_{k+2}}\n$$\n\n而\n\n$$\na_{k+4}=a_{k+3}+a_{k+2}=2 a_{k+2}+a_{k+1}=3 a_{k+2}-a_{k} \\Longrightarrow \\frac{a_{k+4}+a_{k}}{a_{k+2}}=3\n$$\n\n因此命题得证. 综上所述, 对于所有正奇数 $m$, 都有 $a_{m+4} a_{m}-1=a_{m+2}^{2}$ 为完全平方数.']
null
false
null
null
null
10
Plane Geometry
null
如图, $A, B, C, D, E$ 是圆 $\Omega$ 上顺次的五点, 满足 $A B C=B C D=C D E$, 点 $P, Q$ 分别在线段 $A D, B E$ 上, 且 $P$ 在线段 $C Q$ 上. 证明: $\angle P A Q=\angle P E Q$. <img_4183>
['设 $A D$ 与 $B E$ 交于点 $S, C Q$ 的延长线交圆 $\\Omega$ 于 $T$, 如下图.\n\n<img_4200>\n\n注意到 $A B C=B C D=C D E$, 因此 $A B, C D$ 所对的圆周角相等, 设为 $\\alpha, B C, D E$所对的圆周角相等, 设为 $\\beta$. 于是\n\n$$\n\\begin{aligned}\n& \\angle A T Q=\\angle A T C=\\alpha+\\beta, \\\\\n& \\angle P T E=\\angle C T E=\\alpha+\\beta, \\\\\n& \\angle P S Q=\\angle B D A+\\angle D B E=\\alpha+\\beta .\n\\end{aligned}\n$$\n\n由 $\\angle A T Q=\\angle P S Q$ 可得 $S, A, T, Q$ 四点共圆, 由 $\\angle P T E=\\angle P S Q$ 可得 $P, S, T, E$ 四点共圆. 所以\n\n$$\n\\angle P A Q=\\angle P T S=\\angle P E Q\n$$']
null
false
null
null
null
10
Plane Geometry
null
如图, $A, B, C, D, E$ 是圆 $\Omega$ 上顺次的五点, 满足 $A B C=B C D=C D E$, 点 $P, Q$ 分别在线段 $A D, B E$ 上, 且 $P$ 在线段 $C Q$ 上. 证明: $\angle P A Q=\angle P E Q$. ![](https://cdn.mathpix.com/cropped/2023_12_20_8925a4423180c575195cg-1.jpg?height=457&width=471&top_left_y=777&top_left_x=838)
['设 $A D$ 与 $B E$ 交于点 $S, C Q$ 的延长线交圆 $\\Omega$ 于 $T$, 如下图.\n\n![](https://cdn.mathpix.com/cropped/2023_12_20_efae274ae8a76abcdf72g-1.jpg?height=482&width=489&top_left_y=1381&top_left_x=835)\n\n注意到 $A B C=B C D=C D E$, 因此 $A B, C D$ 所对的圆周角相等, 设为 $\\alpha, B C, D E$所对的圆周角相等, 设为 $\\beta$. 于是\n\n$$\n\\begin{aligned}\n& \\angle A T Q=\\angle A T C=\\alpha+\\beta, \\\\\n& \\angle P T E=\\angle C T E=\\alpha+\\beta, \\\\\n& \\angle P S Q=\\angle B D A+\\angle D B E=\\alpha+\\beta .\n\\end{aligned}\n$$\n\n由 $\\angle A T Q=\\angle P S Q$ 可得 $S, A, T, Q$ 四点共圆, 由 $\\angle P T E=\\angle P S Q$ 可得 $P, S, T, E$ 四点共圆. 所以\n\n$$\n\\angle P A Q=\\angle P T S=\\angle P E Q\n$$']
['证明题']
false
null
Need_human_evaluate
null
11
Set Theory
null
设集合 $A=\{1,2, \cdots, 19\}$. 证明:存在集合 $A$ 的非空子集 $S_{1}, S_{2}$, 满足 (1) $S_{1} \cap S_{2}=\varnothing$, $S_{1} \cup S_{2}=A$; (2) $S_{1}, S_{2}$ 都至少有 4 个元素; (3) $S_{1}$ 的所有元素的和等于 $S_{2}$ 的所有元素的乘积.
['$A$ 中所有元素之和为 190, 而 $6 !=720$, 因此 $S_{2}$ 中至多有 5 个元素. 验证\n\n$$\n\\{1,2,3,5,6\\},\\{1,2,3,4,6\\},\\{1,2,3,4,5\\}\n$$\n\n可知不存在符合题意的 5 元集合 $S_{2}$. 考虑 4 元集合 $S_{2}=\\{a, b, x, y\\}$, 且 $a<b<x<y$,则\n\n$$\na+b+x+y+a b x y=190, \\Longleftrightarrow(a b x+1)(a b y+1)=a b(190-a-b)+1,\n$$\n\n讨论如下\n\n| $a$ | $b$ | 方程 | $(x, y)$ |\n| :---: | :---: | :---: | :---: |\n| 1 | 2 | $(2 x+1)(2 y+1)=3 \\cdot 5^{2}$ | $(7,12)$ |\n| 1 | 3 | $(3 x+1)(3 y+1)=13 \\cdot 43$ | $(4,14)$ |\n| 1 | 4 | $(4 x+1)(4 y+1)=3 \\cdot 13 \\cdot 19$ | 无解 |\n| 2 | 3 | $(6 x+1)(6 y+1)=11 \\cdot 101$ | 无解 |\n\n其他情形下 $a b x y>190$, 无解. 因此存在符合题意的 $S_{1}, S_{2}$, 所有的 $S_{2}=\\{1,2,7,12\\}$或 $\\{1,3,4,14\\}, S_{1}=A \\backslash S_{2}$.']
null
false
null
null
null
12
Algebra
null
给定整数 $n \geqslant 2$. 设 $a_{1}, a_{2}, \cdots, a_{n}, b_{1}, b_{2}, \cdots, b_{n}>0$, 满足 $$ a_{1}+a_{2}+\cdots+a_{n}=b_{1}+b_{2}+\cdots+b_{n} $$ 且对任意 $i, j(1 \leqslant i<j \leqslant n)$ 均有 $a_{i} a_{j} \geqslant b_{i}+b_{j}$. 求 $a_{1}+a_{2}+\cdots+a_{n}$ 的最小值.
['设 $S=a_{1}+a_{2}+\\cdots+a_{n}=b_{1}+b_{2}+\\cdots+b_{n}$, 则有\n\n$$\n\\sum_{1 \\leqslant i<j \\leqslant n} a_{i} a_{j} \\geqslant \\sum_{1 \\leqslant i<j \\leqslant n}\\left(b_{i}+b_{j}\\right)=(n-1) S\n$$\n\n又\n\n$$\n\\sum_{1 \\leqslant i<j \\leqslant n} a_{i} a_{j} \\leqslant \\sum_{1 \\leqslant i<j \\leqslant n} \\frac{a_{i}^{2}+a_{j}^{2}}{2}=\\frac{n-1}{2} \\cdot \\sum_{i=1}^{n} a_{i}^{2}\n$$\n\n于是\n\n$$\nS^{2}=\\left(\\sum_{i=1}^{n} a_{i}\\right)^{2}=\\sum_{i=1}^{n} a_{i}^{2}+2 \\sum_{1 \\leqslant i<j \\leqslant n} a_{i} a_{j} \\geqslant\\left(\\frac{2}{n-1}+2\\right) \\sum_{1 \\leqslant j<j \\leqslant n} a_{i} a_{j} \\geqslant 2 n S,\n$$\n\n从而 $S \\geqslant 2 n$, 又当 $a_{i}=b_{i}=2(i=1,2, \\cdots, n)$ 时, 有 $S=2 n$,']
['$2 n$']
false
null
Expression
null
13
Number Theory
null
设 $a, b$ 为不超过 12 的正整数, 满足: 存在常数 $C$, 使得 $a^{n}+b^{n+9} \equiv C(\bmod 13)$ 对任意正整数 $n$ 成立. 求所有满足条件的有序数对 $(a, b)$.
['根据题意, 对任意正整数 $n$, 有\n\n$$\na^{n}+b^{n+9} \\equiv a^{n+3}+b^{n+12} \\quad(\\bmod 13)\n$$\n\n注意到 13 为素数, $a, b$ 均与 13 互素, 由费马小定理, 可得\n\n$$\na^{12} \\equiv b^{12} \\equiv 1 \\quad(\\bmod 13)\n$$\n\n因此取 $n=12$, 化简可得\n\n$$\n1+b^{9} \\equiv a^{3}+1 \\quad(\\bmod 13)\n$$\n\n故\n\n$$\nb^{9} \\equiv a^{3} \\quad(\\bmod 13)\n$$\n\n代入可得\n\n$$\na^{n}+a^{3} b^{n} \\equiv a^{n+3}+b^{n+12} \\equiv a^{n+3}+b^{n} \\quad(\\bmod 13)\n$$\n\n即\n\n$$\n\\left(a^{n}-b^{n}\\right)\\left(1-a^{3}\\right) \\equiv 0 \\quad(\\bmod 13) .\n$$\n\n情形一 $a^{3} \\equiv 1(\\bmod 13)$, 此时\n\n$$\nb^{3} \\equiv a^{3} b^{3} \\equiv b^{12} \\equiv 1 \\quad(\\bmod 13)\n$$\n\n又 $a, b \\in\\{1,2, \\cdots, 12\\}$, 经检验可知 $a, b \\in\\{1,3,9\\}$. 此时\n\n$$\na^{n}+b^{n+9} \\equiv a^{n}+b^{n} \\quad(\\bmod 13)\n$$\n\n由条件知\n\n$$\na+b \\equiv a^{3}+b^{3} \\equiv 3 \\quad(\\bmod 13)\n$$\n\n从而 $a=b=1$, 经检验, $(a, b)=(1,1)$ 符合题意. \n\n情形二 $a^{3} \\not \\equiv 1(\\bmod 13)$, 则对任意正整数 $n$, 有\n\n$$\na^{n} \\equiv b^{n} \\quad(\\bmod 13)\n$$\n\n特别地, 有 $a \\equiv b(\\bmod 13)$, 故 $a=b$, 从而\n\n$$\na^{3} \\equiv b^{9}=a^{9} \\quad(\\bmod 13) \\Longleftrightarrow a^{3}\\left(a^{3}-1\\right)\\left(a^{3}+1\\right) \\equiv 0 \\quad(\\bmod 13)\n$$\n\n故\n\n$$\na^{3} \\equiv-1 \\quad(\\bmod 13) \\Longrightarrow a=4,10,12\n$$\n\n\n\n经检验, 当 $(a, b)=(4,4),(10,10),(12,12)$ 时, 对任意正整数 $n$ 有\n\n$$\na^{n}+b^{n+9}=a^{n}+a^{n+9}=a^{n}\\left(1+\\left(a^{3}\\right)^{3}\\right) \\equiv 0 \\quad(\\bmod 13),\n$$\n\n满足条件.']
['$(1,1),(4,4),(10,10),(12,12)$']
true
null
Tuple
null
14
Trigonometric Functions
null
在 $\triangle A B C$ 中, $B C=a, C A=b, A B=c$. 若 $b$ 是 $a$ 与 $c$ 的等比中项, 且 $\sin A$ 是 $\sin (B-A)$ 与 $\sin C$ 的等差中项, 求 $\cos B$ 的值.
['罗列条件 $b^{2}=a c$,\n\n\n\n$2 \\sin A=\\sin (B-A)+\\sin C=\\sin (B-A)+\\sin (B+A)=2 \\sin B \\cos A$.\n\n$\\cos A=\\frac{a}{b}=\\frac{b^{2}+c^{2}-a^{2}}{2 b c}$.\n\n即 $2 a c=b^{2}+c^{2}-a^{2}$, 由 $b^{2}=a c$\n\n可得 $c^{2}-c a-a^{2}=0$. 于是 $\\frac{c}{a}=\\frac{\\sqrt{5}+1}{2}$.\n\n于是 $b^{2}=\\frac{\\sqrt{5}+1}{2} a^{2}, c^{2}=\\frac{3+\\sqrt{5}}{2} a^{2}$.\n\n解得 $\\cos B=\\frac{a^{2}+c^{2}-b^{2}}{2 a c}=\\frac{\\sqrt{5}-1}{2}$.']
['$\\frac{\\sqrt{5}-1}{2}$']
false
null
Numerical
null
15
Plane Geometry
null
在平面直角坐标系 $x O y$ 中, 圆 $\Omega$ 与抛物线 $\Gamma: y^{2}=4 x$ 恰有一个公共点, 且圆 $\Omega$ 与 $x$轴相切于 $\Gamma$ 的焦点 $F$. 求圆 $\Gamma$ 的半径.
['设抛物线上的点为 $A\\left(x_{0}, y_{0}\\right)$, 则该点处的切线为 $y_{0} y=2\\left(x+x_{0}\\right)$,\n\n其法向量 $\\vec{n}=\\left(2,-y_{0}\\right)$\n\n则线段 $A F$ 的中垂线为 $\\left(x_{0}-1\\right)\\left(x-\\frac{x_{0}+1}{2}\\right)+y_{0}\\left(y-\\frac{y_{0}}{2}\\right)=0$.\n\n其与 $x=1$ 的交点为 $\\Omega\\left(1, \\frac{\\left(x_{0}+1\\right)^{2}}{2 y_{0}}\\right)$.\n\n向量 $\\overrightarrow{A \\Omega}=\\left(1-x_{0}, \\frac{\\left(x_{0}+1\\right)^{2}}{2 y_{0}}-y_{0}\\right)$,\n\n由 $\\overrightarrow{A \\Omega}$ 与 $\\vec{n}$ 共线解得 $x_{0}=\\frac{1}{3}, y_{0}= \\pm \\frac{2}{\\sqrt{3}}$.\n\n故 $r=\\frac{\\left(x_{0}+1\\right)^{2}}{2\\left|y_{0}\\right|}=\\frac{4 \\sqrt{3}}{9}$.']
['$\\frac{4 \\sqrt{3}}{9}$']
false
null
Numerical
null
16
Sequence
null
称一个复数列 $\left\{z_{n}\right\}$ 是” 有趣的”, 若 $\left|z_{1}\right|=1$, 且对任意正整数 $n$, 均有 $4 z_{n+1}^{2}+$ $2 z_{n} z_{n+1}+z_{n}^{2}=0$. 求最大的常数 $C$, 使得对一切有趣的数列 $\left\{z_{n}\right\}$ 及任意正整数 $m$, 均有 $\left|z_{1}+z_{2}+\cdots+z_{m}\right| \geqslant C$.
['根据题意, 有\n\n$$\nz_{n+1}=\\frac{-1 \\pm \\sqrt{3} \\mathrm{i}}{4} z_{n} \\Longleftrightarrow z_{n+1}=\\left( \\pm \\frac{2 \\pi}{3}: \\frac{1}{2}\\right) \\cdot z_{n}\n$$\n\n记 $q_{1}=\\left(\\frac{2 \\pi}{3}: \\frac{1}{2}\\right), q_{2}=\\left(-\\frac{2 \\pi}{3}: \\frac{1}{2}\\right)$. 取 $z_{1}=1$, 且\n\n$$\nz_{n+1}= \\begin{cases}q_{1} \\cdot z_{n}, & 2 \\nmid n \\\\ q_{2} \\cdot z_{n}, & 2 \\mid n\\end{cases}\n$$\n\n\n\n则\n\n$$\n\\begin{aligned}\n\\lim _{m \\rightarrow+\\infty}\\left(z_{1}+z_{2}+\\cdots+z_{m}\\right) & =(0: 1)+\\left(\\frac{2 \\pi}{3}: \\frac{1}{2}+\\frac{1}{8}+\\cdots\\right)+\\left(-\\frac{2 \\pi}{3}: \\frac{1}{4}+\\frac{1}{16}+\\cdots\\right) \\\\\n& =(0: 1)+\\left(\\frac{2 \\pi}{3}: \\frac{2}{3}\\right)+\\left(-\\frac{2 \\pi}{3}: \\frac{1}{3}\\right) \\\\\n& =\\frac{1}{2}+\\frac{\\sqrt{3}}{6} \\mathrm{i},\n\\end{aligned}\n$$\n\n因此\n\n$$\n\\lim _{m \\rightarrow+\\infty}\\left|z_{1}+z_{2}+\\cdots+z_{m}\\right|=\\frac{\\sqrt{3}}{3}\n$$\n\n接下来证明 $C=\\frac{\\sqrt{3}}{3}$ 符合题意. 考虑\n\n$$\n\\left|z_{n}+z_{n+1}\\right|=\\left|z_{n}\\right| \\cdot\\left|1+\\frac{z_{n+1}}{z_{n}}\\right|=\\frac{1}{2^{n-1}} \\cdot\\left|1+\\left( \\pm \\frac{2 \\pi}{3}: \\frac{1}{2}\\right)\\right|=\\frac{\\sqrt{3}}{2^{n}}\n$$\n\n于是当 $m=1$ 时, 有 $\\left|z_{1}\\right|=1>\\frac{\\sqrt{3}}{3}$; 当 $m$ 为偶数时, 有\n\n$$\n\\begin{aligned}\n\\left|z_{1}+z_{2}+\\cdots+z_{m}\\right| & \\geqslant\\left|z_{1}+z_{2}\\right|-\\left|z_{3}+z_{4}\\right|-\\cdots-\\left|z_{m-1}+z_{m}\\right| \\\\\n& >\\frac{\\sqrt{3}}{2}-\\left(\\frac{\\sqrt{3}}{8}+\\frac{\\sqrt{3}}{32}+\\cdots\\right) \\\\\n& =\\frac{\\sqrt{3}}{3}\n\\end{aligned}\n$$\n\n当 $m$ 为不小于 3 的奇数时, 有\n\n$$\n\\begin{aligned}\n\\left|z_{1}+z_{2}+\\cdots+z_{m}\\right| & \\geqslant\\left|z_{1}+z_{2}\\right|-\\left|z_{3}+z_{4}\\right|-\\cdots-\\left|z_{m-2}+z_{m-1}\\right|-\\left|z_{m}\\right| \\\\\n& >\\frac{\\sqrt{3}}{2}-\\left(\\frac{\\sqrt{3}}{8}+\\frac{\\sqrt{3}}{32}+\\cdots+\\frac{\\sqrt{3}}{2^{m-2}}\\right)-\\frac{1}{2^{m}} \\\\\n& >\\frac{\\sqrt{3}}{2}-\\left(\\frac{\\sqrt{3}}{8}+\\frac{\\sqrt{3}}{32}+\\cdots+\\frac{\\sqrt{3}}{2^{m-2}}+\\frac{\\sqrt{3}}{2^{m}}\\right) \\\\\n& >\\frac{\\sqrt{3}}{3},\n\\end{aligned}\n$$']
['$\\frac{\\sqrt{3}}{3}$']
false
null
Numerical
null
17
Plane Geometry
null
如图, 在锐角 $\triangle A B C$ 中, $M$ 是 $B C$ 边的中点, 点 $P$ 在 $\triangle A B C$ 内, 使得 $A P$ 平分 $\angle B A C$. 直线 $M P$ 与 $\triangle A B P, \triangle A C P$ 的外接圆分别相交于不同于点 $P$ 的两点 $D, E$.证明: 若 $D E=M P$, 则 $B C=2 B P$.
['作辅助线如下图. \n\n<img_4166>\n\n由于 $M$ 是中点, 对 $\\triangle P B C$ 有 $\\frac{B P}{C P}=\\frac{\\sin \\angle B P M}{\\sin \\angle C P M}$.\n\n而对 $\\triangle B D P$ 由正弦定理有 $\\frac{B P}{\\sin \\angle B D P}=\\frac{B D}{\\sin \\angle B P D}$.\n\n同理也有 $\\frac{C E}{\\sin \\angle C P E}=\\frac{C P}{\\sin \\angle C E P}$.\n\n故由 $\\angle B D P=\\angle B A P=\\angle C A P=\\angle C E P$ 有 $B D=B P \\cdot \\frac{\\sin \\angle B P D}{\\sin \\angle B D P}=C P$. $\\frac{\\sin \\angle C P M}{\\sin \\angle B P M} \\cdot \\frac{\\sin \\angle B P D}{\\sin \\angle B D P}=C P \\cdot \\frac{\\sin \\angle C P E}{\\sin \\angle C E P}=C E$\n\n于是由 $\\angle B D P=\\angle C E M$ 且有条件可知 $D E=M P$, 我们有 $\\triangle B D P \\cong \\triangle C E M$, 即 $B P=C M$. 故 $B C=2 B P$.']
null
false
null
null
null
18
Algebra
null
设整数 $a_{1}, a_{2}, \cdots, a_{2019}$ 满足 $1=a_{1} \leqslant a_{2} \leqslant \cdots \leqslant a_{2019}=99$. 记 $f=\left(a_{1}^{2}+a_{2}^{2}+\cdots+a_{2019}^{2}\right)-\left(a_{1} a_{3}+a_{2} a_{4}+a_{3} a_{5}+\cdots+a_{2017} a_{2019}\right)$. 求 $f$ 的最小值 $f_{0}$. 并确定使 $f=f_{0}$ 成立的数组 $\left(a_{1}, a_{2}, \cdots, a_{2019}\right)$ 的个数.
['$$\n\\begin{aligned}\nf & =\\frac{1}{2} \\sum_{i=1}^{2017}\\left(a_{i}-a_{i+2}\\right)^{2}+\\frac{a_{1}^{2}}{2}+\\frac{a_{2}^{2}}{2}+\\frac{a_{2018}^{2}}{2}+\\frac{a_{2019}^{2}}{2} \\\\\n& \\geqslant \\frac{1}{2} \\sum_{i=1}^{2015}\\left(a_{i}-a_{i+2}\\right)^{2}+\\frac{\\left(a_{2016}-a_{2017}\\right)^{2}}{2}+\\frac{\\left(a_{2017}-a_{2019}\\right)^{2}}{2}+\\frac{a_{2}^{2}}{2}+\\frac{a_{2017}^{2}}{2}+4901 \\\\\n& (因为a_{2017} \\leqslant a_{2018})\\\\\n& \\geqslant \\frac{1}{2} \\sum_{i=1}^{2015}\\left(a_{i+2}-a_{i}\\right)+\\frac{\\left(a_{2017}-a_{2016} \\right)}{2}+\\frac{\\left(a_{2017}-a_{2019}\\right)^{2}}{2}+\\frac{a_{2}^{2}}{2}+\\frac{a_{2017}^{2}}{2}+4901\\\\\n& (因为整数平方大于等于自己) \\\\\n&=a_{2017}-\\frac{a_{2}}{2}-\\frac{1}{2}+\\frac{\\left(a_{2017}-99\\right)^{2}}{2}+\\frac{a_{2}^{2}}{2}+\\frac{a_{2017}^{2}}{2}+4901\\\\\n&\\geqslant a_{2017}^{2}-98 a_{2017}+9801\\\\\n&\\geqslant 7400\\\\\n\\end{aligned}\n$$\n\n第一个不等号取等在于 $\\left(a_{2016}-a_{2018}\\right)^{2}+a_{2018}^{2}=\\left(a_{2016}-a_{2017}\\right)^{2}+a_{2017}^{2}$, 即我们有 $a_{2018}=a_{2017}$.\n\n第二个不等号取等在于 $a_{i}-a_{i+2}=0$ 或 $-1(1 \\leqslant i \\leqslant 2015)$ 和 $a_{2016}-a_{2017}=0$ 或 -1 .第三个不等号取等在于 $a_{2}^{2}=a_{2}$, 也是 $a_{2}=1$.\n\n第四个不等号取等条件在于 $a_{2017}=\\frac{98}{2}=49$.\n\n所以综上, $f \\geqslant 7400$ 的取等条件是 $a_{2}=1, a_{2017}=a_{2018}=49$, 而 $a_{2}$ 到 $a_{2017}$ 之间每两个相邻的数的差为 0 或 -1 .\n\n所以取等的组数等于 $a_{i}-a_{i-2}(4 \\leqslant i \\leqslant 2017)$ 和 $a_{2017}-a_{2016}$ 中的 1 的个数.\n\n由于这些数可以分两组:\n\n$2,4,6, \\ldots, 2016,2018$\n\n$1,3,5, \\ldots, 2015,2017$\n\n并由于 $a_{1}=a_{2}=1, a_{2017}=a_{2018}=49$, 所以显然对每一个 $i$ 而言, $a_{i-1}$ 和 $a_{i+1}$ 总有一个与 $a_{i}$ 相等.\n\n故我们设 $a_{i}=j(1 \\leqslant j \\leqslant 49)$ 的解数为 $b_{j}+1$.\n\n则 $b_{j} \\geqslant 1$, 且 $b_{1}+\\ldots+b_{49}=2018-49=1969$.\n\n所以 $\\left(b_{1}, \\ldots, b_{49}\\right)$ 的解数为 $C_{1968}^{48}$, 即 $\\left(a_{3}, \\ldots, a_{2016}\\right)$ 的取法有 $C_{1968}^{48}$ 种.']
['$\\binom{1968}{48}$']
false
null
Need_human_evaluate
null
19
Sequence
null
设 $m$ 为整数, $|m|>2$. 整数数列 $a_{1}, a_{2}, \ldots$ 满足: $a_{1}, a_{2}$ 不全为零, 且对任意正整数 $n$,均有 $a_{n+2}=a_{n+1}-m a_{n}$. 证明: 若存在整数 $r, s(r>s \geqslant 2)$ 使得 $a_{r}=a_{s}=a_{1}$, 则 $r-s \geqslant|m|$.
['首先我们不妨设 $\\left(a_{1}, a_{2}\\right)=1$, 不然整个数列每一项都除以一个 $\\left(a_{1}, a_{2}\\right)$ 可以得到同样的结果和结论.\n\n\n\n我们先证一个非常显然的引理.\n\n引理: 对任意 $a_{n}, n \\geqslant 3$, 我们都可以将 $a_{n}$ 写成 $a_{n}=k_{n} a_{2}+l_{n} a_{1}$, 其中 $m \\mid k_{n}-1$, $m \\mid l_{n}$.\n\n引理的证明: $a_{3}=a_{2}-m a_{1}, a_{4}=a_{3}-m a_{2}=(1-m) a_{2}-m a_{1}$. 所以 $n=3,4$ 时显然. 若 $n-2$ 和 $n-1$ 时都成立的话, 则\n\n$a_{n}=a_{n-1}-m a_{n-2}=\\left(k_{n-1}-m k_{n-2}\\right) a_{2}+\\left(l_{n-1}-m l_{n-2}\\right) a_{1}$ 所以还是显然.\n\n所以在模 $m$ 的时候我们总有 $a_{n} \\equiv a_{2}(\\bmod m)$, 故 $a_{r}=a_{s}=a_{1}$ 就表明了 $a_{2} \\equiv a_{1}$ $(\\bmod m)$, 由 $\\left(a_{1}, a_{2}\\right)=1$ 可得 $\\left(a_{1}, m\\right)=1$.\n\n那么 $\\frac{a_{r}-a_{s}}{m}=\\sum_{i=s+1}^{r} \\frac{a_{i}-a_{i-1}}{m}=\\sum_{i=s+1}^{r} a_{i-2} \\equiv(r-s) a_{1}(\\bmod m)$\n\n故 $m \\mid(r-s) a_{1}$, 所以 $m \\mid r-s$. 于是 $r-s \\geqslant|m|$.']
null
false
null
null
null
20
Combinatorics
null
设 $V$ 是空间中 2019 个点构成的集合, 其中任意四点不共面. 某些点之间连有线段, 记 $E$ 为这些线段构成的集合. 试求最小的正整数 $n$, 满足条件: 若 $E$ 至少有 $n$ 个元素, 则 $E$ 一定含有 908 个二元子集, 其中每个二元子集中的两条线段有公共端点, 且任意两个二元子集的交为空集.
['为了叙述方便,称一个图中的两条相邻的边构成一个 “角”.\n\n先证明一个引理: 设 $G=(V, E)$ 是一个简单图, 且 $G$ 是连通的, 则 $G$ 含有 $\\left[\\frac{|E|}{2}\\right]$ 个两两无公共边的角 (这里 $[\\alpha]$ 表示实数 $\\alpha$ 的整数部分).\n\n引理的证明: 对 $E$ 的元素个数 $|E|$ 归纳证明. 当 $|E|=0,1,2,3$ 时, 结论显然成立. 下面假设 $|E| \\geqslant 4$, 并且结论在 $|E|$ 较小时均成立.\n\n只需证明, 在 $G$ 中可以选取两条边 $a, b$ 构成一个角, 在 $G$ 中删去 $a, b$ 这两条边后, 剩下的图含有一个连通分支包含 $|E|-2$ 条边. 对这个连通分支应用归纳假设即得结论成立.\n\n考虑 $G$ 中的最长路 $P: v_{1} v_{2} \\ldots v_{k}$, 其中 $v_{1}, v_{2}, \\ldots, v_{k}$ 是互不相同的顶点. 因为 $G$ 连通, 故 $k \\geqslant 3$.\n\n情形 1: $\\operatorname{deg}\\left(v_{1}\\right) \\geqslant 2$. 由于 $P$ 是最长路, $v_{1}$ 的邻点均在 $v_{2}, \\ldots, v_{k}$ 中, 设 $v_{1} v_{i} \\in E$, 其中 $3 \\leqslant i \\leqslant k$. 则 $\\left\\{v_{1} v_{2}, v_{1} v_{i}\\right\\}$ 是一个角, 在 $E$ 中删去这两条边. 若 $v_{1}$ 处还有第三条边,则剩下的图是连通的: 若 $v_{1}$ 处仅有被删去的两条边, 则 $v_{1}$ 成为孤立点, 其余顶点仍互相连通. 总之在剩下的图中有一个连通分支含有 $|E|-2$ 条边.\n\n情形 $2: \\operatorname{deg}\\left(v_{1}\\right)=1, \\operatorname{deg}\\left(v_{2}\\right)=2$. 则 $\\left\\{v_{1} v_{2}, v_{2} v_{3}\\right\\}$ 是一个角, 在 $G$ 中删去这两条边后, $v_{1}, v_{2}$ 都成为孤立点, 其余的点互相连通, 因此有一个连通分支含有 $|E|-2$ 条边.\n\n情形 3: $\\operatorname{deg}\\left(v_{1}\\right)=1, \\operatorname{deg}\\left(v_{2}\\right) \\geqslant 3$, 且 $v_{2}$ 与 $v_{4}, \\ldots, v_{k}$ 中某个点相邻. 则 $\\left\\{v_{1} v_{2}, v_{2} v_{3}\\right\\}$是一个角, 在 $G$ 中删去这两条边后, $v_{1}$ 成为孤立点, 其余点互相连通, 因此有一个连通分支含有 $|E|-2$ 条边.\n\n情形 4: $\\operatorname{deg}\\left(v_{1}\\right)=1, \\operatorname{deg}\\left(v_{2}\\right) \\geqslant 3$, 且 $v_{2}$ 与某个 $u \\notin\\left\\{v_{1}, v_{3}, \\ldots, v_{k}\\right\\}$ 相邻. 由于 $P$ 是最长路, 故 $u$ 的邻点均在 $v_{2}, \\ldots, v_{k}$ 之中. 因 $\\left\\{v_{1} v_{2}, v_{2} u\\right\\}$ 是一个角, 在 $G$ 中删去这两条边, 则 $v_{1}$ 是孤立点, 若 $u$ 处仅有边 $u v_{2}$, 则删去所述边后 $u$ 也是孤立点, 而其余点互相连通. 若 $u$ 处还有其他边 $u v_{i}, 3 \\leqslant i \\leqslant k$, 则删去所述边后, 除 $v_{1}$ 外其余点互相连通, 总之, 剩下的图中有一个连通分支含有 $|E|-2$ 条边. 引理获证.\n\n回到原题, 题中的 $V$ 和 $E$ 可看作一个图 $G=(V, E)$.\n\n首先证明 $n \\geqslant 2795$.\n\n设 $V=\\left\\{v_{1}, v_{2}, \\ldots, v_{2019}\\right\\}$. 在 $v_{1}, v_{2}, \\ldots, v_{61}$ 中, 首先两两连边, 再删去其中 15 条边 (例如 $v_{1} v_{2}, v_{1} v_{3}, \\ldots, v_{1} v_{16}$ ), 共连了 $C_{61}^{2}-15=1815$ 条边, 则这 61 个点构成的图是连通图. 再将剩余的 $2019-61=1958$ 个点配成 979 对, 每对两点之间连一条边, 则图 $G$ 中一共连了 $1815+979=2794$ 条线段.\n\n由上述构造可见, $G$ 中的任何一个角必须使用 $v_{1}, v_{2}, \\ldots, v_{61}$ 相连的边, 因此至多有 $\\left[\\frac{1815}{2}\\right]=907$ 个两两无公共边的角. 故满足要求的 $n$ 不小于 2795 . 另一方面, 若 $|E| \\geqslant$ 2795 , 可任意删去若干条边, 只考虑 $|E|=2795$ 的情形. 设 $G$ 有 $k$ 个连通分支, 分别有 $m_{1}, \\ldots, m_{k}$ 个点, 及 $e_{1}, \\ldots, e_{k}$ 条边. 下面证明 $e_{1}, \\ldots, e_{k}$ 中至多有 979 个奇数.\n\n反证法, 假设 $e_{1}, \\ldots, e_{k}$ 中有至少 980 个奇数, 由于 $e_{1}+\\ldots+e_{k}=2795$ 是奇数, 故 $e_{1}, \\ldots, e_{k}$ 中至少有 981 个奇数, 故 $k \\geqslant 981$. 不妨设 $e_{1}, e_{2}, \\ldots, e_{981}$ 都是奇数, 显然 $m_{1}, m_{2}, \\ldots, m_{981} \\geqslant 2$. 令 $m=m_{981}+\\ldots+m_{k} \\geqslant 2$. 则有 $C_{m_{i}}^{2} \\geqslant e_{i}(1 \\leqslant i \\leqslant 980)$, $C_{m}^{2} \\geqslant e_{981}+\\ldots+e_{k}$, 故\n\n$$\n2795=\\sum_{i=1}^{k} e_{i} \\leqslant C_{m}^{2}+\\sum_{i=1}^{980} C_{m_{i}}^{2}\n$$\n\n利用组合数的凸性, 即对 $x \\geqslant y \\geqslant 3$, 有 $C_{x}^{2}+C_{y}^{2} \\leqslant C_{x+1}^{2}+C_{y-1}^{2}$, 可知当 $m_{1}, \\ldots, m_{980}, m$由 980 个 2 以及一个 59 构成时, $C_{m}^{2}+\\sum_{i=1}^{980} C_{m_{i}}^{2}$ 取得最大值. 于是\n\n$$\nC_{m_{i}}^{2}+\\sum_{i=1}^{980} C_{m_{i}}^{2} \\leqslant C_{59}^{2}+980 C_{2}^{2}=2691<2795\n$$\n\n这与前式矛盾.\n\n从而 $e_{1}, \\ldots, e_{k}$ 中至多有 979 个奇数. 对每个连通分支应用引理, 可知 $G$ 中含有 $N$ 个两两无公共边的角, 其中\n\n$$\nN=\\sum_{i=1}^{k}\\left[\\frac{e_{i}}{2}\\right] \\geqslant \\frac{1}{2}\\left(\\sum_{i=1}^{k} e_{i}-979\\right)=\\frac{1}{2}(2795-979)=908\n$$\n\n综上, 所求最小的 $n$ 是 2795 .']
['$2795$']
false
null
Numerical
null
21
Plane Geometry
null
在椭圆 $\Gamma$ 中, $F$ 为一个焦点, $A, B$ 为两个顶点. 若 $|F A|=3,|F B|=2$, 求 $|A B|$ 的所有可能值.
['不妨设平面直角坐标系中椭圆 $\\Gamma$ 的标准方程为 $\\frac{x^{2}}{a^{2}}+\\frac{y^{2}}{b^{2}}=1(a>b>0)$, 并记 $c=$ $\\sqrt{a^{2}-b^{2}}$. 由对称性, 可设 $F$ 为 $\\Gamma$ 的右焦点.\n\n易知 $F$ 到 $\\Gamma$ 的左顶点的距离为 $a+c$, 到右顶点的距离为 $a-c$, 到上, 下顶点的距离均为 $a$. 分以下情况讨论:\n\n(1) $A, B$ 分别为左右顶点, 此时 $a+c=3, a-c=2$, 故 $|A B|=2 a=5$ (相应地, $b^{2}=(a+c)(a-c)=6, \\Gamma$ 的方程为 $\\left.\\frac{4 x^{2}}{25}+\\frac{y^{2}}{6}=1\\right)$\n\n(2) $A$ 为左顶点, $B$ 为上顶点或下顶点, 此时 $a+c=3, a=2$, 故 $c=1$, 进而 $b^{2}=a^{2}-c^{2}=3$, 所以 $|A B|=\\sqrt{a^{2}+b^{2}}=\\sqrt{7}$ (相应的 $\\Gamma$ 的方程为 $\\frac{x^{2}}{4}+\\frac{y^{2}}{3}=1$ )\n\n(3) $A$ 为上顶点或下顶点, $B$ 为右顶点, 此时 $a=3, a-c=2$ 故 $c=1$, 进而 $b^{2}=$ $a^{2}-c^{2}=8$, 所以 $|A B|=\\sqrt{a^{2}+b^{2}}=\\sqrt{17}$ (相应的 $\\Gamma$ 的方程为 $\\frac{x^{2}}{9}+\\frac{y^{2}}{8}=1$ )']
['$5, \\sqrt{7}, \\sqrt{17}$']
true
null
Numerical
null
22
Algebra
null
设 $a, b, c$ 均大于 1 , 满足 $\left\{\begin{array}{l}\log a+\log _{b} c=3 \\ \log b+\log _{a} c=4\end{array}\right.$ 求 $\log a \cdot \log c$ 的最大值.
['设 $\\log _{a}=x, \\log _{b}=y, \\log _{c}=z$, 由 $a, b, c>1$ 可知 $x, y, z>0$.\n\n由条件及换底公式知 $x+\\frac{z}{y}=3, y+\\frac{z}{x}=4$, 即 $x y+z=3 y=4 x$.\n\n由此, 令 $x=3 t, y=4 t(t>0)$, 则 $z=4 x-x y=12 t-12 t^{2}$. 其中由 $z>0$ 可知 $t \\in(0,1)$.\n\n因此, 结合三元平均不等式得\n\n$\\log _{a} \\log _{c}=x z=3 t \\cdot 12 t(1-t)=18 \\cdot t^{2}(2-2 t) \\leqslant 18 \\cdot\\left(\\frac{t+t+(2-2 t)}{3}\\right)^{3}=18 \\cdot\\left(\\frac{2}{3}\\right)^{3}=$ $\\frac{16}{3}$\n\n当 $t=2-2 t$, 即 $t=\\frac{2}{3}$ (相应的 $a, b, c$ 分别为 $\\left.100,10^{\\frac{8}{3}}, 10^{\\frac{8}{3}}\\right)$ 时, $\\log _{a} \\log _{c}$ 取到最大值 $\\frac{16}{3}$.']
['$\\frac{16}{3}$']
false
null
Numerical
null
23
Sequence
null
设复数数列 $\left\{z_{n}\right\}$ 满足: $\left|z_{1}\right|=1$, 且对任意正整数 $n$, 均有 $4 z_{n+1}^{2}+2 z_{n} z_{n+1}+z_{n}^{2}=0$.证明; 对任意正整数 $m$, 均有 $\left|z_{1}+z_{2}+\cdots+z_{m}\right|<\frac{2 \sqrt{3}}{3}$.
['归纳地可知 $z_{n} \\neq 0\\left(n \\in \\mathbb{N}^{*}\\right)$, 由条件得 $4\\left(\\frac{z_{n+1}}{z_{n}}\\right)^{2}+2\\left(\\frac{z_{n+1}}{z_{n}}\\right)+1=0\\left(n \\in \\mathbb{N}^{*}\\right)$\n\n解得 $\\frac{z_{n+1}}{z_{n}}=\\frac{-1 \\pm \\sqrt{3} i}{4}\\left(n \\in \\mathbb{N}^{*}\\right)$\n\n\n\n因此 $\\frac{\\left|z_{n+1}\\right|}{\\left|z_{n}\\right|}=\\left|\\frac{z_{n+1}}{z_{n}}\\right|=\\left|\\frac{-1 \\pm \\sqrt{3} i}{4}\\right|=\\frac{1}{2}$, 故 $\\left|z_{n}\\right|=\\left|z_{1}\\right| \\cdot \\frac{1}{2^{n-1}}=\\frac{1}{2^{n-1}}\\left(n \\in \\mathbb{N}^{*}\\right)$ (1)进而有 $\\left|z_{n}+z_{n+1}\\right|=\\left|z_{n}\\right| \\cdot\\left|1+\\frac{z_{n+1}}{z_{n}}\\right|=\\frac{1}{2^{n-1}} \\cdot\\left|\\frac{3 \\pm \\sqrt{3} i}{4}\\right|=\\frac{\\sqrt{3}}{2^{n}}\\left(n \\in \\mathbb{N}^{*}\\right)$ (2)当 $m$ 为偶数时, 设 $m=2 s\\left(s \\in \\mathbb{N}^{*}\\right)$. 利用 (2) 可得 $\\left|z_{1}+z_{2}+\\cdots+z_{m}\\right| \\leqslant \\sum_{k=1}^{s}\\left|z_{2 k-1}+z_{2 k}\\right|<\\sum_{k=1}^{\\infty}\\left|z_{2 k-1}+z_{2 k}\\right|=\\sum_{k=1}^{\\infty} \\frac{\\sqrt{3}}{2^{2 k-1}}=\\frac{2 \\sqrt{3}}{3}$.当 $m$ 为奇数时, 设 $m=2 s+1(s \\in \\mathbb{N})$. 由 (1), (2) 可知 $\\left|z_{2 s+1}\\right|=\\frac{1}{2^{s}}<\\frac{\\sqrt{3}}{3 \\cdot 2^{2 s-1}}=\\sum_{k=s+1}^{\\infty} \\frac{\\sqrt{3}}{2^{2 s-1}}=\\sum_{k=s+1}^{\\infty}\\left|z_{2 k-1}+z_{2 k}\\right|$.\n故 $\\left|z_{1}+z_{2}+\\cdots+z_{m}\\right| \\leqslant\\left(\\sum_{k=1}^{s}\\left|z_{2 k-1}+z_{2 k}\\right|\\right)+\\left|z_{2 k+1}\\right|<\\sum_{k=1}^{\\infty}\\left|z_{2 k-1}+z_{2 k}\\right|=\\frac{2 \\sqrt{3}}{3}$.综上, 结论获证.']
null
false
null
null
null
24
Algebra
null
设正实数 $a_{1}, a_{2}, \cdots, a_{100}$ 满足 $a_{i} \geqslant a_{101-i}(i=1,2, \cdots, 50)$. 记 $x_{k}=\frac{k a_{k+1}}{a_{1}+a_{2}+\cdots+a_{k}}(k=$ $1,2, \cdots, 99)$. 证明: $x_{1} x_{2}^{2} \cdots x_{99}^{99} \leqslant 1$.
['注意到 $a_{1}, a_{2}, \\cdots, a_{100}>0$. 对 $k=1,2, \\cdots, 99$, 由平均值不等式知 $0<\\left(\\frac{k}{a_{1}+a_{2}+\\cdots+a_{k}}\\right)^{k} \\leqslant$ $\\frac{1}{a_{1} a_{2} \\cdots a_{k}}$\n\n从而有 $x_{1} x_{2}^{2} \\cdots x_{99}^{99}=\\prod_{k=1}^{99} a_{k+1}^{k}\\left(\\frac{k}{a_{1}+a_{2}+\\cdots+a_{k}}\\right) \\leqslant \\prod_{k=1}^{99} \\frac{a_{k+1}^{k}}{a_{1} a_{2} \\cdots a_{k}}$ (1)\n\n记 (1) 的右端为 $T$, 则对任意 $i=1,2, \\cdots, 100, a_{i}$ 在 $T$ 的分子中的次数为 $i-1$, 在 $T$的分母中的次数为 $100-i$. 从而\n\n$T=\\prod_{i=1}^{100} a_{i}^{2 i-101}=\\prod_{i=1}^{50} a_{i}^{2 i-101} a_{101-i}^{2(101-i)-101}=\\prod_{i=1}^{50}\\left(\\frac{a_{101-i}}{a_{i}}\\right)^{101-2 i}$\n\n又 $0 \\leqslant a_{101-i} \\leqslant a_{i}(i=1,2, \\cdots, 50)$, 故 $T \\leqslant 1$, 结合 (1) 得 $x_{1} x_{2}^{2} \\cdots x_{99}^{99} \\leqslant T \\leqslant 1$.']
null
false
null
null
null
25
Number Theory
null
求满足以下条件的所有正整数 $n$ : $n$ 至少有 4 个正约数; 若 $d_{1}<d_{2}<\cdots<d_{k}$ 是 $n$ 的所有正约数, 则 $d_{2}-d_{1}, d_{3}-d_{2}, \cdots, d_{k}-d_{k-1}$ 构成等比数列.
['由条件知 $k \\geqslant 4$, 且 $\\frac{d_{3}-d_{2}}{d_{2}-d_{1}}=\\frac{d_{k}-d_{k-1}}{d_{k-1}-d_{k-2}}$.\n\n易知 $d_{1}=1, d_{k}=n, d_{k-1}=\\frac{n}{d_{2}}, d_{k-2}=\\frac{n}{d_{3}}$, 代入上式得\n\n$\\frac{d_{3}-d_{2}}{d_{2}-1}=\\frac{n-\\frac{n}{d_{2}}}{\\frac{n}{d_{2}}-\\frac{n}{d_{3}}}$, 化简得 $\\left(d_{3}-d_{2}\\right)^{2}=\\left(d_{2}-1\\right)^{2} d_{3}$.\n\n由此可知 $d_{3}$ 是完全平方数. 由于 $d_{2}=p$ 是 $n$ 的最小素因子, $d_{3}$ 是平方数, 故只能 $d_{3}=p^{2}$.\n\n从而序列 $d_{2}-d_{1}, d_{3}-d_{2}, \\cdots, d_{k}-d_{k-1}$ 为 $p-1, p^{2}-p, p^{3}-p^{2}, \\cdots, p^{k-1}-p^{k-2}$, 即 $d_{1}, d_{2}, d_{3}, \\cdots, d_{k}$ 为 $1, p, p^{2}, \\cdots, p^{k-1}$, 而此时相应的 $n$ 为 $p^{k-1}$.\n\n综上可知, 满足条件的 $n$ 为所有形如 $p^{a}$ 的数, 其中 $p$ 是素数, 整数 $a \\geqslant 3$.']
['所有形如 $p^{a}$ 的数, 其中 $p$ 是素数, 整数 $a \\geqslant 3$']
true
null
Need_human_evaluate
null
26
Plane Geometry
null
如图, 点 $A, B, C, D, E$ 在一条直线上顺次排列, 满足 $B C=C D=\sqrt{A B \cdot D E}$, 点 $P$在该直线外, 满足 $P B=P D$. 点 $K, L$ 分别在线段 $P B, P D$ 上, 满足 $K C$ 平分 $\angle B K E$, $L C$ 平分 $\angle A L D$. 证明: $A, K, L, E$ 四点共圆. <img_4097>
['令 $A B=1, B C=C D=t(>0)$, 由条件知 $D E=t^{2}$.\n\n注意到 $\\angle B K E<\\angle A B K=\\angle P D E<180^{\\circ}-\\angle D E K$, 可在 $C B$ 延长线上取一点 $A^{\\prime}$,使得 $\\angle A^{\\prime} K E=\\angle A B K=\\angle A^{\\prime} B K$, 并连接 $LE$, 如下图.\n\n<img_4165>\n\n此时有 $\\triangle A^{\\prime} B K \\sim \\triangle A^{\\prime} K E$, 故 $\\frac{A^{\\prime} B}{A^{\\prime} K}=\\frac{A^{\\prime} K}{A^{\\prime} E}=\\frac{B K}{K E}$.\n\n又 $K C$ 平分 $\\angle B K E$, 故 $\\frac{B K}{K E}=\\frac{B C}{C E}=\\frac{1}{t+t^{2}}=\\frac{1}{1+t}$. 于是有\n\n$\\frac{A^{\\prime} B}{A^{\\prime} E}=\\frac{A^{\\prime} B}{A^{\\prime} K} \\cdot \\frac{A^{\\prime} K}{A^{\\prime} E}=\\left(\\frac{B K}{K E}\\right)^{2}=\\frac{1}{1+2 t+t^{2}}=\\frac{A B}{A E}$.\n\n由上式两端减 1 , 得 $\\frac{B E}{A^{\\prime} E}=\\frac{B E}{A E}$, 从而 $A^{\\prime}=A$. 因此 $\\angle A K E=\\angle A^{\\prime} K E=\\angle A B K$.\n\n同理可得 $\\angle A L E=\\angle E D L$.\n\n而 $\\angle A B K=\\angle E D L$, 所以 $\\angle A K E=\\angle A L E$. 因此 $A, K, L, E$ 四点共圆.']
null
false
null
null
null
26
Plane Geometry
null
如图, 点 $A, B, C, D, E$ 在一条直线上顺次排列, 满足 $B C=C D=\sqrt{A B \cdot D E}$, 点 $P$在该直线外, 满足 $P B=P D$. 点 $K, L$ 分别在线段 $P B, P D$ 上, 满足 $K C$ 平分 $\angle B K E$, $L C$ 平分 $\angle A L D$. 证明: $A, K, L, E$ 四点共圆. ![](https://cdn.mathpix.com/cropped/2023_12_20_c60b04d3565991d80f64g-1.jpg?height=526&width=919&top_left_y=1870&top_left_x=614)
['令 $A B=1, B C=C D=t(>0)$, 由条件知 $D E=t^{2}$.\n\n注意到 $\\angle B K E<\\angle A B K=\\angle P D E<180^{\\circ}-\\angle D E K$, 可在 $C B$ 延长线上取一点 $A^{\\prime}$,使得 $\\angle A^{\\prime} K E=\\angle A B K=\\angle A^{\\prime} B K$, 并连接 $LE$, 如下图.\n\n![](https://cdn.mathpix.com/cropped/2023_12_20_15eee5c47c745235be2ag-1.jpg?height=528&width=923&top_left_y=1341&top_left_x=612)\n\n此时有 $\\triangle A^{\\prime} B K \\sim \\triangle A^{\\prime} K E$, 故 $\\frac{A^{\\prime} B}{A^{\\prime} K}=\\frac{A^{\\prime} K}{A^{\\prime} E}=\\frac{B K}{K E}$.\n\n又 $K C$ 平分 $\\angle B K E$, 故 $\\frac{B K}{K E}=\\frac{B C}{C E}=\\frac{1}{t+t^{2}}=\\frac{1}{1+t}$. 于是有\n\n$\\frac{A^{\\prime} B}{A^{\\prime} E}=\\frac{A^{\\prime} B}{A^{\\prime} K} \\cdot \\frac{A^{\\prime} K}{A^{\\prime} E}=\\left(\\frac{B K}{K E}\\right)^{2}=\\frac{1}{1+2 t+t^{2}}=\\frac{A B}{A E}$.\n\n由上式两端减 1 , 得 $\\frac{B E}{A^{\\prime} E}=\\frac{B E}{A E}$, 从而 $A^{\\prime}=A$. 因此 $\\angle A K E=\\angle A^{\\prime} K E=\\angle A B K$.\n\n同理可得 $\\angle A L E=\\angle E D L$.\n\n而 $\\angle A B K=\\angle E D L$, 所以 $\\angle A K E=\\angle A L E$. 因此 $A, K, L, E$ 四点共圆.']
['证明题']
false
null
Need_human_evaluate
null
27
Combinatorics
null
将一个凸 2019 边形的每条边任意染为红, 黄, 蓝三种颜色之一, 每种颜色的边各 673 条. 证明: 可作这个凸 2019 边形的 2016 条在内部互不相交的对角线将其剖分成 2017 个三角形, 并将所作的每条对角线也染为红, 黄, 蓝三种颜色之一, 使得每个三角形的三条边或者颜色全部相同, 或者颜色互不相同.
['我们对 $n \\geqslant 5$ 归纳证明加强的命题: 如果将凸 $n$ 边形的边染为三种颜色 $a, b, c$, 并且三种颜色的边均至少有一条, 那么可作满足要求的三角形剖分.\n\n当 $n=5$ 时, 若三种颜色的边数为 $1,1,3$, 由对称性, 只需考虑如下图两种情形, 分别可作图中所示的三角形剖分.\n<img_4234>\n\n若三种颜色的边数为 $1,2,2$, 由对称性, 只需考虑如下图三种情形, 分别可作图中所示的三角形剖分.\n\n<img_4102>\n\n\n\n<img_4065>\n\n假设结论对 $n(n \\geqslant 5)$ 成立, 考虑 $n+1$ 的情形, 将凸 $n+1$ 边形记为 $A_{1} A_{2} \\cdots A_{n+1}$.情形一: 有两种颜色的边各只有一条. 不妨设 $a, b$ 色边只有一条. 由于 $n+1 \\geqslant 6$, 故存在连续两条边均为 $c$ 色, 不妨设是 $A_{n} A_{n+1}, A_{n+1} A_{1}$. 作对角线 $A_{1} A_{n}$, 并将 $A_{1} A_{n}$染为 $c$ 色, 则三角形 $A_{n} A_{n+1} A_{1}$ 的三边全部同色. 此时凸 $n$ 边形 $A_{1} A_{2} \\cdots A_{n}$ 的三种颜色的边均至少有一条, 由归纳假设, 可对其作符合要求的三角形剖分.\n\n情形二: 某种颜色的边只有一条, 其余颜色的边均至少两条. 不妨设 $a$ 色边只有一条,于是可以选择两条相邻边均不是 $a$ 色, 不妨设 $A_{n} A_{n+1}, A_{n+1} A_{1}$ 均不是 $a$ 色, 作对角线 $A_{1} A_{n}$, 则 $A_{1} A_{n}$ 有唯一的染色方式, 使得三角形 $A_{n} A_{n+1} A_{1}$ 的三边全部同色或互不同色. 此时凸 $n$ 边形 $A_{1} A_{2} \\cdots A_{n}$ 的三种颜色的边均至少有一条, 由归纳假设, 可对其作符合要求的三角形剖分.\n\n情形三: 每种颜色的边均至少两条. 作对角线 $A_{1} A_{n}$, 则 $A_{1} A_{n}$ 有唯一的染色方式, 使得三角形 $A_{n} A_{n+1} A_{1}$ 的三边全部同色或互不同色. 此时凸 $n$ 边形 $A_{1} A_{2} \\cdots A_{n}$ 的三种颜色的边均至少有一条, 由归纳假设, 可对其作符合要求的三角形剖分.\n\n综合以上 3 种情形, 可知 $n+1$ 的情形下结论也成立.\n\n由数学归纳法, 结论获证.']
null
false
null
null
null
28
Elementary Functions
null
已知定义在 $\mathrm{R}^{*}$ 上的函数 $f(x)=\left\{\begin{array}{l}\left|\log _{3} x-1\right|, 0<x \leqslant 9 \\ 4-\sqrt{x}, x>9\end{array}\right.$ 设 $a, b, c$ 是三个互不相同的实数, 满足 $f(a)=f(b)=f(c)$, 求 $a b c$ 的取值范围.
['不妨假设 $a<b<c$, 由于 $f(x)$ 在 $(0,3]$ 上严格递减, 在 $[3,9]$ 上严格递增, 在 $[9,+\\infty)$上严格递减, 且 $f(3)=0, f(9)=1$, 故结合图像可知 $a \\in(0,3), b \\in(3,9), c \\in(9,+\\infty)$,并且 $f(a)=f(b)=f(c) \\in(0,1)$. 由 $f(a)=f(b)$ 得 $1-\\log _{3} a=\\log _{3} b-1$, 取 $\\log _{3} a+\\log _{3} b=2$, 因此 $a b=3^{2}=9$. 于是 $a b c=9 c$. 又 $0<f(c)=4-\\sqrt{c}<1$, 故 $c \\in(9,16)$. 进而 $a b c=9 c \\in(81,144)$.']
['$(81, 144)$']
false
null
Interval
null
29
Sequence
null
已知实数列 $a_{1}, a_{2}, a_{3}, \cdots$ 满足: 对任意正整数 $n$, 有 $a_{n}\left(2 S_{n}-a_{n}\right)=1$, 其中 $S_{n}$ 表示数列的前 $n$ 项和. 证明: 对任意正整数 $n$, 有 $a_{n}<2 \sqrt{n}$;
['根据题意, 有\n\n$$\na_{1}^{2}=1\n$$\n\n命题成立. 当 $n \\geqslant 2$ 时, 有\n\n$$\n\\left(S_{n}-S_{n-1}\\right)\\left(S_{n}+S_{n-1}\\right)=1\n$$\n\n于是\n\n$$\nS_{n}^{2}=S_{n-1}^{2}+1,\n$$\n\n从而\n\n$$\nS_{n}^{2}=n, n=1,2, \\cdots \\text {. }\n$$\n\n\n\n因此当 $n \\geqslant 2$ 时, 有\n\n$$\na_{n}=S_{n}-S_{n-1} \\leqslant \\sqrt{n}+\\sqrt{n-1}<2 \\sqrt{n},\n$$\n\n命题成立.']
null
false
null
null
null
30
Sequence
null
已知实数列 $a_{1}, a_{2}, a_{3}, \cdots$ 满足: 对任意正整数 $n$, 有 $a_{n}\left(2 S_{n}-a_{n}\right)=1$, 其中 $S_{n}$ 表示数列的前 $n$ 项和. 证明: 对任意正整数 $n$, 有 $a_{n} a_{n+1}<1$.
['只需要考虑 $a_{n}$ 与 $a_{n+1}$ 同号的情形, 不妨设 $a_{n}, a_{n+1}$ 均为正数, 则\n\n$$\nS_{n+1}>S_{n}>S_{n-1}>-\\sqrt{n},\n$$\n\n于是\n\n$$\nS_{n}=\\sqrt{n}, S_{n+1}=\\sqrt{n+1},\n$$\n\n于是\n\n$$\na_{n}=\\sqrt{n} \\pm \\sqrt{n-1}, a_{n+1}=\\sqrt{n+1}-\\sqrt{n},\n$$\n\n从而\n\n$$\n\\begin{aligned}\na_{n} a_{n+1} & \\leqslant(\\sqrt{n}+\\sqrt{n-1}) \\cdot(\\sqrt{n+1}-\\sqrt{n}) \\\\\n& <(\\sqrt{n+1}+\\sqrt{n}) \\cdot(\\sqrt{n+1}-\\sqrt{n}) \\\\\n& =1,\n\\end{aligned}\n$$\n\n命题得证.']
null
false
null
null
null
31
Plane Geometry
null
在平面直角坐标系 $x O y$ 中, 设 $A B$ 是抛物线 $y^{2}=4 x$ 的过点 $F(1,0)$ 的弦, $\triangle A O B$ 的外接圆交抛物线于点 $P$ (不同于点 $O, A, B$ ). 若 $P F$ 平分 $\angle A P B$, 求 $|P F|$ 的所有可能值.
['设 $A\\left(\\frac{y_{1}^{2}}{4}, y_{1}\\right), B\\left(\\frac{y_{2}^{2}}{4}, y_{2}\\right), P\\left(\\frac{y_{3}^{2}}{4}, y_{3}\\right)$, 由条件知 $y_{1}, y_{2}, y_{3}$ 两两不等且非零. 设直线 $A B$ 的方程为 $x=t y+1$, 与抛物线方程联立可得 $y^{2}-4 t y-4=0$, 故 $y_{1} y_{2}=-4$. (1)\n\n注意到 $\\triangle A O B$ 的外接圆过点 $O$, 可设该圆的方程为 $x^{2}+y^{2}+d x+e y=0$, 与 $x=\\frac{y^{2}}{4}$联立得, $\\frac{y^{4}}{16}+\\left(1+\\frac{d}{4}\\right) y^{2}+e y=0$. 该四次方程有 $y=y_{1} y_{2}, y_{3}, 0$ 这四个不同的实根, 故由韦达定理得 $y_{1}+y_{2}+y_{3}+0=0$, 从而 $y_{3}=-\\left(y_{1}+y_{2}\\right)$. (2)\n\n因 $P F$ 平分 $\\angle A P B$, 由角平分线定理知, $\\frac{|P A|}{|P B|}=\\frac{|F A|}{|F B|}=\\frac{\\left|y_{1}\\right|}{\\left|y_{2}\\right|}$, 结合 (1), (2), 有 $\\frac{y_{1}^{2}}{y_{2}^{2}}=\\frac{|P A|^{2}}{|P B|^{2}}=\\frac{\\left(\\frac{y_{3}^{2}}{4}-\\frac{y_{1}^{2}}{4}\\right)^{2}+\\left(y_{3}-y_{1}\\right)^{2}}{\\left(\\frac{y_{3}^{2}}{4}-\\frac{y_{2}^{2}}{4}\\right)^{2}+\\left(y_{3}-y_{2}\\right)^{2}}=\\frac{\\left(\\left(y_{1}+y_{2}\\right)^{2}-y_{1}^{2}\\right)^{2}+16\\left(2 y_{1}+y_{2}\\right)^{2}}{\\left(\\left(y_{1}+y_{2}\\right)^{2}-y_{2}^{2}\\right)^{2}+16\\left(2 y_{2}+y_{1}\\right)^{2}}=$ $\\frac{\\left(y_{2}^{2}-8\\right)^{2}+16\\left(4 y_{1}^{2}+y_{2}^{2}-16\\right)}{\\left(y_{1}^{2}-8\\right)^{2}+16\\left(4 y_{2}^{2}+y_{1}^{2}-16\\right)}=\\frac{y_{2}^{4}+64 y_{1}^{2}-192}{y_{1}^{4}+64 y_{2}^{2}-192}$\n\n即 $y_{1}^{6}+64 y_{1}^{2} y_{2}^{2}-192 y_{1}^{2}=y_{2}^{6}+64 y_{2}^{2} y_{1}^{2}-192 y_{2}^{2}$, 故 $\\left(y_{1}^{2}-y_{2}^{2}\\right)\\left(y_{1}^{4}+y_{1}^{2} y_{2}^{2}+y_{2}^{4}-192\\right)=0$.\n\n\n\n当 $y_{1}^{2}=y_{2}^{2}$ 时, $y_{2}=-y_{1}$, 故 $y_{3}=0$, 此时 $P$ 与 $O$ 重合, 与条件不符.\n\n当 $y_{1}^{4}+y_{1}^{2} y_{2}^{2}+y_{2}^{4}-192=0$ 时,注意到 (1), 有 $\\left(y_{1}^{2}+y_{2}^{2}\\right)^{2}=192+\\left(y_{1} y_{2}\\right)^{2}=208$. 因 $y_{1}^{2}+y_{2}^{2}=4 \\sqrt{13}>8=\\left|2 y_{1} y_{2}\\right|$, 故满足 (1) 以及 $y_{1}^{2}+y_{2}^{2}=4 \\sqrt{13}$ 的实数 $y_{1}$, $y_{2}$ 存在, 对应可得满足条件的点 $A, B$. 此时, 结合 (1), (2) 知 $|P F|=\\frac{y_{3}^{2}}{4}+1=\\frac{\\left(y_{1}+y_{2}\\right)^{2}+4}{4}=$ $\\frac{y_{1}^{2}+y_{2}^{2}-4}{4}=\\frac{\\sqrt{208}-4}{4}=\\sqrt{13}-1$.']
['$\\sqrt{13}-1$']
false
null
Numerical
null
32
Algebra
null
设 $n$ 是正整数, $a_{1}, a_{2}, \cdots, a_{n}, b_{1}, b_{2}, \cdots, b_{n}, A, B$ 均为正实数, 满足 $a_{i} \leqslant b_{i}, a_{i} \leqslant A, i=$ $1,2, \cdots, n$, 且 $\frac{b_{1} b_{2} \cdots b_{n}}{a_{1} a_{2} \cdots a_{n}} \leqslant \frac{B}{A}$. 证明: $\frac{\left(b_{1}+1\right)\left(b_{2}+1\right) \cdots\left(b_{n}+1\right)}{\left(a_{1}+1\right)\left(a_{2}+1\right) \cdots\left(a_{n}+1\right)} \leqslant \frac{B+1}{A+1}$.
['由条件知, $k_{i}=\\frac{b_{i}}{a_{i}} \\geqslant 1, i=1,2, \\cdots, n$. 记 $\\frac{B}{A}=K$, 则 $\\frac{b_{1} b_{2} \\cdots b_{n}}{a_{1} a_{2} \\cdots a_{n}} \\leqslant \\frac{B}{A}$ 化为 $k_{1} k_{2} \\cdots k_{n} \\leqslant K$. 要证明\n\n$\\prod_{i=1}^{n} \\frac{k_{i} a_{i}+1}{a_{i}+1} \\leqslant \\frac{K A+1}{A+1}$. (1)\n\n对 $i=1,2, \\cdots, n$, 由于 $k_{i} \\geqslant 1$ 及 $0<a_{i} \\leqslant A$ 知, $\\frac{k_{i} a_{i}+1}{a_{i}+1}=k_{i}-\\frac{k_{i}-1}{a_{i}+1} \\leqslant k_{i}-\\frac{k_{i}-1}{A+1}=$ $\\frac{k_{i} A+1}{A+1}$.\n\n结合 $K \\geqslant k_{1} k_{2} \\cdots k_{n}$ 知, 为证明 (1), 仅需证明当 $A>0, k_{i} \\geqslant 1(i=1,2, \\cdots, n)$ 时, 有 $\\prod_{i=1}^{n} \\frac{k_{i} A+1}{A+1} \\leqslant \\frac{k_{1} k_{2} \\cdots k_{n} A+1}{A+1}$. (2)\n\n对 $n$ 进行归纳, 当 $n=1$ 时, 结论显然成立.\n\n当 $n=2$ 时, 由 $A>0, k_{1}, k_{2} \\geqslant 1$ 可知 $\\frac{k_{1} A+1}{A+1} \\cdot \\frac{k_{2} A+1}{A+1} - \\frac{k_{1} k_{2} A+1}{A+1}=-\\frac{A\\left(k_{1}-1\\right)\\left(k_{2}-1\\right)}{(A+1)^{2}} \\leqslant 0$ . (3)\n\n因此 $n=2$ 时结论成立.\n\n设 $n=m$ 时结论成立, 则当 $n=m+1$ 时, 利用归纳假设知, $\\prod_{i=1}^{m+1} \\frac{k_{1} A+1}{A+1}=\\left(\\prod_{i=1}^{m} \\frac{k_{i} A+1}{A+1}\\right)$. $\\frac{k_{m+1} A+1}{A+1} \\leqslant \\frac{k_{1} k_{2} \\cdots k_{m} A+1}{A+1} \\cdot \\frac{k_{m+1} A+1}{A+1} \\leqslant \\frac{k_{1} k_{2} \\cdots k_{m+1} A+1}{A+1}$,最后一步是在 (3) 中用 $k_{1} k_{2} \\cdots k_{m}, k_{m+1}$ (注意 $k_{1} k_{2} \\cdots k_{m} \\geqslant 1, k_{m+1} \\geqslant 1$ ) 分别代替 $k_{1}, k_{2}$. 从而 $n=m+1$ 时结论成立.\n\n由数学归纳法可知, (2) 对所有正整数 $n$ 成立, 故命题得证.']
null
false
null
null
null
33
Plane Geometry
null
如图, $\triangle A B C$ 为锐角三角形, $A B<A C, M$ 为 $B C$ 边的中点, 点 $D$ 和 $E$ 分别为 $\triangle A B C$ 的外接圆弧 $B A C$ 和弧 $B C$ 的中点, $F$ 为 $\triangle A B C$ 的内切圆在 $A B$ 边上的切点, $G$ 为 $A E$ 与 $B C$ 的交点, $N$ 在线段 $E F$ 上, 满足 $N B \perp A B$. 证明: 若 $B N=E M$, 则 $D F \perp F G$. <img_4148>
['作辅助线如下图.\n\n<img_4139>\n\n\n由条件知, $D E$ 为 $\\triangle A B C$ 外接圆的直径, $D E \\perp B C$ 于 $M, A E \\perp A D$. 记 $I$ 为 $\\triangle A B C$的内心, 则 $I$ 在 $A E$ 上, $I F \\perp A B$. 由 $N B \\perp A B$ 可知 $\\angle N B E=\\angle A B E-\\angle A B N=$ $\\left(180^{\\circ}-\\angle A D E\\right)-90^{\\circ}=90^{\\circ}-\\angle A D E=\\angle M E I$. (1)\n\n又根据内心的性质, 有 $\\angle E B I=\\angle E B C+\\angle C B I=\\angle E A C+\\angle A B I=\\angle E A B+$ $\\angle A B I=\\angle E I B$, 从而 $B E=E I$. 结合 $B N=E M$ 及 (1) 知, $\\triangle N B E \\cong \\triangle M E I$. 于是 $\\angle E M I=\\angle B N E=90^{\\circ}+\\angle B F E=180^{\\circ}-\\angle E F I$, 故 $E, F, I, M$ 四点共圆. 进而可知 $\\angle A F M=90^{\\circ}+\\angle I F M=90^{\\circ}+\\angle I E M=\\angle A G M$, 从而 $A, F, G, M$ 四点共圆.再由 $\\angle D A G=\\angle D M G=90^{\\circ}$ 知, $A, G, M, D$ 四点共圆, 所以 $A, F, G, M, D$ 五点共圆.从而 $\\angle D F G=\\angle D A G=90^{\\circ}$ ,即 $D F \\perp F G$.']
null
false
null
null
null
33
Plane Geometry
null
如图, $\triangle A B C$ 为锐角三角形, $A B<A C, M$ 为 $B C$ 边的中点, 点 $D$ 和 $E$ 分别为 $\triangle A B C$ 的外接圆弧 $B A C$ 和弧 $B C$ 的中点, $F$ 为 $\triangle A B C$ 的内切圆在 $A B$ 边上的切点, $G$ 为 $A E$ 与 $B C$ 的交点, $N$ 在线段 $E F$ 上, 满足 $N B \perp A B$. 证明: 若 $B N=E M$, 则 $D F \perp F G$. ![](https://cdn.mathpix.com/cropped/2023_12_20_10d1b65ab08391a1650ag-1.jpg?height=623&width=637&top_left_y=1416&top_left_x=752)
['作辅助线如下图.\n\n![](https://cdn.mathpix.com/cropped/2023_12_20_a64f1e2de13a97afda0bg-1.jpg?height=152&width=140&top_left_y=1289&top_left_x=1004)\n\n\n由条件知, $D E$ 为 $\\triangle A B C$ 外接圆的直径, $D E \\perp B C$ 于 $M, A E \\perp A D$. 记 $I$ 为 $\\triangle A B C$的内心, 则 $I$ 在 $A E$ 上, $I F \\perp A B$. 由 $N B \\perp A B$ 可知 $\\angle N B E=\\angle A B E-\\angle A B N=$ $\\left(180^{\\circ}-\\angle A D E\\right)-90^{\\circ}=90^{\\circ}-\\angle A D E=\\angle M E I$. (1)\n\n又根据内心的性质, 有 $\\angle E B I=\\angle E B C+\\angle C B I=\\angle E A C+\\angle A B I=\\angle E A B+$ $\\angle A B I=\\angle E I B$, 从而 $B E=E I$. 结合 $B N=E M$ 及 (1) 知, $\\triangle N B E \\cong \\triangle M E I$. 于是 $\\angle E M I=\\angle B N E=90^{\\circ}+\\angle B F E=180^{\\circ}-\\angle E F I$, 故 $E, F, I, M$ 四点共圆. 进而可知 $\\angle A F M=90^{\\circ}+\\angle I F M=90^{\\circ}+\\angle I E M=\\angle A G M$, 从而 $A, F, G, M$ 四点共圆.再由 $\\angle D A G=\\angle D M G=90^{\\circ}$ 知, $A, G, M, D$ 四点共圆, 所以 $A, F, G, M, D$ 五点共圆.从而 $\\angle D F G=\\angle D A G=90^{\\circ}$ ,即 $D F \\perp F G$.']
['证明题']
false
null
Need_human_evaluate
null
34
Algebra
null
设 $n, k, m$ 是正整数, 满足 $k \geqslant 2$, 且 $n \leqslant m<\frac{2 k-1}{k} n$. 设 $A$ 是 $\{1,2, \cdots, m\}$ 的 $n$ 元子集. 证明: 区间 $\left(0, \frac{n}{k-1}\right)$ 中每个整数均可表示为 $a-a^{\prime}$, 其中 $a, a^{\prime} \in A$.
['用反证法. 假设存在整数 $x \\in\\left(0, \\frac{n}{k-1}\\right)$ 不可表示为 $a-a^{\\prime}, a, a^{\\prime} \\in A$. 作带余除法 $m=x q+r$, 其中 $0 \\leqslant r<x$. 将 $1,2, \\cdots, m$ 按模 $x$ 的同余类划分成 $x$ 个公差为 $x$的等差数列, 其中 $r$ 个等差数列有 $q+1$ 项, $x-r$ 个等差数列有 $q$ 项. 由于 $A$ 中没有两数之差为 $x$, 故 $A$ 不能包含以 $x$ 为公差的等差数列的相邻两项. 从而 $n=|A| \\leqslant r\\left[\\frac{q+1}{2}\\right]+(x-r)\\left[\\frac{q}{2}\\right]=\\left\\{\\begin{array}{l}x \\cdot \\frac{q+1}{2}, 2 \\nmid q \\\\ x \\cdot \\frac{q}{2}+r, 2 \\mid q\\end{array}\\right. \\quad \\quad (1)$\n\n这里 $\\lceil\\alpha\\rceil$ 表示不小于 $\\alpha$ 的最小整数. 由条件, 我们有\n\n$n>\\frac{k}{2 k-1} m=\\frac{k}{2 k-1}(x q+r) \\quad \\quad (2)$.\n\n又 $x \\in\\left(0, \\frac{n}{k-1}\\right)$, 故 $n>(k-1) x \\quad \\quad (3)$.\n\n情形一\n\n$q$ 是奇数, 则由 (1) 知, $n \\leqslant x \\cdot \\frac{q+1}{2}$. (4)\n\n结合 (2), (4) 可知, $x \\cdot \\frac{q+1}{2} \\geqslant n>\\frac{k}{2 k-1}(x q+r) \\geqslant \\frac{k}{2 k-1} x q$, 从而 $q<2 k-1$. 再由 $q$ 是奇数可知, $q \\leqslant 2 k-3$, 于是 $n \\leqslant x \\cdot \\frac{q+1}{2} \\leqslant(k-1) x$, 与 (3)矛盾.\n\n情形二\n\n$q$ 是偶数, 则由 (1) 知, $n \\leqslant x \\cdot \\frac{q}{2}+r$. (5)\n\n结合 (2), (5) 可知, $x \\cdot \\frac{q}{2}+r \\geqslant n>\\frac{k}{2 k-1}(x q+r)$, 从而 $\\frac{x q}{2(2 k-1)}<\\frac{k-1}{2 k-1} r<\\frac{(k-1) x}{2 k-1}$,故 $q<2(k-1)$. 再由 $q$ 是偶数可知, $q \\leqslant 2 k-4$, 于是 $n \\leqslant x \\cdot \\frac{q}{2}+r \\leqslant(k-2) x+r<(k-1) x$,与 (3) 矛盾.\n\n综上可知, 反证法假设不成立, 结论获证.']
null
false
null
null
null
35
Sequence
null
数列 $\left\{a_{n}\right\}$ 定义如下: $a_{1}$ 是任意正整数, 对整数 $n \geqslant 1, a_{n+1}$ 是与 $\sum_{i=1}^{n} a_{i}$ 互素, 且不等于 $a_{1}, \cdots, a_{n}$ 的最小整数. 证明: 每个正整数均在数列 $\left\{a_{n}\right\}$ 中出现.
['显然 $a_{1}=1$ 或 $a_{2}=1$. 下面考虑整数 $m>1$, 设 $m$ 有 $k$ 个不同素因子, 我们对 $k$ 归纳证明 $m$ 在 $\\left\\{a_{n}\\right\\}$ 中出现. 记 $S_{n}=a_{1}+\\cdots+a_{n}, n \\geqslant 1$.\n\n$k=1$ 时, $m$ 是素数方幂, 设 $m=p^{a}$, 其中 $a>0, p$ 是素数. 假设 $m$ 不在 $\\left\\{a_{n}\\right\\}$ 中出现. 由于 $\\left\\{a_{n}\\right\\}$ 各项互不相同, 因此存在正整数 $N$, 当 $n \\geqslant N$ 时, 都有 $a_{n}>p^{a}$. 若对某个 $n \\geqslant N, p \\nmid S_{n}$, 那么 $p^{a}$ 与 $S_{n}$ 互素, 又 $a_{1}, \\cdots, a_{n}$ 中无一项是 $p^{a}$, 故由数列定义知 $a_{n+1} \\leqslant p^{a}$, 但是 $a_{n+1}>p^{a}$, 矛盾!\n\n因此对每个 $n \\geqslant N$, 都有 $p \\mid S_{n}$, 但由 $p \\mid S_{n+1}$ 及 $p \\mid S_{n}$ 知 $p \\mid a_{n+1}$, 从而 $a_{n+1}$ 与 $S_{n}$ 不互素, 这与 $a_{n+1}$ 的定义矛盾.\n\n假设 $k \\geqslant 2$, 且结论对 $k-1$ 成立. 设 $m$ 的标准分解为 $m=p_{1}^{a_{1}} p_{2}^{a_{2}} \\cdots p_{k}^{a_{k}}$, 假设 $m$ 不在 $\\left\\{a_{n}\\right\\}$ 中出现, 于是存在正整数 $N^{\\prime}$, 当 $n \\geqslant N^{\\prime}$ 时, 都有 $a_{n}>m$. 取充分大的正整数 $\\beta_{1}, \\cdots, \\beta_{k-1}$, 使得 $M=p_{1}^{\\beta_{1}} \\cdots p_{k-1}^{\\beta_{k-1}}>\\max _{1 \\leqslant n \\leqslant N^{\\prime}} a_{n}$.\n\n我们证明, 对 $n \\geqslant N^{\\prime}$, 有 $a_{n+1} \\neq M$. 对任意 $n \\geqslant N^{\\prime}$, 若 $S_{n}$ 与 $p_{1} p_{2} \\cdots p_{k}$ 互素, 则 $m$与 $S_{n}$ 互素, 又 $m$ 在 $a_{1}, \\cdots, a_{n}$ 中均未出现, 而 $a_{n+1}>m$, 这与数列的定义矛盾. 因此我们推出: 对任意 $n \\geqslant N^{\\prime}, S_{n}$ 与 $p_{1} p_{2} \\cdots p_{k}$ 不互素. (*)\n\n情形一\n\n若存在 $i(1 \\leqslant i \\leqslant k-1)$, 使得 $p_{i} \\mid S_{n}$, 因 $\\left(a_{n+1}, S_{n}\\right)=1$, 故 $p_{i} \\nmid a_{n+1}$, 从而 $a_{n+1} \\neq M$ (因 $\\left.p_{i} \\mid M\\right)$.\n\n情形二\n\n若对每个 $i(1 \\leqslant i \\leqslant k-1)$, 均有 $p_{i} \\nmid S_{n}$, 则由 $(*)$ 知必有 $p_{k} \\mid S_{n}$. 于是 $p_{k} \\nmid a_{n+1}$, 进而 $p_{k} \\nmid S_{n}+a_{n+1}$, 即 $p_{k} \\nmid S_{n+1}$. 故由 (*) 知, 存在 $i_{0}\\left(1 \\leqslant i_{0} \\leqslant k-1\\right)$, 使得 $p_{i_{0}} \\mid S_{n+1}$, 再由 $S_{n+1}=S_{n}+a_{n+1}$ 及前面的假设 $p_{i} \\nmid S_{n}(1 \\leqslant i \\leqslant k-1)$, 可知 $p_{i_{0}} \\nmid a_{n+1}$, 故 $a_{n+1} \\neq M$.因此对 $n \\geqslant N^{\\prime}+1$, 均有 $a_{n} \\neq M$, 而 $M>\\max _{1 \\leqslant i \\leqslant N^{\\prime}} a_{n}$, 故 $M$ 不在 $\\left\\{a_{n}\\right\\}$ 中出现, 这与归纳假设矛盾. 因此, 若 $m$ 有 $k$ 个不同素因子, 则 $m$ 一定在 $\\left\\{a_{n}\\right\\}$ 中出现. 由数学归纳法知, 所有正整数均在 $\\left\\{a_{n}\\right\\}$ 中出现.']
null
false
null
null
null
36
Sequence
null
已知数列 $\left\{a_{n}\right\}: a_{1}=7, \frac{a_{n+1}}{a_{n}}=a_{n}+2, n=1,2,3, \cdots$. 求满足 $a_{n}>4^{2018}$ 的最小正整数 $n$.
['由 $\\frac{a_{n+1}}{a_{n}}=a_{n}+2$ 可知 $a_{n+1}+1=\\left(a_{n}+1\\right)^{2}$. 因此 $a_{n}+1=\\left(a_{1}+1\\right)^{2^{n-1}}=8^{2^{n-1}}=2^{3 \\times 2^{n-1}}$,\n故 $a_{n}=2^{3 \\times 2^{n-1}}-1$. 显然 $\\left\\{a_{n}\\right\\}$ 单调递增. 由于 $a_{11}=2^{3072}-1<2^{4036}=4^{2018}, a_{12}=$ $2^{6144}-1>2^{4036}=4^{2018}$, 故满足题目条件的 $n$ 的最小值是 12 .']
['$12$']
false
null
Numerical
null
37
Plane Geometry
null
如图所示, 在平面直角坐标系 $x O y$ 中, $A, B$ 与 $C, D$ 分别是椭圆 $\Gamma: \frac{x^{2}}{a^{2}}+\frac{y^{2}}{b^{2}}=1(a>$ $b>0)$ 的左右顶点与上下顶点. 设 $P, Q$ 是 $\Gamma$ 上且位于第一象限的两点, 满足 $O Q \| A P$, $M$ 是线段 $A P$ 的中点, 射线 $O M$ 与椭圆交于点 $R$. 证明: 线段 $O Q, O R, B C$ 能构成一个直角三角形. <img_4135>
['设点 $P$ 坐标为 $\\left(x_{0}, y_{0}\\right)$. 由于 $\\overrightarrow{O Q}\\|\\overrightarrow{A P}, \\overrightarrow{A P}=\\overrightarrow{O P}-\\overrightarrow{O A} ; \\overrightarrow{O R}\\| \\overrightarrow{O M}, \\overrightarrow{O M}=\\frac{1}{2}(\\overrightarrow{O P}+$ $\\overrightarrow{O A})$, 故存在实数 $\\lambda, \\mu$, 使得 $\\overrightarrow{O Q}=\\lambda(\\overrightarrow{O P}-\\overrightarrow{O A}), \\overrightarrow{O R}=\\mu(\\overrightarrow{O P}+\\overrightarrow{O A})$. 此时点 $Q, R$的坐标可分别表示是 $\\left(\\lambda\\left(x_{0}+a\\right), \\lambda y_{0}\\right),\\left(\\mu\\left(x_{0}-a\\right), \\mu y_{0}\\right)$. 由于点 $Q, R$ 都在椭圆上, 所以 $\\lambda^{2}\\left(\\frac{\\left(x_{0}+a\\right)^{2}}{a^{2}}+\\frac{y_{0}^{2}}{b^{2}}\\right)=\\mu^{2}\\left(\\frac{\\left(x_{0}-a\\right)^{2}}{a^{2}}+\\frac{y_{0}^{2}}{b^{2}}\\right)=1$.\n\n结合 $\\frac{x_{0}^{2}}{a^{2}}+\\frac{y_{0}^{2}}{b^{2}}=1$ 知, 上式可化为 $\\lambda^{2}\\left(2+\\frac{2 x_{0}}{a}\\right)=\\mu^{2}\\left(2-\\frac{2 x_{0}}{a}\\right)=1$, 解得 $\\lambda^{2}=$ $\\frac{a}{2\\left(a+x_{0}\\right)}, \\mu^{2}=\\frac{a}{2\\left(a-x_{0}\\right)}$.\n\n\n\n因此 $|O Q|^{2}+|O R|^{2}=\\lambda^{2}\\left(\\left(x_{0}+a\\right)^{2}+y_{0}^{2}\\right)+\\mu^{2}\\left(\\left(x_{0}-a\\right)^{2}+y_{0}^{2}\\right)$\n\n$=\\frac{a}{2\\left(a+x_{0}\\right)}\\left(\\left(x_{0}+a\\right)^{2}+y_{0}^{2}\\right)+\\frac{a}{2\\left(a-x_{0}\\right)}\\left(\\left(x_{0}-a\\right)^{2}+y_{0}^{2}\\right)$\n\n$=\\frac{a\\left(a+x_{0}\\right)}{2}+\\frac{a y_{0}^{2}}{2\\left(a+x_{0}\\right)}+\\frac{a\\left(a-x_{0}\\right)}{2}+\\frac{a y_{0}^{2}}{2\\left(a-x_{0}\\right)}$\n\n$=a^{2}+\\frac{a y_{0}^{2}}{2}\\left(\\frac{1}{a+x_{0}}+\\frac{1}{a-x_{0}}\\right)$\n\n$=a^{2}+\\frac{a y_{0}^{2}}{2} \\cdot \\frac{2 a}{a^{2}-x_{0}^{2}}=a^{2}+\\frac{a^{2} \\cdot b^{2}\\left(1-\\frac{x_{0}^{2}}{a^{2}}\\right)}{a^{2}-x_{0}^{2}}=a^{2}+b^{2}=|B C|^{2}$.\n\n从而线段 $O Q, O R, B C$ 能构成一个直角三角形.']
null
false
null
null
null
37
Plane Geometry
null
如图所示, 在平面直角坐标系 $x O y$ 中, $A, B$ 与 $C, D$ 分别是椭圆 $\Gamma: \frac{x^{2}}{a^{2}}+\frac{y^{2}}{b^{2}}=1(a>$ $b>0)$ 的左右顶点与上下顶点. 设 $P, Q$ 是 $\Gamma$ 上且位于第一象限的两点, 满足 $O Q \| A P$, $M$ 是线段 $A P$ 的中点, 射线 $O M$ 与椭圆交于点 $R$. 证明: 线段 $O Q, O R, B C$ 能构成一个直角三角形. ![](https://cdn.mathpix.com/cropped/2023_12_20_aa9923241db0221fe1d7g-1.jpg?height=502&width=600&top_left_y=1328&top_left_x=771)
['设点 $P$ 坐标为 $\\left(x_{0}, y_{0}\\right)$. 由于 $\\overrightarrow{O Q}\\|\\overrightarrow{A P}, \\overrightarrow{A P}=\\overrightarrow{O P}-\\overrightarrow{O A} ; \\overrightarrow{O R}\\| \\overrightarrow{O M}, \\overrightarrow{O M}=\\frac{1}{2}(\\overrightarrow{O P}+$ $\\overrightarrow{O A})$, 故存在实数 $\\lambda, \\mu$, 使得 $\\overrightarrow{O Q}=\\lambda(\\overrightarrow{O P}-\\overrightarrow{O A}), \\overrightarrow{O R}=\\mu(\\overrightarrow{O P}+\\overrightarrow{O A})$. 此时点 $Q, R$的坐标可分别表示是 $\\left(\\lambda\\left(x_{0}+a\\right), \\lambda y_{0}\\right),\\left(\\mu\\left(x_{0}-a\\right), \\mu y_{0}\\right)$. 由于点 $Q, R$ 都在椭圆上, 所以 $\\lambda^{2}\\left(\\frac{\\left(x_{0}+a\\right)^{2}}{a^{2}}+\\frac{y_{0}^{2}}{b^{2}}\\right)=\\mu^{2}\\left(\\frac{\\left(x_{0}-a\\right)^{2}}{a^{2}}+\\frac{y_{0}^{2}}{b^{2}}\\right)=1$.\n\n结合 $\\frac{x_{0}^{2}}{a^{2}}+\\frac{y_{0}^{2}}{b^{2}}=1$ 知, 上式可化为 $\\lambda^{2}\\left(2+\\frac{2 x_{0}}{a}\\right)=\\mu^{2}\\left(2-\\frac{2 x_{0}}{a}\\right)=1$, 解得 $\\lambda^{2}=$ $\\frac{a}{2\\left(a+x_{0}\\right)}, \\mu^{2}=\\frac{a}{2\\left(a-x_{0}\\right)}$.\n\n\n\n因此 $|O Q|^{2}+|O R|^{2}=\\lambda^{2}\\left(\\left(x_{0}+a\\right)^{2}+y_{0}^{2}\\right)+\\mu^{2}\\left(\\left(x_{0}-a\\right)^{2}+y_{0}^{2}\\right)$\n\n$=\\frac{a}{2\\left(a+x_{0}\\right)}\\left(\\left(x_{0}+a\\right)^{2}+y_{0}^{2}\\right)+\\frac{a}{2\\left(a-x_{0}\\right)}\\left(\\left(x_{0}-a\\right)^{2}+y_{0}^{2}\\right)$\n\n$=\\frac{a\\left(a+x_{0}\\right)}{2}+\\frac{a y_{0}^{2}}{2\\left(a+x_{0}\\right)}+\\frac{a\\left(a-x_{0}\\right)}{2}+\\frac{a y_{0}^{2}}{2\\left(a-x_{0}\\right)}$\n\n$=a^{2}+\\frac{a y_{0}^{2}}{2}\\left(\\frac{1}{a+x_{0}}+\\frac{1}{a-x_{0}}\\right)$\n\n$=a^{2}+\\frac{a y_{0}^{2}}{2} \\cdot \\frac{2 a}{a^{2}-x_{0}^{2}}=a^{2}+\\frac{a^{2} \\cdot b^{2}\\left(1-\\frac{x_{0}^{2}}{a^{2}}\\right)}{a^{2}-x_{0}^{2}}=a^{2}+b^{2}=|B C|^{2}$.\n\n从而线段 $O Q, O R, B C$ 能构成一个直角三角形.']
['证明题']
false
null
Need_human_evaluate
null
38
Elementary Functions
null
设 $a, b$ 是实数, 函数 $f(x)=a x+b+\frac{9}{x}$. 证明: 存在 $x_{0} \in[1,9]$, 使得 $\left|f\left(x_{0}\right)\right| \geqslant 2$.
['只需证明存在 $u, v \\in[1,9]$, 满足 $|f(u)-f(v)| \\geqslant 4$, 进而由绝对值不等式得 $|f(u)|+$ $|f(v)| \\geqslant|f(u)-f(v)| \\geqslant 4$, 故 $|f(u)| \\geqslant 2$ 与 $|f(v)| \\geqslant 2$ 中至少有一个成立. 当 $a \\in$ $\\left(-\\infty, \\frac{1}{2}\\right] \\bigcup\\left[\\frac{3}{2},+\\infty\\right)$ 时, 有 $|f(1)-f(9)|=|(a+b+9)-(9 a+b+1)|=8|1-a| \\geqslant 4$.当 $\\frac{1}{2}<a<\\frac{3}{2}$ 时, 有 $\\frac{3}{\\sqrt{a}} \\in[1,9]$. 再分两种情况: 若 $\\frac{1}{2}<a \\leqslant 1$, 则 $\\left|f(1)-f\\left(\\frac{3}{\\sqrt{a}}\\right)\\right|=$ $|(a+b+9)-(6 \\sqrt{a}+b)|=(3-\\sqrt{a})^{2} \\geqslant 4$. 若 $1<a<\\frac{3}{2}$, 则 $\\left|f(9)-f\\left(\\frac{3}{\\sqrt{a}}\\right)\\right|=$ $|(9 a+b+1)-(6 \\sqrt{a}+b)|=(3 \\sqrt{a}-1)^{2} \\geqslant 4$.\n\n综上可知, 存在 $u, v \\in[1,9]$, 满足 $|f(u)-f(v)| \\geqslant 4$, 从而命题得证.', '用反证法. 假设对任意 $x \\in[1,9]$, 均有 $|f(x)|<2$, 则 $|f(1)|<2,|f(3)|<2,|f(9)|<2$.\n\n易知 $f(1)=a+b+9$, (1)\n\n$f(3)=3 a+b+3$, (2)\n\n$f(9)=9 a+b+1$. (3)\n\n由 (1), (2) 得, $2 a-6=f(2)-f(1)$; 又由 (2), (3) 得, $6 a-2=f(3)-f(2)$. 由上述两式消去 $a$, 可知 $f(3)-4 f(2)+3 f(1)=(6 a-2)-3 \\cdot(2 a-6)=16$. 但 $f(3)-4 f(2)+3 f(1)<2+4 \\cdot 2+3 \\cdot 2=16$, 矛盾! 从而命题得证.']
null
false
null
null
null
39
Plane Geometry
null
如图所示, 在等腰 $\triangle A B C$ 中, $A B=A C$, 边 $A C$ 上一点 $D$ 及 $B C$ 延长线上一点 $E$满足 $\frac{A D}{D C}=\frac{B C}{2 C E}$, 以 $A B$ 为直径的圆 $w$ 与线段 $D E$ 交于一点 $F$. 证明: $B, C, F, D$ 四点共圆. ![](https://cdn.mathpix.com/cropped/2023_12_20_2decfda4e638fc803526g-1.jpg?height=513&width=880&top_left_y=346&top_left_x=631)
['如下图, 取 $B C$ 中点 $H$, 则由 $A B=A C$ 知 $A H \\perp B C$, 故 $H$ 在圆 $w$ 上.\n\n![](https://cdn.mathpix.com/cropped/2023_12_20_2decfda4e638fc803526g-1.jpg?height=502&width=879&top_left_y=1294&top_left_x=634)\n\n延长 $F D$ 至 $G$, 使得 $A G \\| B C$, 结合已知条件得, $\\frac{A G}{C E}=\\frac{A D}{D C}=\\frac{B C}{2 C E}$, 故 $A G=$ $\\frac{1}{2} B C=B H=H C$, 从而 $A G B H$ 为矩形, $A G H C$ 为平行四边形. 由 $A G B H$ 为矩形知, $G$ 亦在圆 $w$ 上. 故 $\\angle H G F=\\angle H B F$. 又 $A G H C$ 为平行四边形, 由 $A C \\| G H$,得 $\\angle C D F=\\angle H G F$. 所以 $\\angle C D F=\\angle H B F=\\angle C B F$, 故 $B, C, F, D$ 四点共圆.']
['证明题']
false
null
Need_human_evaluate
null
39
Plane Geometry
null
如图所示, 在等腰 $\triangle A B C$ 中, $A B=A C$, 边 $A C$ 上一点 $D$ 及 $B C$ 延长线上一点 $E$满足 $\frac{A D}{D C}=\frac{B C}{2 C E}$, 以 $A B$ 为直径的圆 $w$ 与线段 $D E$ 交于一点 $F$. 证明: $B, C, F, D$ 四点共圆. <img_4227>
['如下图, 取 $B C$ 中点 $H$, 则由 $A B=A C$ 知 $A H \\perp B C$, 故 $H$ 在圆 $w$ 上.\n\n<img_4161>\n\n延长 $F D$ 至 $G$, 使得 $A G \\| B C$, 结合已知条件得, $\\frac{A G}{C E}=\\frac{A D}{D C}=\\frac{B C}{2 C E}$, 故 $A G=$ $\\frac{1}{2} B C=B H=H C$, 从而 $A G B H$ 为矩形, $A G H C$ 为平行四边形. 由 $A G B H$ 为矩形知, $G$ 亦在圆 $w$ 上. 故 $\\angle H G F=\\angle H B F$. 又 $A G H C$ 为平行四边形, 由 $A C \\| G H$,得 $\\angle C D F=\\angle H G F$. 所以 $\\angle C D F=\\angle H B F=\\angle C B F$, 故 $B, C, F, D$ 四点共圆.']
null
false
null
null
null
40
Algebra
null
设集合 $A=\{1,2, \cdots, n\}, X, Y$ 均为 $A$ 的非空子集 (允许 $X=Y$ ). $X$ 中的最大元与 $Y$ 中的最小元分别记为 $\max X, \min Y$. 求满足 $\max X>\min Y$ 的有序集合对 $(X, Y)$ 的数目.
['先计算满足 $\\max X \\leqslant \\min Y$ 的有序集合对 $(X, Y)$ 的数目. 对给定的 $m=\\max X$, 集合 $X$ 是集合 $\\{1,2, \\cdots, m-1\\}$ 的任意一个子集与 $\\{m\\}$ 的并,故并有 $2^{m-1}$ 种取法. 又 $\\min Y \\geqslant M$, 故 $Y$ 是 $\\{m, m+1, \\cdots, n\\}$ 的任意一个非空子集, 共有 $2^{n+1-m}-1$ 种取法. 因此, 满足 $\\max X \\leqslant \\min Y$ 的有序集合对 $(X, Y)$ 的数目是 $\\sum_{m=1}^{n} 2^{m-1}\\left(2^{n+1-m}-1\\right)=\\sum_{m=1}^{n} 2^{n}-\\sum_{m=1}^{n} 2^{m-1}=n \\cdot 2^{n}-2^{n}+1$. 由于有序集合对 $(X, Y)$ 有 $\\left(2^{n}-1\\right) \\cdot\\left(2^{n}-1\\right)=\\left(2^{n}-1\\right)^{2}$ 个, 于是满足 $\\max X>\\min Y$的有序集合对 $(X, Y)$的数目是$\\left(2^{n}-1\\right)^{2}-n \\cdot 2^{n}+2^{n}-1=2^{2 n}-2^{n}(n+1)$.']
['$2^{2 n}-2^{n}(n+1)$']
false
null
Expression
null
41
Number Theory
null
给定整数 $a \geqslant 2$. 证明: 对任意正整数 $n$, 存在正整数 $k$, 使得连续 $n$ 个数 $a^{k}+1, a^{k}+$ $2, \cdots, a^{k}+n$ 均是合数.
['设 $i_{1}<i_{2}<\\cdots<i_{r}$ 是 $1,2, \\cdots, n$ 中与 $a$ 互素的全体整数, 则对 $1 \\leqslant i \\leqslant n, i \\notin$ $\\left\\{i_{1}, i_{2}, \\cdots, i_{r}\\right\\}$, 无论正整数 $k$ 如何取值, $a^{k}+i$ 均与 $a$ 不互素且大于 $a$, 故 $a^{k}+i$ 为合数.\n\n对任意 $j=1,2, \\cdots, r$, 因 $a+i_{j}>1$, 故 $a+i_{j}$ 有素因子 $p_{j}$.\n\n我们有 $\\left(p_{j}, a\\right)=1$ (否则, 因 $p_{j}$ 是素数, 故 $p_{j} \\mid a$, 但 $p_{j} \\mid a+i_{j}$, 从而 $p_{j} \\mid i_{j}$, 故 $a, i_{j}$不互素, 与 $i_{j}$ 的取法矛盾). 因此, 由费马小定理知, $a^{p_{j}-1} \\equiv 1\\left(\\bmod p_{j}\\right)$. 现取 $k=$ $\\left(p_{1}-1\\right)\\left(p_{2}-1\\right) \\cdots\\left(p_{r}-1\\right)+1$. 对任意 $j=1,2, \\cdots, r$, 注意到 $k \\equiv 1\\left(\\bmod p_{j}-1\\right)$,故有 $a^{k}+i_{j} \\equiv a+i_{j} \\equiv 0\\left(\\bmod p_{j}\\right)$. 又 $a^{k}+i_{j}>a+i_{j} \\geqslant p_{j}$, 故 $a^{k}+i_{j}$ 为合数.\n\n综上所述, 当 $k=\\left(p_{1}-1\\right)\\left(p_{2}-1\\right) \\cdots\\left(p_{r}-1\\right)+1$ 时, $a^{k}+1, a^{k}+2, \\cdots, a^{k}+n$ 均是合数.']
null
false
null
null
null
42
Algebra
null
设 $k, m$ 为实数, 不等式 $\left|x^{2}-k x-m\right| \leqslant 1$ 对所有 $x \in[a, b]$ 成立. 证明: $b-a \leqslant 2 \sqrt{2}$.
['根据题意, 有\n\n$$\n\\forall x \\in[a, b], x^{2}-1 \\leqslant k x+m \\leqslant x^{2}+1\n$$\n\n如下图, 考虑直线 $y=k x+m$ 截图中阴影部分得到的线段在 $x$ 轴上的投影长度. 不妨设 $k \\geqslant 0$. 分别记\n\n$$\n\\Delta_{1}=k^{2}+4 m-4, \\Delta_{2}=k^{2}+4 m+4 \\text {. }\n$$\n\n\n\n<img_4062>\n\n情形 1 : 直线 $y=k x+m$ 与抛物线 $y=x^{2}+1$ 相交, 即 $\\Delta_{1}>0$. 且有\n\n$$\n\\begin{aligned}\nb-a & \\leqslant\\left|\\frac{k+\\sqrt{\\Delta_{2}}}{2}-\\frac{k+\\sqrt{\\Delta_{1}}}{2}\\right| \\\\\n& =\\frac{1}{2}\\left|\\sqrt{k^{2}+4 m+4}-\\sqrt{k^{2}+4 m-4}\\right| \\\\\n& =\\frac{4}{\\sqrt{k^{2}+4 m+4}+\\sqrt{k^{2}+4 m-4}} \\\\\n& <\\frac{4}{\\sqrt{8}+\\sqrt{0}} \\\\\n& =\\sqrt{2} .\n\\end{aligned}\n$$\n\n情形 2: 直线 $y=k x+m$ 与抛物线 $y=x^{2}+1$ 相离且与抛物线 $y=x^{2}-1$ 相交, 此时将直线 $y=k x+m$ 向上平移, 直至与抛物线 $y=x^{2}+1$ 相切可以使得线段投影长度增大, 因此此时无法取得最大值.\n\n情形 3: 直线 $y=k x+m$ 与抛物线 $y=x^{2}+1$ 相切, 即 $\\Delta_{1}=0$. 此时\n\n$$\nb-a \\leqslant \\frac{2 \\sqrt{\\Delta_{2}}}{2}=2 \\sqrt{2}\n$$\n\n综上所述, 原命题得证.', '根据题意, 有\n\n$$\n\\left\\{\\begin{array}{l}\n\\left|b^{2}-k b-m\\right| \\leqslant 1 \\\\\n\\left|a^{2}-k a-m\\right| \\leqslant 1 \\\\\n\\left|\\left(\\frac{b+a}{2}\\right)^{2}-k \\cdot \\frac{b+a}{2}-m\\right| \\leqslant 1\n\\end{array}\\right.\n$$\n\n于是\n\n$$\n(b-a)^{2} \\leqslant 2\\left|b^{2}-k b-m\\right|+2\\left|a^{2}-k a-m\\right|+4\\left|\\left(\\frac{b+a}{2}\\right)^{2}-k \\cdot \\frac{b+a}{2}-m\\right| \\leqslant 8,\n$$\n\n于是原命题得证.']
null
false
null
null
null
43
Algebra
null
设 $x_{1}, x_{2}, x_{3}$ 是非负实数, 满足 $x_{1}+x_{2}+x_{3}=1$, 求 $\left(x_{1}+3 x_{2}+5 x_{3}\right)\left(x_{1}+\frac{x_{2}}{3}+\frac{x_{3}}{5}\right)$的最小值和最大值.
['设题中代数式为 $M$, 则\n\n$$\n\\begin{aligned}\nM & =x_{1}^{2}+x_{2}^{2}+x_{3}^{2}+\\frac{10}{3} x_{1} x_{2}+\\frac{34}{15} x_{2} x_{3}+\\frac{26}{5} x_{3} x_{1} \\\\\n& =\\left(x_{1}+x_{2}+x_{3}\\right)^{2}+\\frac{4}{15}\\left(5 x_{1} x_{2}+x_{2} x_{3}+12 x_{3} x_{1}\\right) \\\\\n& =1+\\frac{4}{15}\\left(5 x_{1} x_{2}+x_{2} x_{3}+12 x_{3} x_{1}\\right) .\n\\end{aligned}\n$$\n\n一方面,有\n\n$$\n5 x_{1} x_{2}+x_{2} x_{3}+12 x_{3} x_{1} \\geqslant 0\n$$\n\n等号当 $\\left(x_{1}, x_{2}, x_{3}\\right)=(1,0,0)$ 时取得, 因此 $M$ 的最小值为 1 .\n\n另一方面, 有\n\n$$\n\\begin{aligned}\n5 x_{1} x_{2}+x_{2} x_{3}+12 x_{3} x_{1} & \\leqslant 6 x_{1} x_{2}+6 x_{2} x_{3}+6 x_{3} x_{1}+3 x_{3}^{2}+3 x_{1}^{2} \\\\\n& \\leqslant 3\\left(x_{1}+x_{2}+x_{3}\\right)^{2} \\\\\n& =3\n\\end{aligned}\n$$\n\n等号当 $\\left(x_{1}, x_{2}, x_{3}\\right)=\\left(\\frac{1}{2}, 0, \\frac{1}{2}\\right)$ 时取得, 因此 $M$ 的最大值为 $\\frac{9}{5}$.', '设题中代数式为 $M$ ,则\n\n$$\nM=x_{1}^{2}+x_{2}^{2}+x_{3}^{2}+\\frac{10}{3} x_{1} x_{2}+\\frac{34}{15} x_{2} x_{3}+\\frac{26}{5} x_{3} x_{1} \\geqslant\\left(x_{1}+x_{2}+x_{3}\\right)^{2}=1,\n$$\n\n等号当 $\\left(x_{1}, x_{2}, x_{3}\\right)=(1,0,0)$ 时取得, 因此 $M$ 的最小值为 1 .\n\n另一方面, 有\n\n$5 M=\\left(x_{1}+3 x_{2}+5 x_{3}\\right)\\left(5 x_{1}+\\frac{5}{3} x_{2}+x_{3}\\right) \\leqslant\\left(3 x_{1}+\\frac{7}{3} x_{2}+3 x_{3}\\right)^{2} \\leqslant\\left(3 x_{1}+3 x_{2}+3 x_{3}\\right)^{2}$,\n\n等号当 $\\left(x_{1}, x_{2}, x_{3}\\right)=\\left(\\frac{1}{2}, 0, \\frac{1}{2}\\right)$ 时取得, 因此 $M$ 的最大值为 $\\frac{9}{5}$.']
['$1 , \\frac{9}{5}$']
true
null
Numerical
null
44
Complex Numbers
null
设复数 $z_{1}, z_{2}$ 满足 $\operatorname{Re}\left(z_{1}\right)>0, \operatorname{Re}\left(z_{2}\right)>0$, 且 $\operatorname{Re}\left(z_{1}^{2}\right)=\operatorname{Re}\left(z_{2}^{2}\right)=2$, 其中 $\operatorname{Re}(z)$ 表示复数 $z$ 的实部. 求 $\operatorname{Re}\left(z_{1} z_{2}\right)$ 的最小值;
['设 $z_{k}=x_{k}+\\mathrm{i} y_{k}, k=1,2$. 根据题意, 有 $x_{1}, x_{2}>0$, 且\n\n$$\nx_{1}^{2}-y_{1}^{2}=x_{2}^{2}-y_{2}^{2}=2 \\text {. }\n$$\n\n于是 $P_{1}\\left(x_{1}, y_{2}\\right)$ 和 $P_{2}\\left(x_{2}, y_{2}\\right)$ 都是双曲线 $H: x^{2}-y^{2}=2$ 右支上的点. 而\n\n$$\n\\begin{aligned}\n\\operatorname{Re}\\left(z_{1} z_{2}\\right) & =x_{1} x_{2}-y_{1} y_{2} \\\\\n& =\\sqrt{y_{1}^{2}+2} \\cdot \\sqrt{y_{2}^{2}+2}-y_{1} y_{2} \\\\\n& =\\sqrt{y_{1}^{2} y_{2}^{2}+2 y_{1}^{2}+2 y_{2}^{2}+4}-y_{1} y_{2} \\\\\n& \\geqslant \\sqrt{y_{1}^{2} y_{2}^{2}+4 y_{1} y_{2}+4}-y_{1} y_{2} \\\\\n& =\\left|y_{1} y_{2}+2\\right|-y_{1} y_{2} \\\\\n& \\geqslant 2\n\\end{aligned}\n$$\n\n等号当 $y_{1}=y_{2}$ 时取得. 因此所求的最小值为 2 .']
['$2$']
false
null
Numerical
null
45
Complex Numbers
null
设复数 $z_{1}, z_{2}$ 满足 $\operatorname{Re}\left(z_{1}\right)>0, \operatorname{Re}\left(z_{2}\right)>0$, 且 $\operatorname{Re}\left(z_{1}^{2}\right)=\operatorname{Re}\left(z_{2}^{2}\right)=2$, 其中 $\operatorname{Re}(z)$ 表示复数 $z$ 的实部. 求 $\left|z_{1}+2\right|+\left|\overline{z_{2}}+2\right|-\left|\overline{z_{1}}-z_{2}\right|$ 的最小值.
['由共轭复数的性质, 有\n\n$$\nm=\\left|z_{1}+2\\right|+\\left|\\overline{z_{2}}+2\\right|-\\left|\\overline{z_{1}}-z_{2}\\right|=\\left|z_{1}-(-2)\\right|+\\left|\\overline{z_{2}}-(-2)\\right|-\\left|z_{1}-\\overline{z_{2}}\\right|\n$$\n\n记 $F_{1}(-2,0), F_{2}(2,0)$ 为双曲线 $H$ 的左、右焦点, $P_{2}^{\\prime}$ 为 $P_{2}$ 关于 $x$ 轴的对称点(该点仍然在双曲线 $H$ 上), 于是\n\n$$\nm=\\left|P_{1} F_{1}\\right|+\\left|P_{2}^{\\prime} F_{1}\\right|-\\left|P_{1} P_{2}^{\\prime}\\right|=2 \\sqrt{2}+\\left|P_{1} F_{2}\\right|+2 \\sqrt{2}+\\left|P_{2}^{\\prime} F_{2}\\right|-\\left|P_{1} P_{2}^{\\prime}\\right| \\geqslant 4 \\sqrt{2}\n$$\n\n等号当 $F_{2}$ 在线段 $P_{1} P_{2}^{\\prime}$ 时取得. 因此所求的最小值为 $4 \\sqrt{2}$.']
['$4 \\sqrt{2}$']
false
null
Numerical
null
46
Plane Geometry
null
如图, 在 $\triangle A B C$ 中, $A B=A C, I$ 为 $\triangle A B C$ 的内心. 以 $A$ 为圆心, $A B$ 为半径作圆 $\Gamma_{1}$, 以 $I$ 为圆心, $I B$ 为半径作圆 $\Gamma_{2}$, 过点 $B, I$ 的圆 $\Gamma_{3}$ 与 $\Gamma_{1}, \Gamma_{2}$ 分别交于点 $P, Q$ (不同于点 $B$ ). 设 $I P$ 与 $B Q$ 交于点 $R$. 证明: $B R \perp C R$. <img_4120>
['如下图, 连接 $I B, I C, I Q, P B, P C$.\n\n<img_4195>\n\n由于点 $Q$ 在圆 $\\Gamma_{2}$ 上, 故 $I B=I Q$, 所以\n\n$$\n\\angle I B Q=\\angle I Q B .\n$$\n\n又 $B, I, P, Q$ 四点共圆, 所以 $\\angle I Q B=\\angle I P B$, 于是 $\\angle I B Q=\\angle I P B$, 故 $\\triangle I B P$ 与\n$\\triangle I R B$ 相似, 从而有 $\\angle I R B=\\angle I B P$, 且\n\n$$\n\\frac{I B}{I R}=\\frac{I P}{I B}\n$$\n\n注意到 $A B=A C$, 且 $I$ 为 $\\triangle A B C$ 的内心, 故 $I B=I C$, 所以\n\n$$\n\\frac{I C}{I R}=\\frac{I P}{I C}\n$$\n\n于是 $\\triangle I C P$ 与 $\\triangle I R C$ 相似, 故\n\n$$\n\\angle I R C=\\angle I C P .\n$$\n\n又点 $P$ 在圆 $\\Gamma_{1}$ 的弧 $B C$ 上, 故\n\n$$\n\\angle B P C=180^{\\circ}-\\frac{1}{2} \\angle A,\n$$\n\n因此\n\n$$\n\\begin{aligned}\n\\angle B R C & =\\angle I R B+\\angle I R C \\\\\n& =\\angle I B P+\\angle I C P \\\\\n& =360^{\\circ}-\\left(90^{\\circ}+\\frac{1}{2} \\angle A\\right)-\\left(180^{\\circ}-\\frac{1}{2} \\angle A\\right) \\\\\n& =90^{\\circ},\n\\end{aligned}\n$$\n\n故 $B R \\perp C R$.']
null
false
null
null
null
46
Plane Geometry
null
如图, 在 $\triangle A B C$ 中, $A B=A C, I$ 为 $\triangle A B C$ 的内心. 以 $A$ 为圆心, $A B$ 为半径作圆 $\Gamma_{1}$, 以 $I$ 为圆心, $I B$ 为半径作圆 $\Gamma_{2}$, 过点 $B, I$ 的圆 $\Gamma_{3}$ 与 $\Gamma_{1}, \Gamma_{2}$ 分别交于点 $P, Q$ (不同于点 $B$ ). 设 $I P$ 与 $B Q$ 交于点 $R$. 证明: $B R \perp C R$. ![](https://cdn.mathpix.com/cropped/2023_12_20_b1ee5c4b2679673c0c27g-1.jpg?height=551&width=468&top_left_y=815&top_left_x=840)
['如下图, 连接 $I B, I C, I Q, P B, P C$.\n\n![](https://cdn.mathpix.com/cropped/2023_12_20_b1ee5c4b2679673c0c27g-1.jpg?height=549&width=463&top_left_y=1553&top_left_x=842)\n\n由于点 $Q$ 在圆 $\\Gamma_{2}$ 上, 故 $I B=I Q$, 所以\n\n$$\n\\angle I B Q=\\angle I Q B .\n$$\n\n又 $B, I, P, Q$ 四点共圆, 所以 $\\angle I Q B=\\angle I P B$, 于是 $\\angle I B Q=\\angle I P B$, 故 $\\triangle I B P$ 与\n$\\triangle I R B$ 相似, 从而有 $\\angle I R B=\\angle I B P$, 且\n\n$$\n\\frac{I B}{I R}=\\frac{I P}{I B}\n$$\n\n注意到 $A B=A C$, 且 $I$ 为 $\\triangle A B C$ 的内心, 故 $I B=I C$, 所以\n\n$$\n\\frac{I C}{I R}=\\frac{I P}{I C}\n$$\n\n于是 $\\triangle I C P$ 与 $\\triangle I R C$ 相似, 故\n\n$$\n\\angle I R C=\\angle I C P .\n$$\n\n又点 $P$ 在圆 $\\Gamma_{1}$ 的弧 $B C$ 上, 故\n\n$$\n\\angle B P C=180^{\\circ}-\\frac{1}{2} \\angle A,\n$$\n\n因此\n\n$$\n\\begin{aligned}\n\\angle B R C & =\\angle I R B+\\angle I R C \\\\\n& =\\angle I B P+\\angle I C P \\\\\n& =360^{\\circ}-\\left(90^{\\circ}+\\frac{1}{2} \\angle A\\right)-\\left(180^{\\circ}-\\frac{1}{2} \\angle A\\right) \\\\\n& =90^{\\circ},\n\\end{aligned}\n$$\n\n故 $B R \\perp C R$.']
['证明题']
false
null
Need_human_evaluate
null
47
Sequence
null
设数列 $\left\{a_{n}\right\}$ 定义为 $a_{1}=1$, $$ a_{n+1}= \begin{cases}a_{n}+n, & a_{n} \leqslant n, \\ a_{n}-n, & a_{n}>n,\end{cases} n = 1, 2, \cdots . $$ 求满足 $a_{r}<r \leqslant 3^{2017}$ 的正整数 $r$ 的个数.
['先计算数列的前几项:\n\n| $n$ | 1 | 2 | 3 | 4 | 5 | 6 | 7 | 8 | 9 | 10 | 11 | 12 | 13 | 14 | 15 | 16 |\n| :---: | :---: | :---: | :---: | :---: | :---: | :---: | :---: | :---: | :---: | :---: | :---: | :---: | :---: | :---: | :---: | :---: |\n| $a_{n}$ | 1 | 2 | 4 | 1 | 5 | 10 | 4 | 11 | 3 | 12 | 2 | 13 | 1 | 14 | 28 | 13 |\n| | $=$ | $=$ | $>$ | $<$ | $=$ | $>$ | $<$ | $>$ | $<$ | $>$ | $<$ | $>$ | $<$ | $=$ | $>$ | $<$ |\n\n归纳出\n\n引理: 若 $a_{r}=r$, 则对 $k=1,2, \\cdots, r-1$, 有\n\n$$\n\\begin{aligned}\na_{r+2 k-1} & =2 r+k-1>r+2 k-1, \\\\\na_{r+2 k} & =r-k<r+2 k .\n\\end{aligned}\n$$\n\n\n\n对 $k$ 进行归纳证明.\n\n归纳基础: 当 $k=1$ 时, 由于 $a_{r}=r \\leqslant r$, 根据定义,有\n\n$$\n\\begin{aligned}\n& a_{r+1}=a_{r}+r=2 r>r+1, \\\\\n& a_{r+2}=a_{r+1}-(r+1)=r-1<r+2,\n\\end{aligned}\n$$\n\n命题成立.\n\n推理证明: 假设对某个 $k(1 \\leqslant k<r-1)$ 成立, 则由定义\n\n$$\n\\begin{aligned}\n& a_{r+2 k+1}=a_{r+2 k}+(r+2 k)=2 r+k>r+2 k+1, \\\\\n& a_{r+2 k+2}=a_{r+2 k+1}-(r+2 k+1)=r-k-1<r+2 k+2,\n\\end{aligned}\n$$\n\n于是命题对 $k+1$ 也成立.\n\n综上所述, 引理得证.\n\n根据引理, 将所有满足 $a_{r}=r$ 的正整数 $r$ 从小到大排列为\n\n$$\nr_{1}, r_{2}, \\cdots, r_{n}, \\cdots,\n$$\n\n则有\n\n$$\nr_{n+1}=3 r_{n}-1, n=1,2, \\cdots \\text {. }\n$$\n\n利用不动点法容易求得\n\n$$\nr_{n}=\\frac{1}{2} \\cdot 3^{n-1}+\\frac{1}{2}, n \\in \\mathbb{N}^{*}\n$$\n\n由于\n\n$$\nr_{2018}=\\frac{3^{2017}+1}{2}<3^{2017}<\\frac{3^{2018}+1}{2}=r_{2019},\n$$\n\n于是不超过 $3^{2017}$ 的正整数被如下分段\n\n$$\nr_{1}, r_{2}, \\cdots, r_{3}, \\cdots, r_{4}, \\cdots, \\cdots, \\cdots, r_{2017}, \\cdots, r_{2018}, \\cdots, 3^{2017}\n$$\n\n其中在 $r_{k}$ 和 $r_{k+1}$ 之间的数恰有一半满足 $a_{r}<r$, 在 $r_{2018}$ (这是一个偶数) 到 $3^{2017}$ 中所有的偶数 $r$ 均满足 $a_{r}<r$, 所有奇数 $r$ 均满足 $a_{r}>r$. 因此满足条件的正整数 $r$ 的个数为\n\n$$\n\\frac{1}{2}\\left(3^{2017}-2018-1\\right)=\\frac{3^{2017}-2019}{2} .\n$$']
['$\\frac{3^{2017}-2019}{2}$']
false
null
Numerical
null
48
Combinatorics
null
将 $33 \times 33$ 方格纸中每个小方格染三种颜色之一, 使得每种颜色的小方格的个数相等.若相邻两个小方格的颜色不同, 则称它们的公共边为 “分隔边”. 试求分隔边条数的最小值.
['如下图, 分隔边的条数 $L=56$.\n\n<img_4187>\n\n下面证明 $L \\geqslant 56$. 将方格纸的行从上至下依次记为 $A_{1}, A_{2}, \\cdots, A_{33}$, 列从左至右依次记为 $B_{1}, B_{2}, \\cdots, B_{33}$. 行 $A_{i}$ 中方格出现的颜色数记为 $n\\left(A_{i}\\right)$, 列 $B_{i}$ 中方格出现的颜色数记为 $n\\left(B_{i}\\right)$, 三种颜色分别记为 $c_{1}, c_{2}, c_{3}$, 对于一种颜色 $c_{j}$, 设 $n\\left(c_{j}\\right)$ 是含有 $c_{j}$ 色方格的行数与列数之和. 若 $A_{i}$ 行中含有 $c_{j}$ 色的方格, 则记 $\\delta\\left(A_{i}, c_{j}\\right)$ 为 1 , 否则记 $\\delta\\left(A_{i}, c_{j}\\right)$为 0 . 类似的定义 $\\delta\\left(B_{i}, c_{j}\\right)$, 于是\n\n$$\n\\begin{aligned}\n\\sum_{i=1}^{33}\\left(n\\left(A_{i}\\right)+n\\left(B_{i}\\right)\\right) & =\\sum_{i=1}^{33} \\sum_{j=1}^{3}\\left(\\delta\\left(A_{i}, c_{j}\\right)+\\delta\\left(B_{i}, c_{j}\\right)\\right) \\\\\n& =\\sum_{j=1}^{3} \\sum_{i=1}^{33}\\left(\\delta\\left(A_{i}, c_{j}\\right)+\\delta\\left(B_{j}, c_{j}\\right)\\right) \\\\\n& =\\sum_{j=1}^{3} n\\left(c_{j}\\right)\n\\end{aligned}\n$$\n\n由于染 $c_{j}$ 色的方格数为\n\n$$\n\\frac{1}{3} \\cdot 33^{2}=363\n$$\n\n设含有 $c_{j}$ 色方格的行有 $a$ 个, 列有 $b$ 个, 则 $c_{j}$ 色方格一定在这 $a$ 行和 $b$ 列的交叉方格中, 因此\n\n$$\na b \\geqslant 363\n$$\n\n从而\n\n$$\nn\\left(c_{j}\\right)=a+b \\geqslant 2 \\sqrt{a b}>2 \\sqrt{363}>38\n$$\n\n故\n\n$$\nn\\left(c_{j}\\right) \\geqslant 39, j=1,2,3\n$$\n\n\n\n由于在行 $A_{i}$ 中有 $n\\left(A_{i}\\right)$ 种颜色的方格, 因此至少有 $n\\left(A_{i}\\right)-1$ 条分割边, 同理在列 $B_{i}$中, 至少有 $n\\left(B_{i}\\right)-1$ 条分割边, 于是\n\n$$\n\\begin{aligned}\nL & \\geqslant \\sum_{i=1}^{33}\\left(n\\left(A_{i}\\right)-1\\right)+\\sum_{i=1}^{33}\\left(n\\left(B_{i}\\right)-1\\right) \\\\\n& =\\sum_{i=1}^{33}\\left(n\\left(A_{i}\\right)+n\\left(B_{i}\\right)\\right)-66 \\\\\n& =\\sum_{j=1}^{3} n\\left(c_{j}\\right)-66 .\n\\end{aligned}\n$$\n\n下面分两种情形讨论.\n\n情形 1: 有一行或一列全部方格同色. 不妨设有一行全部为 $c_{1}$ 色, 从而方格纸的 33 列中均含有 $c_{1}$ 色的方格, 由于 $c_{1}$ 色方格有 363 个, 故至少有 11 行中含有 $c_{1}$ 色方格, 于是\n\n$$\nn\\left(c_{1}\\right) \\geqslant 11+33=44\n$$\n\n于是\n\n$$\nL \\geqslant n\\left(c_{1}\\right)+n\\left(c_{2}\\right)+n\\left(c_{3}\\right)-66 \\geqslant 44+39+39-66=56 .\n$$\n\n情形 2: 没有一行也没有一列的全部方格同色, 则对任意 $1 \\leqslant i \\leqslant 33$, 具有 $n\\left(A_{i}\\right) \\geqslant 2$, $n\\left(B_{i}\\right) \\geqslant 2$, 从而可得\n\n$$\nL \\geqslant \\sum_{i=1}^{33}\\left(n\\left(A_{i}\\right)+n\\left(B_{i}\\right)\\right)-66 \\geqslant 33 \\cdot 4-66=66\n$$']
['$56$']
false
null
Numerical
null
49
Number Theory
null
设 $m, n$ 均是大于 1 的整数, $m \geqslant n . a_{1}, a_{2}, \cdots, a_{n}$ 是 $n$ 个不超过 $m$ 的互不相同的正整数, 且 $a_{1}, a_{2}, \cdots, a_{n}$ 互素. 证明: 对任意实数 $x$, 均存在一个 $i(1 \leqslant i \leqslant n)$, 使得 $\left\|a_{i} x\right\| \geqslant \frac{2}{m(m+1)}\|x\|$, 这里 $\|y\|$ 表示实数 $y$ 到与它最近的整数的距离.
['首先证明以下两个结论.\n\n结论一: 存在整数 $c_{1}, c_{2}, \\cdots, c_{n}$, 满足\n\n$$\nc_{1} a_{1}+c_{2} a_{2}+\\cdots+c_{n} a_{n}=1,\n$$\n\n并且 $\\left|c_{i}\\right| \\leqslant m, \\quad 1 \\leqslant i \\leqslant n$.\n\n证明: 由于 $\\left(a_{1}, a_{2}, \\cdots, a_{n}\\right)=1$, 由裴蜀定理,存在正整数 $c_{1}, c_{2}, \\cdots, c_{n}$ 满足\n\n$$\nc_{1} a_{1}+c_{2} a_{2}+\\cdots+c_{n} a_{n}=1\n$$\n\n\n\n下面证明, 通过调整, 存在一组 $c_{1}, c_{2}, \\cdots, c_{n}$ 满足上式, 且绝对值均不超过 $m$. 记\n\n$$\n\\begin{aligned}\n& S_{1}=S_{1}\\left(c_{1}, c_{2}, \\cdots, c_{n}\\right)=\\sum_{c_{i}>m} c_{i} \\geqslant 0 \\\\\n& S_{2}=S_{2}\\left(c_{1}, c_{2}, \\cdots, c_{n}\\right)=\\sum_{c_{i}<-m}\\left|c_{i}\\right| \\geqslant 0\n\\end{aligned}\n$$\n\n如果 $S_{1}>0$, 那么存在 $c_{i}>m>1$, 于是 $c_{i} a_{i}>1$, 又因为 $a_{1}, a_{2}, \\cdots, a_{n}$ 均为正数, 故由上式可知存在 $c_{j}<0$. 令\n\n$$\n\\left(c_{i}^{\\prime}, c_{j}^{\\prime}, c_{k}^{\\prime}\\right)=\\left(c_{i}-a_{j}, c_{j}+a_{i}, c_{k}\\right),\n$$\n\n其中 $1 \\leqslant k \\leqslant n, k \\neq i, j$, 则\n\n$$\nc_{1}^{\\prime} a_{1}+c_{2}^{\\prime} a_{2}+\\cdots+c_{n}^{\\prime} a_{n}=1\n$$\n\n并且\n\n$$\n0 \\leqslant m-a_{j}<c_{i}^{\\prime}<c_{i}, c_{j}<c_{j}^{\\prime}<a_{i} \\leqslant m\n$$\n\n因为 $c_{i}^{\\prime}<c_{i}$, 且 $c_{j}^{\\prime}<m$, 所以\n\n$$\nS_{1}\\left(c_{1}^{\\prime}, c_{2}^{\\prime}, \\cdots, c_{n}^{\\prime}\\right)<S_{1}\\left(c_{1}, c_{2}, \\cdots, c_{n}\\right)\n$$\n\n又 $c_{j}^{\\prime}>c_{j}$ 及 $c_{i}^{\\prime}>0$, 故\n\n$$\nS_{2}\\left(c_{1}^{\\prime}, c_{2}^{\\prime}, \\cdots, c_{n}^{\\prime}\\right) \\leqslant S_{2}\\left(c_{1}, c_{2}, \\cdots, c_{n}\\right)\n$$\n\n如果 $S_{2}>0$, 那么存在 $c_{j}<-m$, 因此有一个 $c_{i}>0$. 令\n\n$$\n\\left(c_{i}^{\\prime}, c_{j}^{\\prime}, c_{k}^{\\prime}\\right)=\\left(c_{i}-a_{j}, c_{j}+a_{i}, c_{k}\\right),\n$$\n\n其中 $1 \\leqslant k \\leqslant n, k \\neq i, j$, 那么\n\n$$\nc_{1}^{\\prime} a_{1}+c_{2}^{\\prime} a_{2}+\\cdots+c_{n}^{\\prime} a_{n}=1\n$$\n\n成立, 并且\n\n$$\n-m<c_{i}^{\\prime}<c_{i}, c_{j}<c_{j}^{\\prime}<0\n$$\n\n与上面类似地可知\n\n$$\nS_{1}\\left(c_{1}^{\\prime}, c_{2}^{\\prime}, \\cdots, c_{n}^{\\prime}\\right) \\leqslant S_{1}\\left(c_{1}, c_{2}, \\cdots, c_{n}\\right)\n$$\n\n且\n\n$$\nS_{2}\\left(c_{1}^{\\prime}, c_{2}^{\\prime}, \\cdots, c_{n}^{\\prime}\\right)<S_{2}\\left(c_{1}, c_{2}, \\cdots, c_{n}\\right)\n$$\n\n因为 $S_{1}, S_{2}$ 均为非负整数, 故通过有限次上述的调整, 可得到一组 $c_{1}, c_{2}, \\cdots, c_{n}$ 使得\n\n$$\nc_{1} a_{1}+c_{2} a_{2}+\\cdots+c_{n} a_{n}=1,\n$$\n\n\n\n并且 $\\left|c_{i}\\right| \\leqslant m, 1 \\leqslant i \\leqslant n$. 且 $S_{1}=S_{2}=0$, 结论一获证.\n\n结论二: 对任意实数 $a, b$, 均有\n\n$$\n\\|a+b\\| \\leqslant\\|a\\|+\\|b\\|\n$$\n\n对任意整数 $u$ 和实数 $y$, 均有\n\n$$\n\\|u y\\| \\leqslant|u| \\cdot\\|y\\| .\n$$\n\n证明: 由于对任意整数 $u$ 和实数 $x$, 有\n\n$$\n\\|x+u\\|=\\|x\\|\n$$\n\n故不妨设 $a, b \\in\\left[-\\frac{1}{2}, \\frac{1}{2}\\right]$, 此时\n\n$$\n\\|a\\|=|a|,\\|b\\|=|b| .\n$$\n\n因为 $||x|| \\leqslant|x|$, 所以\n\n$$\n\\|a+b|| \\leqslant|a+b| \\leqslant|a|+|b|=\\| a\\|+\\| b \\|,\n$$\n\n多次使用该结论有 $|| u y|| \\leqslant|u| \\cdot|| y||$, 于是结论二获证.\n\n回到原题: 由结论一, 存在整数 $c_{1}, c_{2}, \\cdots, c_{n}$, 使得\n\n$$\nc_{1} a_{1}+c_{2} a_{2}+\\cdots+c_{n} a_{n}=1\n$$\n\n并且 $\\left|c_{i}\\right| \\leqslant m, \\quad 1 \\leqslant i \\leqslant n$, 于是\n\n$$\n\\sum_{i=1}^{n} c_{i} a_{i} x=x\n$$\n\n利用结论二得\n\n$$\n\\begin{aligned}\n\\|x\\| & =\\left\\|\\sum_{i=1}^{n} c_{i} a_{i} x\\right\\| \\\\\n& \\leqslant \\sum_{i=1}^{n} \\left( | c_{i} | \\cdot \\| a_{i} x \\| \\right) \\\\\n& \\leqslant m \\sum_{i=1}^{n}\\left\\|a_{i} x\\right\\| \\\\\n& \\leqslant m n \\cdot \\max _{1 \\leqslant i \\leqslant n}\\left\\|a_{i} x\\right\\|,\n\\end{aligned}\n$$\n\n因此\n\n$$\n\\max _{1 \\leqslant i \\leqslant n}\\left\\|a_{i} x\\right\\| \\geqslant \\frac{1}{m n}\\|x\\| .\n$$\n\n\n\n若 $n \\leqslant \\frac{1}{2}(m+1)$, 则\n\n$$\n\\max _{1 \\leqslant i \\leqslant n}\\left\\|a_{i} x\\right\\| \\geqslant \\frac{\\|x\\|}{m n} \\geqslant \\frac{2\\|x\\|}{m(m+1)}\n$$\n\n若 $n>\\frac{1}{2}(m+1)$, 则在 $a_{1}, a_{2}, \\cdots, a_{n}$ 中存在两个相邻正整数, 不妨设 $a_{1}, a_{2}$ 相邻, 则\n\n$$\n\\|x\\|=\\left\\|a_{2} x-a_{1} x\\right\\| \\leqslant\\left\\|a_{2} x\\right\\|+\\left\\|a_{1} x\\right\\|,\n$$\n\n故\n\n$$\n\\max \\left\\{\\left\\|a_{1} x\\right\\|,\\left\\|a_{2} x\\right\\|\\right\\} \\geqslant \\frac{\\|x\\|}{2} \\geqslant \\frac{2\\|x\\|}{m(m+1)} .\n$$\n\n综上所述, 总存在一个 $i(1 \\leqslant i \\leqslant n)$, 满足\n\n$$\n\\left\\|a_{i} x\\right\\| \\geqslant \\frac{2}{m(m+1)}\\|x\\|,\n$$\n\n命题得证.']
null
false
null
null
null
50
Inequality
null
设不等式 $\left|2^{x}-a\right|<\left|5-2^{x}\right|$ 对所有 $x \in[1,2]$ 成立, 求实数 $a$ 的取值范围.
['根据题意, 有\n\n$$\n\\forall x \\in[1,2],\\left|2^{x}-a\\right|<5-2^{x},\n$$\n\n即\n\n$$\n\\forall x \\in[1,2], 2^{x}-5<2^{x}-a<5-2^{x}\n$$\n\n也即\n\n$$\n\\forall x \\in[1,2], 2 \\cdot 2^{x}-5<a<5\n$$\n\n也即\n\n$$\n3<a<5 \\text {. }\n$$']
['$(3,5)$']
false
null
Interval
null
51
Sequence
null
设数列 $\left\{a_{n}\right\}$ 是等差数列, 数列 $\left\{b_{n}\right\}$ 满足 $b_{n}=a_{n+1} a_{n+2}-a_{n}^{2}, n=1,2, \cdots$. 证明: 数列 $\left\{b_{n}\right\}$ 也是等差数列;
['不妨设 $a_{n}=a+n d, n \\in \\mathbb{N}^{*}$, 则\n\n$$\n\\begin{aligned}\nb_{n} & =[a+(n+1) d] \\cdot[a+(n+2) d]-(a+n d)^{2} \\\\\n& =[d n+(a+d)] \\cdot[d n+(a+2 d)]-(d n+a)^{2} \\\\\n& =\\left[d^{2} n^{2}+\\left(2 a d+3 d^{2}\\right) n+\\left(a^{2}+3 a d+2 d^{2}\\right)\\right]-\\left(d^{2} n^{2}+2 a d n+a^{2}\\right) \\\\\n& =3 d^{2} n+\\left(3 a d+2 d^{2}\\right)\n\\end{aligned}\n$$\n\n因此数列 $\\left\\{b_{n}\\right\\}$ 也是等差数列.']
null
false
null
null
null
52
Sequence
null
设数列 $\left\{a_{n}\right\}$ 是等差数列, 数列 $\left\{b_{n}\right\}$ 满足 $b_{n}=a_{n+1} a_{n+2}-a_{n}^{2}, n=1,2, \cdots$. 设数列 $\left\{a_{n}\right\},\left\{b_{n}\right\}$ 的公差均是 $d \neq 0$, 并且存在正整数 $s, t$, 使得 $a_{s}+b_{t}$ 是整数,求 $\left|a_{1}\right|$ 的最小值.
['根据题意, 有 $3 d^{2}=d$, 因此 $d=\\frac{1}{3}$, 于是\n\n$$\nb_{n}=\\frac{1}{3} n+a+\\frac{2}{9}=a_{n}+\\frac{2}{9}\n$$\n\n进而\n\n$$\n\\begin{aligned}\na_{s}+b_{t} & =a_{s}+a_{t}+\\frac{2}{9} \\\\\n& =2 a_{1}+\\frac{s+t-2}{3}+\\frac{2}{9}\n\\end{aligned}\n$$\n\n于是\n\n$$\n18 a_{1}=3\\left[3\\left(a_{s}+b_{t}\\right)-s-t+1\\right]+1,\n$$\n\n因此\n\n$$\n\\left|18 a_{1}\\right| \\geqslant 1\n$$\n\n又当 $\\left(a_{1}, s, t\\right)=\\left(\\frac{1}{18}, 1,3\\right)$ 时, $a_{s}+b_{t}=1$ 符合题意, 因此所求 $\\left|a_{1}\\right|$ 的最小值为 $\\frac{1}{18}$.']
['$\\frac{1}{18}$']
false
null
Numerical
null
53
Plane Geometry
null
在平面直角坐标系 $x O y$ 中, 曲线 $C_{1}: y^{2}=4 x$, 曲线 $C_{2}:(x-4)^{2}+y^{2}=8$. 经过 $C_{1}$上一点 $P$ 作一条倾斜角为 $45^{\circ}$ 的直线 $l$, 与 $C_{2}$ 交于两个不同的点 $Q, R$, 求 $|P Q| \cdot|P R|$的取值范围. ![](https://cdn.mathpix.com/cropped/2023_12_20_d8628742fd3bd519f30cg-1.jpg?height=397&width=346&top_left_y=1509&top_left_x=889)
['设 $P\\left(4 t^{2}, 4 t\\right)$, 则直线\n\n$$\nl: x-y-4 t^{2}+4 t=0\n$$\n\n该直线与圆相交, 因此\n\n$$\n\\frac{\\left|4-4 t^{2}+4 t\\right|}{\\sqrt{2}}<2 \\sqrt{2}\n$$\n\n即\n\n$$\n-1<t^{2}-t-1<1\n$$\n\n\n\n解得 $t$ 的取值范围是 $(-1,0) \\cup(1,2)$.\n\n联立曲线 $C_{1}, C_{2}$ 的方程可得\n\n$$\nx^{2}-4 x+8=0\n$$\n\n于是 $C_{1}$ 与 $C_{2}$ 没有公共点. 因此有\n\n$$\n|P Q| \\cdot|P R|=\\overrightarrow{P Q} \\cdot \\overrightarrow{P R}\n$$\n\n根据圆幂定理, 有\n\n$$\n\\begin{aligned}\n\\overrightarrow{P Q} \\cdot \\overrightarrow{P R} & =P M^{2}-8 \\\\\n& =\\left(4 t^{2}-4\\right)^{2}+(4 t)^{2}-8 \\\\\n& =16 t^{4}-16 t^{2}+8 \\\\\n& =16\\left(t^{2}-\\frac{1}{2}\\right)^{2}+4,\n\\end{aligned}\n$$']
['$[4,8) \\cup(8,200)$']
false
null
Interval
null
53
Plane Geometry
null
在平面直角坐标系 $x O y$ 中, 曲线 $C_{1}: y^{2}=4 x$, 曲线 $C_{2}:(x-4)^{2}+y^{2}=8$. 经过 $C_{1}$上一点 $P$ 作一条倾斜角为 $45^{\circ}$ 的直线 $l$, 与 $C_{2}$ 交于两个不同的点 $Q, R$, 求 $|P Q| \cdot|P R|$的取值范围. <img_4163>
['设 $P\\left(4 t^{2}, 4 t\\right)$, 则直线\n\n$$\nl: x-y-4 t^{2}+4 t=0\n$$\n\n该直线与圆相交, 因此\n\n$$\n\\frac{\\left|4-4 t^{2}+4 t\\right|}{\\sqrt{2}}<2 \\sqrt{2}\n$$\n\n即\n\n$$\n-1<t^{2}-t-1<1\n$$\n\n\n\n解得 $t$ 的取值范围是 $(-1,0) \\cup(1,2)$.\n\n联立曲线 $C_{1}, C_{2}$ 的方程可得\n\n$$\nx^{2}-4 x+8=0\n$$\n\n于是 $C_{1}$ 与 $C_{2}$ 没有公共点. 因此有\n\n$$\n|P Q| \\cdot|P R|=\\overrightarrow{P Q} \\cdot \\overrightarrow{P R}\n$$\n\n根据圆幂定理, 有\n\n$$\n\\begin{aligned}\n\\overrightarrow{P Q} \\cdot \\overrightarrow{P R} & =P M^{2}-8 \\\\\n& =\\left(4 t^{2}-4\\right)^{2}+(4 t)^{2}-8 \\\\\n& =16 t^{4}-16 t^{2}+8 \\\\\n& =16\\left(t^{2}-\\frac{1}{2}\\right)^{2}+4,\n\\end{aligned}\n$$']
['$[4,8) \\cup(8,200)$']
false
null
Interval
null
54
Algebra
null
设实数 $a, b, c$ 满足 $a+b+c=0$, 令 $d=\max \{|a|,|b|,|c|\}$. 证明: $$ |(1+a)(1+b)(1+c)| \geqslant 1-d^{2} . $$
['不妨设 $a \\leqslant b \\leqslant c$, 则 $a \\leqslant 0$ 且 $c \\geqslant 0$. 若 $d \\geqslant 1$, 则不等式显然成立. 下面考虑 $0 \\leqslant d<1$的情形, 此时 $a, b, c \\in(-1,1)$, 于是\n\n$$\nm=|(1+a)(1+b)(1+c)|=(1+a)(1+b)(1+c) .\n$$\n\n当 $b \\geqslant 0$ 时, 有\n\n$$\nm \\geqslant(1+a)(1+b+c)=1-a^{2} \\geqslant 1-d^{2}\n$$\n\n类似的, 当 $b<0$ 时, 有\n\n$$\nm \\geqslant(1+a+b)(1+c)=1-c^{2} \\geqslant 1-d^{2}\n$$\n\n因此原不等式得证.']
null
false
null
null
null
55
Algebra
null
给定正整数 $m$, 证明: 存在正整数 $k$, 使得可将正整数集 $\mathbb{N}^{*}$ 分拆为 $k$ 个互不相交的子集 $A_{1}, A_{2}, \cdots, A_{k}$, 每个子集 $A_{i}$ 中均不存在 4 个数 $a, b, c, d$ (可以相同), 满足 $a b-c d=m$.
['取 $k=m+1$, 令\n\n$$\nA_{i}=\\left\\{x \\mid x \\equiv i \\quad(\\bmod m+1), x \\in \\mathbb{N}^{*}\\right\\},\n$$\n\n其中 $i=1,2, \\cdots, m+1$. 设 $a, b, c, d \\in A_{i}$, 则\n\n$$\na b-c d \\equiv i \\cdot i-i \\cdot i=0 \\equiv 0 \\quad(\\bmod m+1)\n$$\n\n故 $m+1 \\mid a b-c d$, 而 $m+1 \\nmid m$, 所以 $A_{i}$ 中不存在 4 个数 $a, b, c, d$ 满足 $a b-c d=m$.']
null
false
null
null
null
56
Plane Geometry
null
如图, 点 $D$ 是锐角 $\triangle A B C$ 的外接圆 $\omega$ 上弧 $B C$ 的中点, 直线 $D A$ 与圆 $\omega$ 过点 $B, C$的切线分别相交于点 $P, Q, B Q$ 与 $A C$ 的交点为 $X, C P$ 与 $A B$ 的交点为 $Y, B Q$ 与 $C P$ 的交点为 $T$. 求证: $A T$ 平分线段 $X Y$. ![](https://cdn.mathpix.com/cropped/2023_12_20_0bf9557f6b2c57a254f7g-1.jpg?height=380&width=625&top_left_y=347&top_left_x=767)
['首先证明 $Y X \\| B C$, 即证 $\\frac{A X}{X C}=\\frac{A Y}{Y B}$. 连接 $B D, C D$, 如下图.\n\n![](https://cdn.mathpix.com/cropped/2023_12_20_0bf9557f6b2c57a254f7g-1.jpg?height=380&width=619&top_left_y=938&top_left_x=770)\n\n为了方便我们直接用三角形代表该三角形的面积, 因为\n\n$$\n\\frac{\\triangle A C Q}{\\triangle A B C} \\cdot \\frac{\\triangle A B C}{\\triangle A B P}=\\frac{\\triangle A C Q}{\\triangle A B P}\n$$\n\n于是\n\n$$\n\\frac{A C \\cdot C Q \\cdot \\sin \\angle A C Q}{A B \\cdot B C \\cdot \\sin \\angle A B C} \\cdot \\frac{A C \\cdot B C \\cdot \\sin \\angle A C B}{A B \\cdot B P \\cdot \\sin \\angle A B P}=\\frac{A C \\cdot A Q \\cdot \\angle C A Q}{A B \\cdot A P \\cdot \\sin \\angle B A P}\n$$\n\n根据弦切角定理, 有\n\n$$\n\\angle A C Q=\\angle A B C, \\angle A C B=\\angle A B P\n$$\n\n又 $D$ 是弧 $B C$ 的中点, 于是\n\n$$\n\\angle C A Q=\\angle D B C=\\angle D C B=\\angle B A P,\n$$\n\n于是可得\n\n$$\n\\frac{A B \\cdot A Q}{A C \\cdot A P}=\\frac{C Q}{B P}\n$$\n\n因为 $\\angle C A Q=\\angle B A P$, 所以 $\\angle B A Q=\\angle C A P$, 于是\n\n$$\n\\frac{\\triangle A B Q}{\\triangle A C P}=\\frac{A B \\cdot A Q \\cdot \\sin \\angle B A Q}{A C \\cdot A P \\cdot \\sin \\angle C A P}=\\frac{A B \\cdot A Q}{A C \\cdot A P}\n$$\n\n\n\n而\n\n$$\n\\frac{\\triangle B C Q}{\\triangle B C P}=\\frac{B C \\cdot C Q \\cdot \\sin \\angle B C Q}{B C \\cdot B P \\cdot \\sin \\angle C B P}=\\frac{C Q}{B P}\n$$\n\n因此可得\n\n$$\n\\frac{\\triangle A B Q}{\\triangle A C P}=\\frac{\\triangle C B Q}{\\triangle B C P}\n$$\n\n即\n\n$$\n\\frac{\\triangle A B Q}{\\triangle C B Q}=\\frac{\\triangle A C P}{\\triangle B C P}\n$$\n\n又\n\n$$\n\\frac{\\triangle A B Q}{\\triangle C B Q}=\\frac{A X}{X C}, \\frac{\\triangle A C P}{\\triangle B C P}=\\frac{A Y}{Y B}\n$$\n\n故 $\\frac{A X}{X C}=\\frac{A Y}{Y B}$.\n\n设 $B C$ 边的中点为 $M$, 因为\n\n$$\n\\frac{A X}{X C} \\cdot \\frac{C M}{M B} \\cdot \\frac{B Y}{Y A}=1\n$$\n\n所以由塞瓦定理的逆定理可知 $A M, B X, C Y$ 三线共点于 $T$, 故由 $Y X \\| B C$ 可知 $A T$平分线段 $X Y$.']
['证明题']
false
null
Need_human_evaluate
null
56
Plane Geometry
null
如图, 点 $D$ 是锐角 $\triangle A B C$ 的外接圆 $\omega$ 上弧 $B C$ 的中点, 直线 $D A$ 与圆 $\omega$ 过点 $B, C$的切线分别相交于点 $P, Q, B Q$ 与 $A C$ 的交点为 $X, C P$ 与 $A B$ 的交点为 $Y, B Q$ 与 $C P$ 的交点为 $T$. 求证: $A T$ 平分线段 $X Y$. <img_4068>
['首先证明 $Y X \\| B C$, 即证 $\\frac{A X}{X C}=\\frac{A Y}{Y B}$. 连接 $B D, C D$, 如下图.\n\n<img_4225>\n\n为了方便我们直接用三角形代表该三角形的面积, 因为\n\n$$\n\\frac{\\triangle A C Q}{\\triangle A B C} \\cdot \\frac{\\triangle A B C}{\\triangle A B P}=\\frac{\\triangle A C Q}{\\triangle A B P}\n$$\n\n于是\n\n$$\n\\frac{A C \\cdot C Q \\cdot \\sin \\angle A C Q}{A B \\cdot B C \\cdot \\sin \\angle A B C} \\cdot \\frac{A C \\cdot B C \\cdot \\sin \\angle A C B}{A B \\cdot B P \\cdot \\sin \\angle A B P}=\\frac{A C \\cdot A Q \\cdot \\angle C A Q}{A B \\cdot A P \\cdot \\sin \\angle B A P}\n$$\n\n根据弦切角定理, 有\n\n$$\n\\angle A C Q=\\angle A B C, \\angle A C B=\\angle A B P\n$$\n\n又 $D$ 是弧 $B C$ 的中点, 于是\n\n$$\n\\angle C A Q=\\angle D B C=\\angle D C B=\\angle B A P,\n$$\n\n于是可得\n\n$$\n\\frac{A B \\cdot A Q}{A C \\cdot A P}=\\frac{C Q}{B P}\n$$\n\n因为 $\\angle C A Q=\\angle B A P$, 所以 $\\angle B A Q=\\angle C A P$, 于是\n\n$$\n\\frac{\\triangle A B Q}{\\triangle A C P}=\\frac{A B \\cdot A Q \\cdot \\sin \\angle B A Q}{A C \\cdot A P \\cdot \\sin \\angle C A P}=\\frac{A B \\cdot A Q}{A C \\cdot A P}\n$$\n\n\n\n而\n\n$$\n\\frac{\\triangle B C Q}{\\triangle B C P}=\\frac{B C \\cdot C Q \\cdot \\sin \\angle B C Q}{B C \\cdot B P \\cdot \\sin \\angle C B P}=\\frac{C Q}{B P}\n$$\n\n因此可得\n\n$$\n\\frac{\\triangle A B Q}{\\triangle A C P}=\\frac{\\triangle C B Q}{\\triangle B C P}\n$$\n\n即\n\n$$\n\\frac{\\triangle A B Q}{\\triangle C B Q}=\\frac{\\triangle A C P}{\\triangle B C P}\n$$\n\n又\n\n$$\n\\frac{\\triangle A B Q}{\\triangle C B Q}=\\frac{A X}{X C}, \\frac{\\triangle A C P}{\\triangle B C P}=\\frac{A Y}{Y B}\n$$\n\n故 $\\frac{A X}{X C}=\\frac{A Y}{Y B}$.\n\n设 $B C$ 边的中点为 $M$, 因为\n\n$$\n\\frac{A X}{X C} \\cdot \\frac{C M}{M B} \\cdot \\frac{B Y}{Y A}=1\n$$\n\n所以由塞瓦定理的逆定理可知 $A M, B X, C Y$ 三线共点于 $T$, 故由 $Y X \\| B C$ 可知 $A T$平分线段 $X Y$.']
null
false
null
null
null
57
Algebra
null
设 $a_{1}, a_{2}, \cdots, a_{20} \in\{1,2, \cdots, 5\}, b_{1}, b_{2}, \cdots, b_{20} \in\{1,2, \cdots, 10\}$, 集合 $X=\{(i, j) \mid$ $\left.1 \leqslant i<j \leqslant 20,\left(a_{i}-a_{j}\right)\left(b_{i}-b_{j}\right)<0\right\}$, 求 $X$ 的元素个数的最大值.
['考虑一组满足条件的正整数\n\n$$\n\\left\\{a_{1}, a_{2}, \\cdots, a_{20}, b_{1}, b_{2}, \\cdots, b_{20}\\right\\}\n$$\n\n对 $k=1,2, \\cdots, 5$, 设 $a_{1}, a_{2}, \\cdots, a_{20}$ 中取值为 $k$ 的数有 $t_{k}$ 个. 根据 $X$ 的定义, 当 $a_{i}=a_{j}$ 时, $(i, j) \\notin X$, 因此至少有 $\\sum_{k=1}^{5} \\mathrm{C}_{t_{k}}^{2}$ 个 $(i, j)$ 不在 $X$ 中. 注意到\n\n$$\n\\sum_{k=1}^{5} t_{k}=20\n$$\n\n由柯西不等式, 有\n\n$$\n\\sum_{k=1}^{5} \\mathrm{C}_{t_{k}}^{2}=\\frac{1}{2}\\left(\\sum_{k=1}^{5} t_{k}^{2}-\\sum_{k=1}^{5} t_{k}\\right) \\geqslant \\frac{1}{2}\\left(\\frac{1}{5}\\left(\\sum_{k=1}^{5} t_{k}\\right)^{2}-\\sum_{k=1}^{5} t_{k}\\right)=30\n$$\n\n从而 $X$ 的元素个数不超过\n\n$$\n\\mathrm{C}_{20}^{2}-30=160\n$$\n\n另一方面, 取\n\n$$\na_{4 k-3}=a_{4 k-2}=a_{4 k-1}=a_{4 k}=k\n$$\n\n其中 $k=1,2, \\cdots, 5$ ,且\n\n$$\nb_{i}=6-a_{i},\n$$\n\n其中 $i=1,2, \\cdots, 20$, 则对任意 $i, j(1 \\leqslant i<j \\leqslant 20)$, 有\n\n$$\n\\left(a_{i}-a_{j}\\right)\\left(b_{i}-b_{j}\\right)=\\left(a_{i}-a_{j}\\right)\\left[\\left(6-a_{i}\\right)-\\left(6-a_{j}\\right)\\right]=-\\left(a_{i}-a_{j}\\right)^{2} \\leqslant 0\n$$\n\n等号成立当且仅当 $a_{i}=a_{j}$, 这恰好发生 $5 \\mathrm{C}_{4}^{2}=30$ 次. 此时 $X$ 的元素个数取得 160 .\n\n综上所述, $X$ 的元素个数的最大值为 160 .']
['$160$']
false
null
Numerical
null
58
Trigonometric Functions
null
在 $\triangle A B C$ 中, 已知 $\overrightarrow{A B} \cdot \overrightarrow{A C}+2 \overrightarrow{B A} \cdot \overrightarrow{B C}=3 \overrightarrow{C A} \cdot \overrightarrow{C B}$. 求 $\sin C$ 的最大值.
['统一起点, 有\n\n$$\n(\\overrightarrow{C A}-\\overrightarrow{C B}) \\cdot \\overrightarrow{C A}+2(\\overrightarrow{C B}-\\overrightarrow{C A}) \\cdot \\overrightarrow{C B}=3 \\overrightarrow{C A} \\cdot \\overrightarrow{C B}\n$$\n\n即\n\n$$\n6 \\overrightarrow{C A} \\cdot \\overrightarrow{C B}=C A^{2}+2 C B^{2}\n$$\n\n也即\n\n$$\n\\cos C=\\frac{C A^{2}+2 C B^{2}}{6 C A \\cdot C B} \\geqslant \\frac{\\sqrt{2}}{3}\n$$\n\n等号当 $C A=\\sqrt{2} \\cdot C B$ 时取得. 因此 $\\cos C$ 的最小值为 $\\frac{\\sqrt{2}}{3}$, 对应 $\\sin C$ 的最大值为 $\\frac{\\sqrt{7}}{3}$.']
['$\\frac{\\sqrt{7}}{3}$']
false
null
Numerical
null
59
Elementary Functions
null
已知 $f(x)$ 是 $\mathbb{R}$ 上的奇函数, $f(1)=1$, 且对任意 $x<0$, 均有 $f\left(\frac{x}{x-1}\right)=x f(x)$. 求 $$ f(1) f\left(\frac{1}{100}\right)+f\left(\frac{1}{2}\right) f\left(\frac{1}{99}\right)+f\left(\frac{1}{3}\right) f\left(\frac{1}{98}\right)+\cdots+f\left(\frac{1}{50}\right) f\left(\frac{1}{51}\right) $$ 的值.
['令 $x=-\\frac{1}{n}, n \\in \\mathbb{N}^{*}$, 则\n\n$$\n\\frac{x}{x-1}=\\frac{-\\frac{1}{n}}{-\\frac{1}{n}-1}=\\frac{1}{n+1}\n$$\n\n\n\n于是有\n\n$$\nf\\left(\\frac{1}{n+1}\\right)=-\\frac{1}{n} f\\left(-\\frac{1}{n}\\right)=\\frac{1}{n} f\\left(\\frac{1}{n}\\right)\n$$\n\n记 $a_{n}=f\\left(\\frac{1}{n}\\right), n \\in \\mathbb{N}^{*}$, 则有 $\\frac{a_{n+1}}{a_{n}}=\\frac{1}{n}$, 从而\n\n$$\na_{n}=\\frac{1}{(n-1) !}.\n$$\n\n于是所求代数式即\n\n$$\n\\sum_{i=1}^{50} a_{i} a_{101-i}=\\sum_{i=1}^{50} \\frac{1}{(i-1) ! \\cdot(100-i) !}=\\frac{1}{99 !} \\sum_{i=0}^{49} \\mathrm{C}_{99}^{i}=\\frac{1}{99 !} \\cdot \\frac{1}{2} \\sum_{i=0}^{99} \\mathrm{C}_{99}^{i}=\\frac{2^{98}}{99 !}\n$$']
['$\\frac{2^{98}}{99 !}$']
false
null
Numerical
null
60
Plane Geometry
null
如图所示, 在平面直角坐标系 $x O y$ 中, $F$ 是 $x$ 轴正半轴上的一个动点, 以 $F$ 为焦点、 $O$ 为顶点作抛物线 $C$, 设 $P$ 是第一象限内 $C$ 上的一点, $Q$ 是 $x$ 轴负半轴上一点, 使得 $P Q$ 为 $C$ 的切线, 且 $|P Q|=2$, 圆 $C_{1}, C_{2}$ 均与直线 $O P$ 相切于点 $P$, 且均与 $x$ 轴相切, 求点 $F$ 的坐标, 使圆 $C_{1}$ 与 $C_{2}$ 的面积之和取到最小值. ![](https://cdn.mathpix.com/cropped/2023_12_20_b2a6aceba9e8932d0f8ag-1.jpg?height=548&width=622&top_left_y=1234&top_left_x=754)
['如下图, 设抛物线方程为 $C: y^{2}=2 p x$, 焦点 $F\\left(\\frac{p}{2}, 0\\right)$, 连接 $C_{1} C_{2}, P F$.\n\n\n\n![](https://cdn.mathpix.com/cropped/2023_12_20_8f010abcf998421dbc51g-1.jpg?height=563&width=636&top_left_y=347&top_left_x=747)\n\n设 $P\\left(2 p a^{2}, 2 p a\\right)$, 则根据抛物线的光学性质, $|F Q|=|P F|$, 于是 $Q\\left(-2 p a^{2}, 0\\right)$, 进而由 $|P Q|=2$, 可得\n\n$$\n4 p^{2} a^{4}+p^{2} a^{2}=1,\n$$\n\n即\n\n$$\np^{2} a^{2}=\\frac{1}{4 a^{2}+1}\n$$\n\n圆心 $C_{1}, C_{2}$ 都在过点 $P$ 且与 $O P$ 垂直的直线 $l$ 上, 设直线 $l$ 的参数方程为\n\n$$\n\\left\\{\\begin{array}{l}\nx=2 p a^{2}+t \\\\\ny=2 p a-a t\n\\end{array}\\right.\n$$\n\n根据题意, $C_{1}, C_{2}$ 对应的参数满足\n\n$$\n\\sqrt{1+a^{2}} \\cdot|t|=2 p a-a t\n$$\n\n即\n\n$$\nt^{2}+4 p a^{2} t-4 p^{2} a^{2}=0\n$$\n\n因此圆 $C_{1}$ 的面积 $S_{1}$ 与圆 $C_{2}$ 的面积 $S_{2}$ 之和\n\n$$\n\\begin{aligned}\nS_{1}+S_{2} & =\\pi\\left[\\left(1+a^{2}\\right) t_{1}^{2}+\\left(1+a^{2}\\right) t_{2}^{2}\\right] \\\\\n& =\\pi\\left(1+a^{2}\\right)\\left[\\left(-4 p a^{2}\\right)^{2}+2 \\cdot 4 p^{2} a^{2}\\right] \\\\\n& =\\pi\\left(1+a^{2}\\right)\\left(16 a^{2}+8\\right) \\cdot p^{2} a^{2} \\\\\n& =\\frac{\\pi\\left(16 a^{4}+24 a^{2}+8\\right)}{4 a^{2}+1} \\\\\n& =\\pi\\left(4 a^{2}+1+\\frac{3}{4 a^{2}+1}+4\\right) \\\\\n& \\geqslant \\pi(4+2 \\sqrt{3})\n\\end{aligned}\n$$\n\n\n\n等号当 $4 a^{2}+1=\\sqrt{3}$, 即 $a^{2}=\\frac{\\sqrt{3}-1}{4}$ 时取得. 此时\n\n$$\np^{2}=\\frac{1}{a^{2}\\left(4 a^{2}+1\\right)}=\\frac{4}{(\\sqrt{3}-1) \\cdot \\sqrt{3}}\n$$\n\n于是 $F$ 点的坐标为 $\\left(\\frac{1}{\\sqrt{3-\\sqrt{3}}}, 0\\right)$.']
['$\\left(\\frac{1}{\\sqrt{3-\\sqrt{3}}}, 0\\right)$']
false
null
Tuple
null
60
Plane Geometry
null
如图所示, 在平面直角坐标系 $x O y$ 中, $F$ 是 $x$ 轴正半轴上的一个动点, 以 $F$ 为焦点、 $O$ 为顶点作抛物线 $C$, 设 $P$ 是第一象限内 $C$ 上的一点, $Q$ 是 $x$ 轴负半轴上一点, 使得 $P Q$ 为 $C$ 的切线, 且 $|P Q|=2$, 圆 $C_{1}, C_{2}$ 均与直线 $O P$ 相切于点 $P$, 且均与 $x$ 轴相切, 求点 $F$ 的坐标, 使圆 $C_{1}$ 与 $C_{2}$ 的面积之和取到最小值. <img_4130>
['如下图, 设抛物线方程为 $C: y^{2}=2 p x$, 焦点 $F\\left(\\frac{p}{2}, 0\\right)$, 连接 $C_{1} C_{2}, P F$.\n\n\n\n<img_4131>\n\n设 $P\\left(2 p a^{2}, 2 p a\\right)$, 则根据抛物线的光学性质, $|F Q|=|P F|$, 于是 $Q\\left(-2 p a^{2}, 0\\right)$, 进而由 $|P Q|=2$, 可得\n\n$$\n4 p^{2} a^{4}+p^{2} a^{2}=1,\n$$\n\n即\n\n$$\np^{2} a^{2}=\\frac{1}{4 a^{2}+1}\n$$\n\n圆心 $C_{1}, C_{2}$ 都在过点 $P$ 且与 $O P$ 垂直的直线 $l$ 上, 设直线 $l$ 的参数方程为\n\n$$\n\\left\\{\\begin{array}{l}\nx=2 p a^{2}+t \\\\\ny=2 p a-a t\n\\end{array}\\right.\n$$\n\n根据题意, $C_{1}, C_{2}$ 对应的参数满足\n\n$$\n\\sqrt{1+a^{2}} \\cdot|t|=2 p a-a t\n$$\n\n即\n\n$$\nt^{2}+4 p a^{2} t-4 p^{2} a^{2}=0\n$$\n\n因此圆 $C_{1}$ 的面积 $S_{1}$ 与圆 $C_{2}$ 的面积 $S_{2}$ 之和\n\n$$\n\\begin{aligned}\nS_{1}+S_{2} & =\\pi\\left[\\left(1+a^{2}\\right) t_{1}^{2}+\\left(1+a^{2}\\right) t_{2}^{2}\\right] \\\\\n& =\\pi\\left(1+a^{2}\\right)\\left[\\left(-4 p a^{2}\\right)^{2}+2 \\cdot 4 p^{2} a^{2}\\right] \\\\\n& =\\pi\\left(1+a^{2}\\right)\\left(16 a^{2}+8\\right) \\cdot p^{2} a^{2} \\\\\n& =\\frac{\\pi\\left(16 a^{4}+24 a^{2}+8\\right)}{4 a^{2}+1} \\\\\n& =\\pi\\left(4 a^{2}+1+\\frac{3}{4 a^{2}+1}+4\\right) \\\\\n& \\geqslant \\pi(4+2 \\sqrt{3})\n\\end{aligned}\n$$\n\n\n\n等号当 $4 a^{2}+1=\\sqrt{3}$, 即 $a^{2}=\\frac{\\sqrt{3}-1}{4}$ 时取得. 此时\n\n$$\np^{2}=\\frac{1}{a^{2}\\left(4 a^{2}+1\\right)}=\\frac{4}{(\\sqrt{3}-1) \\cdot \\sqrt{3}}\n$$\n\n于是 $F$ 点的坐标为 $\\left(\\frac{1}{\\sqrt{3-\\sqrt{3}}}, 0\\right)$.']
['$\\left(\\frac{1}{\\sqrt{3-\\sqrt{3}}}, 0\\right)$']
false
null
Tuple
null
61
Algebra
null
设实数 $a_{1}, a_{2}, \cdots, a_{2016}$ 满足 $9 a_{i}>11 a_{i+1}^{2}(i=1,2, \cdots, 2015)$. 求 $$ \left(a_{1}-a_{2}^{2}\right) \cdot\left(a_{2}-a_{3}^{2}\right) \cdots\left(a_{2015}-a_{2016}^{2}\right) \cdot\left(a_{2016}-a_{1}^{2}\right) $$ 的最大值.
['令原式为 $P$. 由于 $a_{i}-a_{i+1}^{2}>0, i=1,2, \\cdots, 2015$, 因此只需要考虑当 $a_{2016}-a_{1}^{2}>0$的情况, 记 $a_{2017}=a_{1}$, 则\n\n$$\n\\begin{aligned}\nP^{\\frac{1}{2016}} & \\leqslant \\frac{1}{2016} \\sum_{k=1}^{2016}\\left(a_{k}-a_{k+1}^{2}\\right) \\\\\n& =\\frac{1}{2016}\\left(\\sum_{k=1}^{2016} a_{k}-\\sum_{k=1}^{2016} a_{k}^{2}\\right) \\\\\n& =\\frac{1}{2016} \\sum_{k=1}^{2016}\\left[a_{k}\\left(1-a_{k}\\right)\\right] \\leqslant \\frac{1}{4},\n\\end{aligned}\n$$\n\n等号当 $a_{1}=a_{2}=\\cdots=a_{2016}=\\frac{1}{2}$ 时取得. 因此 $P$ 的最大值为 $\\frac{1}{4^{2016}}$.']
['$\\frac{1}{4^{2016}}$']
false
null
Numerical
null
62
Plane Geometry
null
如图所示, 在 $\triangle A B C$ 中, $X, Y$ 是直线 $B C$ 上两点 ( $X, B, C, Y$ 顺次排列), 使得 $$ B X \cdot A C=C Y \cdot A B $$ 设 $\triangle A C X, \triangle A B Y$ 的外心分别为 $O_{1}, O_{2}$, 直线 $O_{1} O_{2}$ 与 $A B, A C$ 分别交于点 $U, V$. 证明: $\triangle A U V$ 是等腰三角形. ![](https://cdn.mathpix.com/cropped/2023_12_20_6b053daa81fcef60274eg-1.jpg?height=306&width=545&top_left_y=1880&top_left_x=801)
['如下图, 设圆 $O_{1}$ 与圆 $O_{2}$ 的公共弦为 $A D, A D$ 交 $X Y$ 于 $E$.\n\n\n\n![](https://cdn.mathpix.com/cropped/2023_12_20_e3d7b934740f0b7776d0g-1.jpg?height=448&width=559&top_left_y=336&top_left_x=794)\n\n由于 $A D$ 为两圆的根轴, 于是 $E$ 点对圆 $O_{1}$ 和圆 $O_{2}$ 的幂相等, 从而\n\n$$\nX E \\cdot C E=B E \\cdot Y E\n$$\n\n进而结合合分比定理有\n\n$$\n\\frac{B E}{C E}=\\frac{X E}{Y E}=\\frac{X B}{Y C}\n$$\n\n又由已知, 有 $\\frac{X B}{Y C}=\\frac{A B}{A C}$, 于是有 $\\frac{B E}{C E}=\\frac{A B}{A C}$, 从而 $A E$ 是 $\\angle B A C$ 的角平分线. 又 $A D \\perp O_{1} O_{2}$, 于是 $U, V$ 关于直线 $A D$ 对称, 因此 $\\triangle A U V$ 是等腰三角形.', '如图, 设 $\\triangle A B C$ 的外心为 $O$, 连结 $O O_{1}, O O_{2}$, 过点 $O, O_{1}, O_{2}$ 分别作直线 $B C$ 的垂线, 垂足分别为 $D, D_{1}, D_{2}$. 作 $O_{1} K \\perp O D$ 于点 $K$.\n\n![](https://cdn.mathpix.com/cropped/2023_12_20_e3d7b934740f0b7776d0g-1.jpg?height=271&width=502&top_left_y=1441&top_left_x=820)\n\n我们证明 $O O_{1}=O O_{2}$. 在直角三角形 $O K O_{1}$ 中,\n\n$$\nO O_{1}=\\frac{O_{1} K}{\\sin \\angle O_{1} O K}\n$$\n\n由外心的性质, $O O_{1} \\perp A C$, 又 $O D \\perp B C$, 故 $\\angle O_{1} O K=\\angle A C B$. 而 $D, D_{1}$ 分别是 $B C, C X$ 的中点, 所以\n\n$$\nD D_{1}=C D_{1}-C D=\\frac{1}{2} C X-\\frac{1}{2} B C=\\frac{1}{2} B X\n$$\n\n因此\n\n$$\nO O_{1}=\\frac{O_{1} K}{\\sin \\angle O_{1} O K}=\\frac{D D_{1}}{\\sin \\angle A C B}=\\frac{\\frac{1}{2} B X}{\\frac{A B}{2 R}}=R \\cdot \\frac{B X}{A B}\n$$\n\n\n\n这里 $R$ 是 $\\triangle A B C$ 的外接圆半径. 同理 $O O_{2}=R \\cdot \\frac{C Y}{A C}$. 由已知条件可得 $\\frac{B X}{A B}=\\frac{C Y}{A C}$,故 $O O_{1}=O O_{2}$. 由于 $O O_{1}=O O_{2}$, 故 $\\angle O O_{1} O_{2}=\\angle O O_{2} O_{1}$, 从而 $\\angle A U V=\\angle A V U$.这样 $A U=A V$, 即 $\\triangle A U V$ 是等腰三角形.']
['证明题']
false
null
Need_human_evaluate
null
62
Plane Geometry
null
如图所示, 在 $\triangle A B C$ 中, $X, Y$ 是直线 $B C$ 上两点 ( $X, B, C, Y$ 顺次排列), 使得 $$ B X \cdot A C=C Y \cdot A B $$ 设 $\triangle A C X, \triangle A B Y$ 的外心分别为 $O_{1}, O_{2}$, 直线 $O_{1} O_{2}$ 与 $A B, A C$ 分别交于点 $U, V$. 证明: $\triangle A U V$ 是等腰三角形. <img_4193>
['如下图, 设圆 $O_{1}$ 与圆 $O_{2}$ 的公共弦为 $A D, A D$ 交 $X Y$ 于 $E$.\n\n\n\n<img_4145>\n\n由于 $A D$ 为两圆的根轴, 于是 $E$ 点对圆 $O_{1}$ 和圆 $O_{2}$ 的幂相等, 从而\n\n$$\nX E \\cdot C E=B E \\cdot Y E\n$$\n\n进而结合合分比定理有\n\n$$\n\\frac{B E}{C E}=\\frac{X E}{Y E}=\\frac{X B}{Y C}\n$$\n\n又由已知, 有 $\\frac{X B}{Y C}=\\frac{A B}{A C}$, 于是有 $\\frac{B E}{C E}=\\frac{A B}{A C}$, 从而 $A E$ 是 $\\angle B A C$ 的角平分线. 又 $A D \\perp O_{1} O_{2}$, 于是 $U, V$ 关于直线 $A D$ 对称, 因此 $\\triangle A U V$ 是等腰三角形.', '如图, 设 $\\triangle A B C$ 的外心为 $O$, 连结 $O O_{1}, O O_{2}$, 过点 $O, O_{1}, O_{2}$ 分别作直线 $B C$ 的垂线, 垂足分别为 $D, D_{1}, D_{2}$. 作 $O_{1} K \\perp O D$ 于点 $K$.\n\n<img_4154>\n\n我们证明 $O O_{1}=O O_{2}$. 在直角三角形 $O K O_{1}$ 中,\n\n$$\nO O_{1}=\\frac{O_{1} K}{\\sin \\angle O_{1} O K}\n$$\n\n由外心的性质, $O O_{1} \\perp A C$, 又 $O D \\perp B C$, 故 $\\angle O_{1} O K=\\angle A C B$. 而 $D, D_{1}$ 分别是 $B C, C X$ 的中点, 所以\n\n$$\nD D_{1}=C D_{1}-C D=\\frac{1}{2} C X-\\frac{1}{2} B C=\\frac{1}{2} B X\n$$\n\n因此\n\n$$\nO O_{1}=\\frac{O_{1} K}{\\sin \\angle O_{1} O K}=\\frac{D D_{1}}{\\sin \\angle A C B}=\\frac{\\frac{1}{2} B X}{\\frac{A B}{2 R}}=R \\cdot \\frac{B X}{A B}\n$$\n\n\n\n这里 $R$ 是 $\\triangle A B C$ 的外接圆半径. 同理 $O O_{2}=R \\cdot \\frac{C Y}{A C}$. 由已知条件可得 $\\frac{B X}{A B}=\\frac{C Y}{A C}$,故 $O O_{1}=O O_{2}$. 由于 $O O_{1}=O O_{2}$, 故 $\\angle O O_{1} O_{2}=\\angle O O_{2} O_{1}$, 从而 $\\angle A U V=\\angle A V U$.这样 $A U=A V$, 即 $\\triangle A U V$ 是等腰三角形.']
null
false
null
null
null
63
Combinatorics
null
给定空间中 10 个点, 其中任意四点不在一个平面上, 将某些点之间用线段相连, 若得到的图形中没有三角形也没有空间四边形, 试确定所连线段数目的最大值.
['记这 10 个点分别为 $P_{i}$ 且从 $P_{i}$ 点引出了 $a_{i}$ 条线段, 其中 $i=1,2, \\cdots, 10$. 这样图形中总共包含 $\\frac{1}{2} \\sum_{i=1}^{10} a_{i}$ 条线段和 $\\sum_{i=1}^{10} \\mathrm{C}_{a_{i}}^{2}$ 个角. 根据题意, 图形中没有空间四边形, 因此任何一个角都与一个点对 $\\left(P_{m}, P_{n}\\right)$ 一一对应, 且不存在线段 $P_{m} P_{n}$. 这样就有\n\n$$\n\\frac{1}{2} \\sum_{i=1}^{10} a_{i}+\\sum_{i=1}^{10} \\mathrm{C}_{a_{i}}^{2}=\\frac{1}{2} \\sum_{i=1}^{10} a_{i}^{2} \\leqslant \\mathrm{C}_{10}^{2}\n$$\n\n于是所连线段数目\n\n$$\n\\frac{1}{2} \\sum_{i=1}^{10} a_{i} \\leqslant \\frac{1}{2} \\sqrt{10 \\cdot \\sum_{i=1}^{10} a_{i}^{2}}=15\n$$\n\n接下来构造包含 15 条线段的图形, 此时从每个顶点出发的线段数均为 3 , 如下图.\n\n<img_4202>']
['$15$']
false
null
Numerical
null
64
Sequence
null
设 $p$ 与 $p+2$ 均是素数, $p>3$. 数列 $\left\{a_{n}\right\}$ 定义为 $$ a_{1}=2, a_{n}=a_{n-1}+\left\lceil\frac{p a_{n-1}}{n}\right\rceil, n=2,3, \cdots $$ 这里 $\lceil x\rceil$ 表示不小于实数 $x$ 的最小整数. 证明: 对 $n=3,4, \cdots, p-1$ 均有 $n \mid p a_{n-1}+1$成立.
['首先注意, $\\left\\{a_{n}\\right\\}$ 是整数数列.\n\n对 $n$ 用数学归纳法. 当 $n=3$ 时, 由条件知 $a_{2}=2+p$, 故 $p a_{2}+1=(p+1)^{2}$. 因 $p$ 与$p+2$ 均是素数, 且 $p>3$, 故必须 $3 \\mid p+1$. 因此 $3 \\mid p a_{2}+1$, 即 $n=3$ 时结论成立.对 $3<n \\leqslant p-1$, 设对 $k=3, \\cdots, n-1$ 成立, 此时 $\\left\\lceil\\frac{p a_{k-1}}{k}\\right\\rceil=\\frac{p a_{k-1}+1}{k}$, 故\n\n$$\n\\begin{aligned}\np a_{k-1}+1 & =p\\left(a_{k-2}+\\left\\lceil\\frac{p a_{k-2}}{k-1}\\right\\rceil\\right)+1 \\\\\n& =p\\left(a_{k-2}+\\frac{p a_{k-2}+1}{k-1}\\right)+1 \\\\\n& =\\frac{\\left(p a_{k-2}+1\\right)(p+k-1)}{k-1} .\n\\end{aligned}\n$$\n\n故对 $3<n \\leqslant p-1$, 有\n\n$$\n\\begin{aligned}\np a_{n-1}+1 & =\\frac{p+n-1}{n-1}\\left(p a_{n-2}+1\\right) \\\\\n& =\\frac{p+n-1}{n-1} \\cdot \\frac{p+n-2}{n-2}\\left(p a_{n-3}+1\\right) \\\\\n& =\\cdots \\\\\n& =\\frac{p+n-1}{n-1} \\cdot \\frac{p+n-2}{n-2} \\cdots \\frac{p+3}{3}\\left(p a_{2}+1\\right),\n\\end{aligned}\n$$\n\n因此\n\n$$\np a_{n-1}+1=\\frac{2 n(p+1)}{(p+n)(p+2)} \\mathrm{C}_{p+n}^{n}\n$$\n\n由此知(注意 $\\mathrm{C}_{p+n}^{n}$ 是整数)\n\n$$\nn \\mid(p+n)(p+2)\\left(p a_{n-1}+1\\right)\n$$\n\n因 $n<p, p$ 为素数, 故 $(n, n+p)=(n, p)=1$, 又 $p+2$ 是大于 $n$ 的素数, 故 $(n, p+2)=1$,从而 $n$ 与 $(p+n)(p+2)$ 互素, 故 $n \\mid p a_{n-1}+1$. 由数学归纳法知, 本题得证.']
null
false
null
null
null
65
Algebra
null
若实数 $a, b, c$ 满足 $2^{a}+4^{b}=2^{c}, 4^{a}+2^{b}=4^{c}$, 求 $c$ 的最小值.
['记 $x=2^{a}, y=2^{b}, z=2^{c}$, 则\n\n$$\n\\left\\{\\begin{array}{l}\nx+y^{2}=z \\\\\nx^{2}+y=z^{2}\n\\end{array}\\right.\n$$\n\n消元得\n\n$$\n\\left(z-y^{2}\\right)^{2}+y=z^{2}\n$$\n\n整理得\n\n$$\nz=\\frac{y^{2}}{2}+\\frac{1}{2 y}=\\frac{y^{2}}{2}+\\frac{1}{4 y}+\\frac{1}{4 y} \\geqslant 3 \\sqrt[3]{\\frac{y^{2}}{2} \\cdot \\frac{1}{4 y} \\cdot \\frac{1}{4 y}}=\\frac{3}{4} \\sqrt[3]{2}\n$$\n\n等号当 $y=\\frac{1}{\\sqrt[3]{2}}, x=\\frac{\\sqrt[3]{2}}{4}$ 时取得. 于是 $z$ 的最小值为 $\\frac{3}{4} \\sqrt[3]{2}$, 进而 $c$ 的最小值为\n\n$$\n\\log _{2}\\left(\\frac{3}{4} \\sqrt[3]{2}\\right)=\\log _{2} 3-\\frac{5}{3}\n$$']
['$\\log _{2} 3-\\frac{5}{3}$']
false
null
Numerical
null
66
Algebra
null
设 $a_{1}, a_{2}, a_{3}, a_{4}$ 是 4 个有理数, 使得 $$ \left\{a_{i} a_{j} \mid 1 \leqslant i<j \leqslant 4\right\}=\left\{-24,-2,-\frac{3}{2},-\frac{1}{8}, 1,3\right\}, $$ 求 $a_{1}+a_{2}+a_{3}+a_{4}$ 的值.
['设 $a_{1}, a_{2}, a_{3}, a_{4}$ 的绝对值从小到大排列, 则\n\n$$\n\\left\\{\\begin{array}{l}\na_{1} a_{2}=-\\frac{1}{8}, \\\\\na_{1} a_{3}=1, \\\\\na_{2} a_{4}=3, \\\\\na_{3} a_{4}=-24, \\\\\n\\left\\{a_{2} a_{3}, a_{1} a_{4}\\right\\}=\\left\\{-2,-\\frac{3}{2}\\right\\},\n\\end{array}\\right.\n$$\n\n\n\n解得\n\n$$\n\\left(a_{1}, a_{2}, a_{3}, a_{4}\\right)=\\left(\\frac{1}{4},-\\frac{1}{2}, 4,-6\\right) \\vee\\left(-\\frac{1}{4}, \\frac{1}{2},-4,6\\right)\n$$\n\n于是 $a_{1}+a_{2}+a_{3}+a_{4}= \\pm \\frac{9}{4}$.']
['$ -\\frac{9}{4}, \\frac{9}{4}$']
true
null
Numerical
null
67
Plane Geometry
null
在平面直角坐标系 $x O y$ 中, $F_{1}, F_{2}$ 分别是椭圆 $\frac{x^{2}}{2}+y^{2}=1$ 的左、右焦点, 设不经过焦点 $F_{1}$ 的直线 $l$ 与椭圆交于两个不同的点 $A, B$, 焦点 $F_{2}$ 到直线 $l$ 的距离为 $d$. 如果直线 $A F_{1}, l, B F_{1}$ 的斜率依次成等差数列, 求 $d$ 的取值范围.
['根据已知, 有 $F_{1}(-1,0), F_{2}(1,0)$.\n\n设 $l: y=k x+m, A\\left(x_{1}, y_{1}\\right), B\\left(x_{2}, y_{2}\\right)$, 则联立直线与椭圆方程得\n\n$$\n\\left(2 k^{2}+1\\right) x^{2}+4 k m x+2 m^{2}-2=0\n$$\n\n判别式\n\n$$\n\\Delta=8\\left(2 k^{2}+1-m^{2}\\right)>0\n$$\n\n即\n\n$$\n2 k^{2}+1>m^{2}\n$$\n\n由题意直线 $A F_{1} 、 l 、 B F_{1}$ 的斜率成等差数列, 得\n\n$$\n\\frac{y_{1}}{x_{1}+1}+\\frac{y_{2}}{x_{2}+1}=2 k\n$$\n\n将 $y_{1}=k x_{1}+m, y_{2}=k x_{2}+m$ 代入并整理得\n\n$$\n(m-k)\\left(x_{1}+x_{2}+2\\right)=0,\n$$\n\n由直线 $l$ 不经过 $F_{1}$ 点, 舍去 $m-k=0$, 于是\n\n$$\nx_{1}+x_{2}=-2\n$$\n\n结合韦达定理有\n\n$$\n-\\frac{4 k m}{2 k^{2}+1}=-2 \\text {, }\n$$\n\n即\n\n$$\nm=k+\\frac{1}{2 k}\n$$\n\n由此可得 $k^{2}$ 的取值范围由\n\n$$\n2 k^{2}+1>m^{2}=\\left(k+\\frac{1}{2 k}\\right)^{2}\n$$\n\n\n\n确定, 即\n\n$$\nk^{2}>\\frac{1}{2}\n$$\n\n点 $F_{2}(1,0)$ 到直线 $l$ 的距离为\n\n$$\nd=\\frac{|k+m|}{\\sqrt{1+k^{2}}}=\\sqrt{\\frac{k^{2}}{1+k^{2}}} \\cdot\\left(2+\\frac{1}{2 k^{2}}\\right)\n$$\n\n结合 $k^{2}$ 的取值范围可得 $d$ 的取值范围为 $(\\sqrt{3}, 2)$.']
['$(\\sqrt{3}, 2)$']
false
null
Interval
null
68
Algebra
null
设 $a_{1}, a_{2}, \cdots, a_{n}(n \geqslant 2)$ 是实数, 证明: 可以选取 $\varepsilon_{1}, \varepsilon_{2}, \cdots, \varepsilon_{n} \in\{-1,1\}$, 使得 $$ \left(\sum_{i=1}^{n} a_{i}\right)^{2}+\left(\sum_{i=1}^{n} \varepsilon_{i} a_{i}\right)^{2} \leqslant(n+1)\left(\sum_{i=1}^{n} a_{i}^{2}\right) $$
['我们证明\n\n$$\n\\left(\\sum_{i=1}^{n} a_{i}\\right)^{2}+\\left(\\sum_{i=1}^{m} a_{i}-\\sum_{i=m+1}^{n} a_{i}\\right)^{2} \\leqslant(n+1)\\left(\\sum_{i=1}^{n} a_{i}^{2}\\right)\n$$\n\n其中 $m=\\left[\\frac{n}{2}\\right]$, 即\n\n$$\n\\varepsilon_{i}= \\begin{cases}1, & i=1,2, \\cdots, m \\\\ -1, & i=m+1, m+2, \\cdots, n\\end{cases}\n$$\n\n事实上,上述不等式的左边为\n\n$$\n\\begin{aligned}\n& \\left(\\sum_{i=1}^{m} a_{i}+\\sum_{j=m+1}^{n} a_{j}\\right)^{2}+\\left(\\sum_{i=1}^{m} a_{i}-\\sum_{j=m+1}^{n} a_{j}\\right)^{2} \\\\\n= & 2\\left(\\sum_{i=1}^{m} a_{i}\\right)^{2}+2\\left(\\sum_{j=m+1}^{n} a_{j}\\right)^{2} \\\\\n\\leqslant & 2 m \\sum_{i=1}^{m} a_{i}^{2}+2(n-m) \\sum_{j=m+1}^{n} a_{j}^{2} \\\\\n= & 2\\left[\\frac{n}{2}\\right] \\sum_{i=1}^{m} a_{i}^{2}+2\\left[\\frac{n+1}{2}\\right] \\sum_{j=m+1}^{n} a_{j}^{2} \\\\\n\\leqslant & n \\sum_{i=1}^{m} a_{i}^{2}+(n+1) \\sum_{j=m+1}^{n} a_{j}^{2} \\\\\n\\leqslant & (n+1) \\sum_{i=1}^{n} a_{i}^{2},\n\\end{aligned}\n$$\n\n其中第三行用到了柯西不等式. 从而不等式得证, 原题得证.', '首先, 由于问题中 $a_{1}, a_{2}, \\cdots, a_{n}$ 的对称性, 可设 $a_{1} \\geqslant a_{2} \\geqslant \\cdots \\geqslant a_{n}$, 此外, 若将$a_{1}, a_{2}, \\cdots, a_{n}$ 中的负数均改变符号, 则问题中的不等式左边的 $\\left(\\sum_{i=1}^{n} a_{i}\\right)^{2}$ 不减, 而右边的 $\\sum_{i=1}^{n} a_{i}^{2}$ 不变, 并且此变化不影响 $\\varepsilon$ 的选取, 因此我们可以进一步设 $a_{1} \\geqslant a_{2} \\geqslant \\cdots a_{n} \\geqslant$ 0.\n\n引理: 设 $a_{1} \\geqslant a_{2} \\geqslant \\cdots a_{n} \\geqslant n$, 则\n\n$$\n0 \\leqslant \\sum_{i=1}^{n}(-1)^{i-1} a_{i} \\leqslant a_{1}\n$$\n\n引理的证明从略.\n\n于是回到原题, 由柯西不等式及上述引理可得\n\n$$\n\\left(\\sum_{i=1}^{n} a_{i}\\right)^{2}+\\left(\\sum_{i=1}^{n}(-1)^{i-1} a_{i}\\right)^{2} \\leqslant n \\sum_{i=1}^{n} a_{i}^{2}+a_{1}^{2} \\leqslant(n+1) \\sum_{i=1}^{n} a_{i}^{2}\n$$\n\n这就证明了结论.']
null
false
null
null
null
69
Algebra
null
设 $S=\left\{A_{1}, A_{2}, \cdots, A_{n}\right\}$, 其中 $A_{1}, A_{2}, \cdots, A_{n}$ 是 $n$ 个互不相同的有限集合 $(n \geqslant 2)$,满足对任意 $A_{i}, A_{j} \in S$, 均有 $A_{i} \cup A_{j} \in S$. 若 $k=\min _{1 \leqslant i \leqslant n}\left|A_{i}\right| \geqslant 2$. 证明: 存在 $x \in \bigcup_{i=1}^{n} A_{i}$,使得 $x$ 属于 $A_{1}, A_{2}, \cdots, A_{n}$ 中的至少 $\frac{n}{k}$ 个集合(这里 $|X|$ 表示有限集合 $X$ 的元素个数).
['不妨设 $\\left|A_{1}\\right|=k$. 设在 $A_{1}, A_{2}, \\cdots, A_{n}$ 中与 $A_{1}$ 不相交的集合有 $s$ 个, 重新记为 $B_{1}, B_{2}, \\cdots, B_{s}$,设包含 $A_{1}$ 的集合有 $t$ 个, 重新记为 $C_{1}, C_{2}, \\cdots, C_{t}$. 由已知条件 $\\left(B_{i} \\cup A_{1}\\right) \\in S$, 即\n\n$$\n\\left(B_{i} \\cup A_{1}\\right) \\in\\left\\{C_{1}, C_{2}, \\cdots, C_{t}\\right\\}\n$$\n\n这样我们得到一个映射\n\n$$\nf:\\left\\{B_{1}, B_{2}, \\cdots, B_{s}\\right\\} \\rightarrow\\left\\{C_{1}, C_{2}, \\cdots, C_{t}\\right\\}, f\\left(B_{i}\\right)=B_{i} \\cup A_{1}\n$$\n\n显然 $f$ 是单射, 于是 $s \\leqslant t$. 设 $A_{1}=\\left\\{a_{1}, a_{2}, \\cdots, a_{k}\\right\\}$. 在 $A_{1}, A_{2}, \\cdots, A_{n}$ 中除去 $B_{1}, B_{2}, \\cdots, B_{s}, C_{1}, C_{2}, \\cdots, C_{t}$ 后在剩下的 $n-s-t$ 个集合中,设包含 $a_{i}$ 的集合有 $x_{i}$ 个 $(1 \\leqslant i \\leqslant k)$, 由于剩下的 $n-s-t$ 个集合中每个集合与 $A_{1}$ 的交集非空, 即包含某个 $a_{i}$, 从而\n\n$$\nx_{1}+x_{2}+\\cdots+x_{k} \\geqslant n-s-t \\text {. }\n$$\n\n不妨设 $x_{1}=\\max \\left\\{x_{1}, x_{2}, \\cdots, x_{k}\\right\\}$, 则由上式知\n\n$$\nx_{1} \\geqslant \\frac{n-s-t}{k}\n$$\n\n\n\n即在剩下的 $n-s-t$ 个集合中, 包含 $a_{1}$ 的集合至少有 $\\frac{n-s-t}{k}$ 个.\n\n又由于 $A_{1} \\subseteq C_{i}(i=1,2, \\cdots, t)$, 故 $C_{1}, C_{2}, \\cdots, C_{t}$ 都包含 $a_{1}$, 因此包含 $a_{1}$ 的集合个数至少为\n\n$$\n\\frac{n-s-t}{k}+t=\\frac{n-s+t(k-1)}{k} \\geqslant \\frac{n-s+t}{k} \\geqslant \\frac{n}{k} .\n$$']
null
false
null
null
null
70
Plane Geometry
null
如图, $\triangle A B C$ 内接于圆 $O, P$ 为弧 $B C$ 上一点, 点 $K$ 在线段 $A P$ 上, 使得 $B K$ 平分 $\angle A B C$. 过 $K 、 P 、 C$ 三点的圆 $\Omega$ 与边 $A C$ 交于点 $D$, 连接 $B D$ 交圆 $\Omega$ 于点 $E$, 连接 $P E$ 并延长与边 $A B$ 交于点 $F$, 证明: $\angle A B C=2 \angle F C B$. ![](https://cdn.mathpix.com/cropped/2023_12_20_ad11b6c1d9a80d60852cg-1.jpg?height=431&width=411&top_left_y=801&top_left_x=868)
['设 $C F$ 与圆 $\\Omega$ 交于点 $L$ (异于 $C$ ). 连接 $P B 、 P C 、 B L 、 K L$, 如下图.\n\n![](https://cdn.mathpix.com/cropped/2023_12_20_ad11b6c1d9a80d60852cg-1.jpg?height=417&width=411&top_left_y=1479&top_left_x=868)\n\n注意此时 $C 、 D 、 L 、 K 、 E 、 P$ 六点均在圆 $\\Omega$ 上, 结合 $A 、 B 、 P 、 C$ 四点共圆, 可知\n\n$$\n\\angle F E B=\\angle D E P=180^{\\circ}-\\angle D C P=\\angle A B P=\\angle F B P\n$$\n\n因此 $\\triangle F B E \\sim \\triangle F P B$, 因此\n\n$$\nF B^{2}=F E \\cdot F P\n$$\n\n又由圆幂定理可知, $F E \\cdot F P=F L \\cdot F C$, 所以\n\n$$\nF B^{2}=F L \\cdot F C,\n$$\n\n\n\n因此 $\\triangle F B L \\sim \\triangle F C B$. 因此\n\n$$\n\\angle F L B=\\angle F B C=\\angle A P C=\\angle K P C=\\angle K L C,\n$$\n\n即 $B 、 K 、 L$ 三点共线. 再根据 $\\triangle F B L \\sim \\triangle F C B$ 得\n\n$$\n\\angle F C B=\\angle F B L=\\angle F B K=\\frac{1}{2} \\angle A B C,\n$$\n\n即 $\\angle A B C=2 \\angle F C B$.', '设 $C F$ 与圆 $\\Omega$ 交于点 $L$ (异于 $C$ ), 如下图.\n\n![](https://cdn.mathpix.com/cropped/2023_12_20_22001b8ef8c0e8091af4g-1.jpg?height=417&width=460&top_left_y=862&top_left_x=844)\n\n对圆内接广义六边形 $D C P E L K$ 应用帕斯卡定理可知, $D C$ 与 $K P$ 的交点 $A 、 C L$ 与 $P E$ 的交点 $F 、 L K$ 与 $E D$ 的交点 $B^{\\prime}$ 共线, 因此 $B^{\\prime}$ 是 $A F$ 与 $E D$ 的交点, 即 $B^{\\prime}=B$,所以 $B 、 K 、 L$ 共线. 根据 $A 、 B 、 P 、 C$ 四点共圆及 $L 、 K 、 P 、 C$ 四点共圆, 得\n\n$$\n\\angle A B C=\\angle A P C=\\angle K L C=\\angle F C B+\\angle L B C,\n$$\n\n又由 $B K$ 平分 $\\angle A B C$ 知\n\n$$\n\\angle A B C=2 \\angle L B C\n$$\n\n从而 $\\angle A B C=2 \\angle F C B$.']
['证明题']
false
null
Need_human_evaluate
null
70
Plane Geometry
null
如图, $\triangle A B C$ 内接于圆 $O, P$ 为弧 $B C$ 上一点, 点 $K$ 在线段 $A P$ 上, 使得 $B K$ 平分 $\angle A B C$. 过 $K 、 P 、 C$ 三点的圆 $\Omega$ 与边 $A C$ 交于点 $D$, 连接 $B D$ 交圆 $\Omega$ 于点 $E$, 连接 $P E$ 并延长与边 $A B$ 交于点 $F$, 证明: $\angle A B C=2 \angle F C B$. <img_4198>
['设 $C F$ 与圆 $\\Omega$ 交于点 $L$ (异于 $C$ ). 连接 $P B 、 P C 、 B L 、 K L$, 如下图.\n\n<img_4215>\n\n注意此时 $C 、 D 、 L 、 K 、 E 、 P$ 六点均在圆 $\\Omega$ 上, 结合 $A 、 B 、 P 、 C$ 四点共圆, 可知\n\n$$\n\\angle F E B=\\angle D E P=180^{\\circ}-\\angle D C P=\\angle A B P=\\angle F B P\n$$\n\n因此 $\\triangle F B E \\sim \\triangle F P B$, 因此\n\n$$\nF B^{2}=F E \\cdot F P\n$$\n\n又由圆幂定理可知, $F E \\cdot F P=F L \\cdot F C$, 所以\n\n$$\nF B^{2}=F L \\cdot F C,\n$$\n\n\n\n因此 $\\triangle F B L \\sim \\triangle F C B$. 因此\n\n$$\n\\angle F L B=\\angle F B C=\\angle A P C=\\angle K P C=\\angle K L C,\n$$\n\n即 $B 、 K 、 L$ 三点共线. 再根据 $\\triangle F B L \\sim \\triangle F C B$ 得\n\n$$\n\\angle F C B=\\angle F B L=\\angle F B K=\\frac{1}{2} \\angle A B C,\n$$\n\n即 $\\angle A B C=2 \\angle F C B$.', '设 $C F$ 与圆 $\\Omega$ 交于点 $L$ (异于 $C$ ), 如下图.\n\n<img_4180>\n\n对圆内接广义六边形 $D C P E L K$ 应用帕斯卡定理可知, $D C$ 与 $K P$ 的交点 $A 、 C L$ 与 $P E$ 的交点 $F 、 L K$ 与 $E D$ 的交点 $B^{\\prime}$ 共线, 因此 $B^{\\prime}$ 是 $A F$ 与 $E D$ 的交点, 即 $B^{\\prime}=B$,所以 $B 、 K 、 L$ 共线. 根据 $A 、 B 、 P 、 C$ 四点共圆及 $L 、 K 、 P 、 C$ 四点共圆, 得\n\n$$\n\\angle A B C=\\angle A P C=\\angle K L C=\\angle F C B+\\angle L B C,\n$$\n\n又由 $B K$ 平分 $\\angle A B C$ 知\n\n$$\n\\angle A B C=2 \\angle L B C\n$$\n\n从而 $\\angle A B C=2 \\angle F C B$.']
null
false
null
null
null
71
Number Theory
null
求具有下列性质的所有正整数 $k$ : 对任意正整数 $n, 2^{(k-1) n+1}$ 不整除 $\frac{(k n) !}{n !}$.
['对正整数 $m$, 设 $v_{2}(m)$ 表示正整数 $m$ 的标准分解中素因子 2 的方幂, 则熟知\n\n$$\nv_{2}(m !)=m-S(m),\n$$\n\n这里 $S(m)$ 表示正整数 $m$ 在二进制表示下的数码之和. 由于 $2^{(k-1) n+1}$ 不整除 $\\frac{(k n) !}{n !}$ 等价于\n\n$$\nv_{2}\\left(\\frac{(k n) !}{n !}\\right) \\leqslant(k-1) n\n$$\n\n\n\n即\n\n$$\nk n-v_{2}((k n) !) \\geqslant n-v_{2}(n !),\n$$\n\n进而本题等价于求所有正整数 $k$, 使得 $S(k n) \\geqslant S(n)$ 对任意正整数 $n$ 成立. 我们证明,所有符合条件的 $k$ 为 $2^{a}$, 其中 $a=0,1,2, \\cdots$ . 一方面, 由于 $S\\left(2^{a} n\\right)=S(n)$ 对任意正整数 $n$ 成立, 故 $k=2^{a}$ 符合条件. 另一方面, 若 $k$ 不是 2 的方幂, 设 $k=2^{a} \\cdot q, a \\geqslant 0$, $q$ 是大于 1 的奇数. 下面构造一个正整数 $n$, 使得 $S(k n)<S(n)$. 因为\n\n$$\nS(k n)=S\\left(2^{a} q n\\right)=S(q n)\n$$\n\n因此问题等价于我们选取的 $q$ 的一个倍数 $m$, 使得\n\n$$\nS(m)<S\\left(\\frac{m}{q}\\right)\n$$\n\n由 $(2, q)=1$, 熟知存在正整数 $u$, 使得 $2^{u} \\equiv 1(\\bmod q)$. (事实上, 由欧拉定理知, $u$可以取 $\\varphi(q)$. )设奇数 $q$ 的二进制表示为\n\n$$\n2^{\\alpha_{1}}+2^{\\alpha_{2}}+\\cdots+2^{\\alpha_{t}}\n$$\n\n其中 $0=\\alpha_{1}<\\alpha_{2}<\\cdots<\\alpha_{t}, t \\geqslant 2$. 取 $m=2^{\\alpha_{1}}+2^{\\alpha_{2}}+\\cdots+2^{\\alpha_{t}+t u}$, 则 $S(m)=t$, 且\n\n$$\nm=q+2^{\\alpha_{t}}\\left(2^{t u}-1\\right) \\equiv 0 \\quad(\\bmod q)\n$$\n\n我们有\n\n$$\n\\begin{aligned}\n\\frac{m}{q} & =1+2^{\\alpha_{t}} \\cdot \\frac{2^{t u}-1}{q} \\\\\n& =1+2^{\\alpha_{t}} \\cdot \\frac{2^{u}-1}{q}\\left(1+2^{u}+\\cdots+2^{(t-1) u}\\right) \\\\\n& =1+\\sum_{i=0}^{t-1} \\frac{2^{u}-1}{q} \\cdot 2^{i u+\\alpha_{t}} .\n\\end{aligned}\n$$\n\n由于 $0<\\frac{2^{u}-1}{q}<2^{u}$, 故正整数 $\\frac{2^{u}-1}{q}$ 的二进制表示中的最高次幂小于 $u$, 由此可得, 对任意整数 $i, j(0 \\leqslant i<j \\leqslant t-1)$, 数 $\\frac{2^{u}-1}{q} \\cdot 2^{i u+\\alpha_{t}}$ 与 $\\frac{2^{u}-1}{q} \\cdot 2^{j u+\\alpha_{t}}$ 的二进制表示中没有相同的项. 又因为 $\\alpha_{t}>0$, 故 $\\frac{2^{u}-1}{q} \\cdot 2^{i u+\\alpha_{t}}(i=0,1,2, \\cdots, t-1)$ 的二进制表示中均不包含 1 , 故\n\n$$\nS\\left(\\frac{m}{q}\\right)=1+S\\left(\\frac{2^{u}-1}{q}\\right) \\cdot t>t=S(m)\n$$\n\n因此上述选取的 $m$ 满足要求.']
['$k = 2^{a}, a=0,1,2, \\cdots$']
true
null
Need_human_evaluate
null
72
Plane Geometry
null
平面直角坐标系中 $x O y$ 中, $P$ 是不在 $x$ 轴上的一个动点, 过 $P$ 作抛物线 $y^{2}=4 x$ 的两条切线, 切点设为 $A, B$, 且直线 $P O \perp A B$ 于 $Q, R$ 为直线 $A B$ 与 $x$ 轴的交点. <img_4085> 求证: $R$ 是定点;
['设 $P(m, n)$, 则 $A B: n y=2(x+m)$, 根据题意, 直线 $P O$ 与直线 $A B$ 垂直, 于是\n\n$$\n\\frac{n}{m} \\cdot \\frac{2}{n}=-1\n$$\n\n因此 $m=-2$. 进而 $R(2,0)$ 为定点.']
null
false
null
null
null
72
Plane Geometry
null
平面直角坐标系中 $x O y$ 中, $P$ 是不在 $x$ 轴上的一个动点, 过 $P$ 作抛物线 $y^{2}=4 x$ 的两条切线, 切点设为 $A, B$, 且直线 $P O \perp A B$ 于 $Q, R$ 为直线 $A B$ 与 $x$ 轴的交点. ![](https://cdn.mathpix.com/cropped/2023_12_20_1b16e0015535caf24a39g-1.jpg?height=409&width=354&top_left_y=892&top_left_x=888) 求证: $R$ 是定点;
['设 $P(m, n)$, 则 $A B: n y=2(x+m)$, 根据题意, 直线 $P O$ 与直线 $A B$ 垂直, 于是\n\n$$\n\\frac{n}{m} \\cdot \\frac{2}{n}=-1\n$$\n\n因此 $m=-2$. 进而 $R(2,0)$ 为定点.']
['证明题']
false
null
Need_human_evaluate
null
73
Plane Geometry
null
平面直角坐标系中 $x O y$ 中, $P$ 是不在 $x$ 轴上的一个动点, 过 $P$ 作抛物线 $y^{2}=4 x$ 的两条切线, 切点设为 $A, B$, 且直线 $P O \perp A B$ 于 $Q, R$ 为直线 $A B$ 与 $x$ 轴的交点. ![](https://cdn.mathpix.com/cropped/2023_12_20_1b16e0015535caf24a39g-1.jpg?height=409&width=354&top_left_y=892&top_left_x=888) 求 $\frac{P Q}{Q R}$ 的最小值.
['设直线 $A B$ 的倾斜角为 $\\theta$, 根据对称性, 不妨设 $\\theta$ 为锐角. 过 $P$ 作 $x$ 轴的垂线, 设垂足为 $H$, 则 $\\triangle O P H$ 与 $\\triangle O R Q$ 相似, 于是\n\n$$\n\\frac{P Q}{Q R}=\\frac{P O+O Q}{Q R}=\\frac{\\frac{2}{\\sin \\theta}+2 \\sin \\theta}{2 \\cos \\theta}=\\frac{3-\\cos 2 \\theta}{\\sin 2 \\theta}\n$$\n\n令右侧代数式为 $t$, 则\n\n$$\nt \\sin 2 \\theta+\\cos 2 \\theta=3\n$$\n\n于是\n\n$$\n\\sqrt{1+t^{2}} \\geqslant 3\n$$\n\n解得 $t \\geqslant 2 \\sqrt{2}$, 等号当 $\\theta=\\frac{1}{2} \\arctan 2 \\sqrt{2}$ 时取得. 因此所求的最小值为 $2 \\sqrt{2}$.\n所以最终答案是 $2 \\sqrt{2}$。']
['$2 \\sqrt{2}$']
false
null
Numerical
null
73
Plane Geometry
null
平面直角坐标系中 $x O y$ 中, $P$ 是不在 $x$ 轴上的一个动点, 过 $P$ 作抛物线 $y^{2}=4 x$ 的两条切线, 切点设为 $A, B$, 且直线 $P O \perp A B$ 于 $Q, R$ 为直线 $A B$ 与 $x$ 轴的交点. <img_4085> 求 $\frac{P Q}{Q R}$ 的最小值.
['设直线 $A B$ 的倾斜角为 $\\theta$, 根据对称性, 不妨设 $\\theta$ 为锐角. 过 $P$ 作 $x$ 轴的垂线, 设垂足为 $H$, 则 $\\triangle O P H$ 与 $\\triangle O R Q$ 相似, 于是\n\n$$\n\\frac{P Q}{Q R}=\\frac{P O+O Q}{Q R}=\\frac{\\frac{2}{\\sin \\theta}+2 \\sin \\theta}{2 \\cos \\theta}=\\frac{3-\\cos 2 \\theta}{\\sin 2 \\theta}\n$$\n\n令右侧代数式为 $t$, 则\n\n$$\nt \\sin 2 \\theta+\\cos 2 \\theta=3\n$$\n\n于是\n\n$$\n\\sqrt{1+t^{2}} \\geqslant 3\n$$\n\n解得 $t \\geqslant 2 \\sqrt{2}$, 等号当 $\\theta=\\frac{1}{2} \\arctan 2 \\sqrt{2}$ 时取得. 因此所求的最小值为 $2 \\sqrt{2}$.\n所以最终答案是 $2 \\sqrt{2}$。']
['$2 \\sqrt{2}$']
false
null
Numerical
null
74
Sequence
null
数列 $\left\{a_{n}\right\}$ 满足 $a_{1}=\frac{\pi}{6}, a_{n+1}=\arctan \left(\sec a_{n}\right)\left(n \in \mathbb{N}^{*}\right)$. 求正整数 $m$, 使得 $$ \sin a_{1} \cdot \sin a_{2} \cdots \sin a_{m}=\frac{1}{100} $$
['由已知条件可知, 对任意正整数 $n$,\n\n$$\na_{n+1} \\in\\left(-\\frac{\\pi}{2}, \\frac{\\pi}{2}\\right)\n$$\n\n且\n\n$$\n\\tan a_{n+1}=\\sec a_{n} , (1)\n$$\n\n由于 $\\sec a_{n}>0$, 故\n\n$$\na_{n+1} \\in\\left(0, \\frac{\\pi}{2}\\right)\n$$\n\n由 (1) 得,\n\n$$\n\\tan ^{2} a_{n+1}=\\sec ^{2} a_{n}=1+\\tan ^{2} a_{n},\n$$\n\n故\n\n$$\n\\tan ^{2} a_{n}=n-1+\\tan ^{2} a_{1}=n-1+\\frac{1}{3}=\\frac{3 n-2}{3},\n$$\n\n即\n\n$$\n\\tan a_{n}=\\sqrt{\\frac{3 n-2}{3}}\n$$\n\n因此\n\n$$\n\\begin{aligned}\n\\sin a_{1} \\cdot \\sin a_{2} \\cdots \\sin a_{m} & =\\frac{\\tan a_{1}}{\\sec a_{1}} \\cdot \\frac{\\tan a_{2}}{\\sec a_{2}} \\cdots \\frac{\\tan a_{m}}{\\sec a_{m}} \\\\\n& =\\frac{\\tan a_{1}}{\\tan a_{2}} \\cdot \\frac{\\tan a_{2}}{\\tan a_{3}} \\cdots \\frac{\\tan a_{m}}{\\tan a_{m+1}} \\\\\n& =\\frac{\\tan a_{1}}{\\tan a_{m+1}}=\\sqrt{\\frac{1}{3 m+1}},\n\\end{aligned}\n$$\n\n由\n\n$$\n\\sqrt{\\frac{1}{3 m+1}}=\\frac{1}{100}\n$$\n\n得 $m=3333$.']
['$3333$']
false
null
Numerical
null
75
Complex Numbers
null
确定所有的复数 $\alpha$, 使得对任意复数 $z_{1}, z_{2},\left(\left|z_{1}\right|,\left|z_{2}\right|<1, z_{1} \neq z_{2}\right)$, 均有 $$ \left(z_{1}+\alpha\right)^{2}+\alpha \overline{z_{1}} \neq\left(z_{2}+\alpha\right)^{2}+\alpha \overline{z_{2}} . $$
['记 $f_{\\alpha}(z)=(z+\\alpha)^{2}+\\alpha \\bar{z}$, 则\n\n$$\nf_{\\alpha}\\left(z_{1}\\right)-f_{\\alpha}\\left(z_{2}\\right)=\\left(z_{1}+z_{2}+2 \\alpha\\right)\\left(z_{1}-z_{2}\\right)+\\alpha\\left(\\overline{z_{1}}-\\overline{z_{2}}\\right) . (1)\n$$\n\n\n\n假如存在复数 $z_{1}, z_{2},\\left(\\left|z_{1}\\right|,\\left|z_{2}\\right|<1, z_{1} \\neq z_{2}\\right)$, 使得 $f_{\\alpha}\\left(z_{1}\\right)=f_{\\alpha}\\left(z_{2}\\right)$, 则由 (1) 知,\n\n$$\n\\left|\\alpha\\left(\\overline{z_{1}}-\\overline{z_{2}}\\right)\\right|=\\left|-\\left(z_{1}+z_{2}+2 \\alpha\\right)\\left(z_{1}-z_{2}\\right)\\right|\n$$\n\n利用\n\n$$\n\\left|\\overline{z_{1}}-\\overline{z_{2}}\\right|=\\left|z_{1}-z_{2}\\right| \\neq 0\n$$\n\n得\n\n$$\n|\\alpha|=\\left|z_{1}+z_{2}+2 \\alpha\\right| \\geqslant 2|\\alpha|-\\left|z_{1}\\right|-\\left|z_{2}\\right|>2|\\alpha|-2\n$$\n\n即 $|\\alpha|<2$.\n\n另一方面, 对任意满足 $|\\alpha|<2$ 的复数 $\\alpha$, 令\n\n$$\nz_{1}=-\\frac{\\alpha}{2}+\\beta \\mathrm{i}, z_{2}=-\\frac{\\alpha}{2}-\\beta \\mathrm{i}\n$$\n\n其中 $0<\\beta<1-\\frac{|\\alpha|}{2}$, 则 $z_{1} \\neq z_{2}$, 而\n\n$$\n\\left|-\\frac{\\alpha}{2} \\pm \\beta \\mathrm{i}\\right| \\leqslant\\left|-\\frac{\\alpha}{2}\\right|+|\\beta|<1,\n$$\n\n故 $\\left|z_{1}\\right|,\\left|z_{2}\\right|<1$. 此时将\n\n$$\nz_{1}+z_{2}=-\\alpha, z_{1}-z_{2}=2 \\beta \\mathrm{i}, \\overline{z_{1}}-\\overline{z_{2}}=-2 \\beta \\mathrm{i},\n$$\n\n代入 (1) 可得,\n\n$$\nf_{\\alpha}\\left(z_{1}\\right)-f_{\\alpha}\\left(z_{2}\\right)=\\alpha \\cdot 2 \\beta \\mathrm{i}+\\alpha \\cdot(-2 \\beta \\mathrm{i})=0,\n$$\n\n即 $f_{\\alpha}\\left(z_{1}\\right)=f_{\\alpha}\\left(z_{2}\\right)$.\n\n综上所述, 符合要求的 $\\alpha$ 的值为 $\\{\\alpha|\\alpha \\in \\mathbb{C},| \\alpha | \\geqslant 2\\}$.']
['$\\{\\alpha|\\alpha \\in \\mathbb{C},| \\alpha | \\geqslant 2\\}$']
true
null
Need_human_evaluate
null
76
Algebra
null
设实数 $a, b, c$ 满足 $a+b+c=1, a b c>0$. 求证: $$ a b+b c+c a<\frac{\sqrt{a b c}}{2}+\frac{1}{4} $$
['若 $a b+b c+c a \\leqslant \\frac{1}{4}$, 则命题已成立.\n\n若 $a b+b c+c a>\\frac{1}{4}$ ,不妨设 $a=\\max \\{a, b, c\\}$.\n\n因为\n\n$$\na+b+c=1,\n$$\n\n所以 $a \\geqslant \\frac{1}{3}$.\n\n又因为\n\n$$\n\\begin{aligned}\na b+b c+c a-\\frac{1}{4} & \\leqslant \\frac{(a+b+c)^{2}}{3}-\\frac{1}{4} \\\\\n& =\\frac{1}{12} \\leqslant \\frac{a}{4}, (1)\n\\end{aligned}\n$$\n\n且\n\n$$\n\\begin{aligned}\na b+b c+c a-\\frac{1}{4} & =a(b+c)-\\frac{1}{4}+b c \\\\\n& =a(1-a)-\\frac{1}{4}+b c \\\\\n& \\leqslant \\frac{1}{4}-\\frac{1}{4}+b c=b c,(2)\n\\end{aligned}\n$$\n\n其中 (1) 式等号在 $a=\\frac{1}{3}$ 时成立, (2) 式等号在 $a=\\frac{1}{2}$ 时成立, 因此 (1)(2) 中的等号不能同时成立.\n\n由于 $a b+b c+c a-\\frac{1}{4}>0$, 将 (1)(2) 两式相乘得\n\n$$\n\\left(a b+b c+c a-\\frac{1}{4}\\right)^{2}<\\frac{a b c}{4}\n$$\n\n即\n\n$$\na b+b c+c a-\\frac{1}{4}<\\frac{\\sqrt{a b c}}{2},\n$$\n\n从而\n\n$$\na b+b c+c a<\\frac{\\sqrt{a b c}}{2}+\\frac{1}{4} .\n$$']
null
false
null
null
null
77
Plane Geometry
null
如图, 在锐角三角形 $A B C$ 中, $\angle B A C \neq 60^{\circ}$, 过点 $B, C$ 分别作三角形 $A B C$ 的外接圆的切线 $B D, C E$, 且满足 $B D=C E=B C$. 直线 $D E$ 与 $A B, A C$ 的延长线分别交于点 $F, G$. 设 $C F$ 与 $B D$ 交于点 $M, C E$ 与 $B G$ 交于点 $N$. ![](https://cdn.mathpix.com/cropped/2023_12_20_562057eb197dbc1e718bg-1.jpg?height=405&width=545&top_left_y=543&top_left_x=798) 证明: $A M=A N$.
['如图, 设两条切线 $B D, C E$ 交于点 $K$, 则 $B K=C K$. 结合 $B D=C E$ 可知 $D E \\| B C$.作 $\\angle B A C$ 的平分线 $A L$ 交 $B C$ 于点 $L$, 连结 $L M, L N$.\n\n![](https://cdn.mathpix.com/cropped/2023_12_20_562057eb197dbc1e718bg-1.jpg?height=588&width=551&top_left_y=1259&top_left_x=798)\n\n由 $D E \\| B C$ 知,\n\n$$\n\\angle A B C=\\angle D F B, \\angle F D B=\\angle D B C=\\angle B A C,\n$$\n\n故 $\\triangle A B C$ 与 $\\triangle D F B$ 相似.\n\n由此并结合 $D E \\| B C, B D=C B$ 及内角平分线定理可得\n\n$$\n\\frac{M C}{M F}=\\frac{B C}{F D}=\\frac{B D}{F D}=\\frac{A C}{A B}=\\frac{L C}{L B},\n$$\n\n因此 $L M \\| B F$.\n\n同理, $L N \\| C G$.\n\n\n\n由此推出\n\n$$\n\\begin{aligned}\n\\angle A L M & =\\angle A L B+\\angle B L M \\\\\n& =\\angle A L B+\\angle A B L \\\\\n& =180^{\\circ}-\\angle B A L \\\\\n& =180^{\\circ}-\\angle C A L \\\\\n& =\\angle A L C+\\angle A C L \\\\\n& =\\angle A L C+\\angle C L N \\\\\n& =\\angle A L N,\n\\end{aligned}\n$$\n\n再结合 $B C \\| F G$ 及内角平分线定理可得\n\n$$\n\\begin{aligned}\n\\frac{L M}{L N} & =\\frac{L M}{B F} \\cdot \\frac{B F}{C G} \\cdot \\frac{C G}{L N} \\\\\n& =\\frac{C L}{B C} \\cdot \\frac{A B}{A C} \\cdot \\frac{B C}{B L} \\\\\n& =\\frac{L C}{L B} \\cdot \\frac{A B}{A C}=1,\n\\end{aligned}\n$$\n\n即 $L M=L N$.\n\n故由\n\n$$\nA L=A L, \\angle A L M=\\angle A L N, L M=L N,\n$$\n\n得到 $\\triangle A L M$ 与 $\\triangle A L N$ 全等, 因而 $A M=A N$, 证毕.']
['证明题']
false
null
Need_human_evaluate
null
77
Plane Geometry
null
如图, 在锐角三角形 $A B C$ 中, $\angle B A C \neq 60^{\circ}$, 过点 $B, C$ 分别作三角形 $A B C$ 的外接圆的切线 $B D, C E$, 且满足 $B D=C E=B C$. 直线 $D E$ 与 $A B, A C$ 的延长线分别交于点 $F, G$. 设 $C F$ 与 $B D$ 交于点 $M, C E$ 与 $B G$ 交于点 $N$. <img_4127> 证明: $A M=A N$.
['如图, 设两条切线 $B D, C E$ 交于点 $K$, 则 $B K=C K$. 结合 $B D=C E$ 可知 $D E \\| B C$.作 $\\angle B A C$ 的平分线 $A L$ 交 $B C$ 于点 $L$, 连结 $L M, L N$.\n\n<img_4245>\n\n由 $D E \\| B C$ 知,\n\n$$\n\\angle A B C=\\angle D F B, \\angle F D B=\\angle D B C=\\angle B A C,\n$$\n\n故 $\\triangle A B C$ 与 $\\triangle D F B$ 相似.\n\n由此并结合 $D E \\| B C, B D=C B$ 及内角平分线定理可得\n\n$$\n\\frac{M C}{M F}=\\frac{B C}{F D}=\\frac{B D}{F D}=\\frac{A C}{A B}=\\frac{L C}{L B},\n$$\n\n因此 $L M \\| B F$.\n\n同理, $L N \\| C G$.\n\n\n\n由此推出\n\n$$\n\\begin{aligned}\n\\angle A L M & =\\angle A L B+\\angle B L M \\\\\n& =\\angle A L B+\\angle A B L \\\\\n& =180^{\\circ}-\\angle B A L \\\\\n& =180^{\\circ}-\\angle C A L \\\\\n& =\\angle A L C+\\angle A C L \\\\\n& =\\angle A L C+\\angle C L N \\\\\n& =\\angle A L N,\n\\end{aligned}\n$$\n\n再结合 $B C \\| F G$ 及内角平分线定理可得\n\n$$\n\\begin{aligned}\n\\frac{L M}{L N} & =\\frac{L M}{B F} \\cdot \\frac{B F}{C G} \\cdot \\frac{C G}{L N} \\\\\n& =\\frac{C L}{B C} \\cdot \\frac{A B}{A C} \\cdot \\frac{B C}{B L} \\\\\n& =\\frac{L C}{L B} \\cdot \\frac{A B}{A C}=1,\n\\end{aligned}\n$$\n\n即 $L M=L N$.\n\n故由\n\n$$\nA L=A L, \\angle A L M=\\angle A L N, L M=L N,\n$$\n\n得到 $\\triangle A L M$ 与 $\\triangle A L N$ 全等, 因而 $A M=A N$, 证毕.']
null
false
null
null
null
78
Algebra
null
设 $S=\{1,2,3, \cdots, 100\}$. 求最大的整数 $k$, 使得 $S$ 有 $k$ 个互不相同的非空子集, 具有性质:对这 $k$ 个子集中任意两个不同子集,若它们的交集非空,则它们交集中的最小元素与这两个子集中的最大元素均不相同.
['对有限非空实数集 $A$, 用 $\\min A$ 与 $\\max A$ 分别表示 $A$ 的最小元素与最大元素.\n\n考虑 $S$ 的所有包含 1 且至少有两个元素的子集, 一共 $2^{99}-1$ 个.\n\n因为\n\n$$\n\\min \\left(A_{i} \\cap A_{j}\\right)=1<\\max A_{i}\n$$\n\n所以它们显然满足要求. 故 $k$ 可以取到 $2^{99}-1$.\n\n下面证明 $k \\geqslant 2^{99}$ 时不存在满足要求的 $k$ 个子集.\n\n我们用数学归纳法证明: 对整数 $n \\geqslant 3$, 在集合 $\\{1,2, \\cdots, n\\}$ 的任意 $m\\left(\\geqslant 2^{n-1}\\right)$ 个不同非空子集 $A_{1}, A_{2}, \\cdots, A_{m}$ 中, 存在两个子集 $A_{i}, A_{j}, i \\neq j$, 满足\n\n$$\nA_{i} \\cap A_{j} \\neq \\varnothing, \\wedge \\min \\left(A_{i} \\cap A_{j}\\right)=\\max A_{i} . (1)\n$$\n\n\n\n显然只需对 $m=2^{n-1}$ 的情况证明上述结论.\n\n归纳基础: 当 $n=3$ 时, 将 $\\{1,2,3\\}$ 的全部 7 个非空子集分成 3 组:\n\n第一组: $\\{3\\},\\{1,3\\},\\{2,3\\}$;\n\n第二组: $\\{2\\},\\{1,2\\}$;\n\n第三组: $\\{1\\},\\{1,2,3\\}$.\n\n由抽屉原理, 任意 4 个非空子集必有两个在同一组中, 取同一组中的的两个子集分别记为 $A_{i}, A_{j}$, 排在前面的记为 $A_{i}$, 则满足 (1).\n\n递推证明: 假设结论在 $n(\\geqslant 3)$ 时成立, 考虑 $n+1$ 的情形.\n\n若 $A_{1}, A_{2}, \\ldots, A_{2^{n}}$ 中至少有 $2^{n-1}$ 个子集不含 $n+1$, 对其中的 $2^{n-1}$ 个子集用归纳假设, 可知存在两个子集满足 (1).\n\n若至多有 $2^{n-1}-1$ 个子集不含 $n+1$, 则至少有 $2^{n-1}+1$ 个子集含 $n+1$. 将其中 $2^{n-1}+1$子集都去掉 $n+1$, 得到 $\\{1,2,3, \\cdots, n\\}$ 的 $2^{n-1}+1$ 个子集.\n\n由于 $\\{1,2,3, \\cdots, n\\}$ 的全体子集可分成 $2^{n-1}$ 组, 每组两个子集互补, 故由抽屉原理,在上述 $2^{n-1}+1$ 个子集中一定有两个属于同一组, 即互为补集. 因此, 相应地有两个子集 $A_{i}, A_{j}$, 满足 $A_{i} \\cap A_{j}=\\{n+1\\}$, 这两个子集显然满足 (1). 故 $n+1$ 时结论成立.']
['$2^{99}-1$']
false
null
Numerical
null
79
Number Theory
null
设整数 $x_{1}, x_{2}, \ldots, x_{2014}$ 模 2014 互不同余, 整数 $y_{1}, y_{2}, \cdots, y_{2014}$ 模 2014 互不同余. 证明: 可将 $y_{1}, y_{2}, \ldots, y_{2014}$ 重新排列为 $z_{1}, z_{2}, \ldots, z_{2014}$, 使得整数 $x_{1}+z_{1}$, $x_{2}+z_{2}, \cdots, x_{2014}+z_{2014}$ 模 4028 互不同余.
['记 $k=1007$ ,不妨设 $x_{i} \\equiv y_{i} \\equiv i(\\bmod 2 k), \\quad 1 \\leqslant i \\leqslant 2 k$.\n\n对每个整数 $i, 1 \\leqslant i \\leqslant k$, 若\n\n$$\nx_{i}+y_{i} \\not \\equiv x_{i+k}+y_{i+k}(\\bmod 4 k)\n$$\n\n则令 $z_{i}=y_{i}, z_{i+k}=y_{i+k}$, 从而有\n\n$$\nx_{i}+z_{i}=x_{i}+y_{i} \\not \\equiv x_{i+k}+y_{i+k}=x_{i+k}+z_{i+k}(\\bmod 4 k) .\n$$\n\n若\n\n$$\nx_{i}+y_{i+k} \\not \\equiv x_{i+k}+y_{i}(\\bmod 4 k)\n$$\n\n令 $z_{i}=y_{i+k}, z_{i+k}=y_{i}$, 则也有\n\n$$\nx_{i}+z_{i}=x_{i}+y_{i+k} \\not \\equiv x_{i+k}+y_{i}=x_{i+k}+z_{i+k}(\\bmod 4 k) .\n$$\n\n若不然,我们有\n\n$$\n\\begin{aligned}\nx_{i}+y_{i} & \\equiv x_{i+k}+y_{i+k}(\\bmod 4 k), \\\\\nx_{i}+y_{i+k} & \\equiv x_{i+k}+y_{i}(\\bmod 4 k),\n\\end{aligned}\n$$\n\n\n\n两式相加可得 $2 x_{i} \\equiv 2 x_{i+k}(\\bmod 4 k)$, 于是\n\n$$\nx_{i} \\equiv x_{i+k}(\\bmod 2 k)\n$$\n\n但 $x_{1}, x_{2}, \\cdots, x_{2014}$ 模 $2014(=2 k)$ 互不同余. 特别地, $x_{i} \\not \\equiv x_{i+k}(\\bmod 2 k)$, 矛盾.由上述构造方法知 $z_{1}, z_{2}, \\cdots, z_{2 k}$ 是 $y_{1}, y_{2}, \\cdots, y_{2 k}$ 的排列.\n\n记 $\\omega_{i}=x_{i}+z_{i}, i=1,2, \\cdots, 2 k$. 下面验证 $\\omega_{1}, \\omega_{2}, \\cdots, \\omega_{2 k}$ 模 $4 k$ 互不同余. 只需证明对任意整数 $i, j, 1 \\leqslant i<j \\leqslant k, \\omega_{i}, \\omega_{j}, \\omega_{i+k}, \\omega_{j+k}$ 模 $4 k$ 两两不同余. (1)\n\n注意, 前面的构造方法已保证\n\n$$\n\\omega_{i} \\not \\equiv \\omega_{i+k}(\\bmod 4 k), \\omega_{j} \\not \\equiv \\omega_{j+k}(\\bmod 4 k) . (2)\n$$\n\n情形 $1: z_{i}=y_{i}$, 且 $z_{j}=y_{j}$.\n\n由前面的构造方式可知\n\n$$\n\\omega_{i} \\equiv \\omega_{i+k} \\equiv 2 i(\\bmod 2 k), \\omega_{j} \\equiv \\omega_{j+k} \\equiv 2 j(\\bmod 2 k)\n$$\n\n由于 $2 i \\not \\equiv 2 j(\\bmod 2 k)$, 故易知 $\\omega_{i}$ 与 $\\omega_{j}$ 及 $\\omega_{j+k}$ 模 $2 k$ 不同余, $\\omega_{i+k}$ 与 $\\omega_{j}$ 及 $\\omega_{j+k}$ 模 $2 k$ 不同余,从而模 $4 k$ 更不同余,再结合 (2) 可见 (1) 得证.\n\n情形 2: $z_{i}=y_{i+k}$, 且 $z_{j}=y_{j+k}$.\n\n由前面的构造方式可知\n\n$$\n\\omega_{i} \\equiv \\omega_{i+k} \\equiv 2 i+k(\\bmod 2 k), \\omega_{j} \\equiv \\omega_{j+k} \\equiv 2 j+k(\\bmod 2 k)\n$$\n\n同样有 $\\omega_{i}$ 与 $\\omega_{j}$ 及 $\\omega_{j+k}$ 模 $2 k$ 不同余, $\\omega_{i+k}$ 与 $\\omega_{j}$ 及 $\\omega_{j+k}$ 模 $2 k$ 不同余, 与情形一相同地可知 (1) 得证.\n\n情形 $3: z_{i}=y_{i}$, 且 $z_{j}=y_{j+k}\\left(z_{i}=y_{i+k}\\right.$, 且 $z_{j}=y_{j}$ 的情形与此相同 $)$.\n\n由前面的构造方式可知\n\n$$\n\\omega_{i} \\equiv \\omega_{i+k} \\equiv 2 i(\\bmod 2 k), \\omega_{j} \\equiv \\omega_{j+k} \\equiv 2 j+k(\\bmod 2 k)\n$$\n\n由于 $k$ 是奇数, 故\n\n$$\n2 i \\not \\equiv 2 j+2(\\bmod 2)\n$$\n\n更有\n\n$$\n2 i \\not \\equiv 2 j+k(\\bmod 2)\n$$\n\n因此仍然有 $\\omega_{i}$ 与 $\\omega_{j}$ 及 $\\omega_{j+k}$ 模 $2 k$ 不同余, $\\omega_{i+k}$ 与 $\\omega_{j}$ 及 $\\omega_{j+k}$ 模 $2 k$ 不同余, 从而 (1) 得证.\n\n因此本题得证.']
null
false
null
null
null
80
Sequence
null
给定正数数列 $\left\{x_{n}\right\}$ 满足 $S_{n} \geqslant 2 S_{n-1}, n=2,3, \cdots$, 这里 $S_{n}=x_{1}+x_{2}+\cdots+x_{n}$. 证明: 存在常数 $C>0$, 使得 $x_{n} \geqslant C \cdot 2^{n}, n=1,2, \cdots$.
['当 $n \\geqslant 2$ 时, $S_{n} \\geqslant 2 S_{n-1}$ 等价于\n\n$$\nx_{n} \\geqslant x_{1}+\\cdots+x_{n-1}, (1)\n$$\n\n对常数 $C=\\frac{1}{4} x_{1}$, 用数学归纳法证明:\n\n$$\nx_{n} \\geqslant C \\cdot 2^{n}, n=1,2, \\cdots, (2)\n$$\n\n$n=1$ 时结论显然成立. 又 $x_{2} \\geqslant x_{1}=C \\cdot 2^{2}$.\n\n对 $n \\geqslant 3$, 假设 $x_{k} \\geqslant C \\cdot 2^{k}, k=1,2, \\cdots, n-1$. 则由 (1) 式知\n\n$$\n\\begin{aligned}\nx_{n} & \\geqslant x_{1}+\\left(x_{2}+\\cdots+x_{n-1}\\right) \\\\\n& \\geqslant x_{1}+\\left(C \\cdot 2^{2}+\\cdots+C \\cdot 2^{n-1}\\right) \\\\\n& =C\\left(2^{2}+2^{2}+2^{3}+\\cdots+2^{n-1}\\right) \\\\\n& =C \\cdot 2^{n},\n\\end{aligned}\n$$\n\n所以, 由数学归纳法知, (2) 式成立.']
null
false
null
null
null
81
Plane Geometry
null
在平面直角坐标系 $x O y$ 中, 椭圆的方程为 $\frac{x^{2}}{a^{2}}+\frac{y^{2}}{b^{2}}=1(a>b>0), A_{1}, A_{2}$ 分别为椭圆的左、右顶点, $F_{1} 、 F_{2}$ 分别为椭圆的左、右焦点, $P$ 为椭圆上不同于 $A_{1}$ 和 $A_{2}$ 的任意一点. 若平面中两个点 $Q 、 R$ 满足 $Q A_{1} \perp P A_{1}, Q A_{2} \perp P A_{2}, R F_{1} \perp P F_{1}, R F_{2} \perp P F_{2}$,试确定线段 $Q R$ 的长度与 $b$ 的大小关系.
['令 $c=\\sqrt{a^{2}-b^{2}}$, 则 $A_{1}(-a, 0), A_{2}(a, 0), F_{1}(-c, 0), F_{2}(c, 0)$.\n\n设 $P\\left(x_{0}, y_{0}\\right), Q\\left(x_{1}, y_{1}\\right), \\mathbb{R}\\left(x_{2}, y_{2}\\right)$, 其中 $\\frac{x_{0}^{2}}{a^{2}}+\\frac{y_{0}^{2}}{b^{2}}=1, y_{0} \\neq 0$.\n\n由 $Q A_{1} \\perp P A_{1}, Q A_{2} \\perp P A_{2}$ 可知\n\n$$\n\\begin{aligned}\n& \\overrightarrow{A_{1} Q} \\cdot \\overrightarrow{A_{1} P}=\\left(x_{1}+a\\right)\\left(x_{0}+a\\right)+y_{1} y_{0}=0, (1) \\\\\n& \\overrightarrow{A_{2} Q} \\cdot \\overrightarrow{A_{2} P}=\\left(x_{1}-a\\right)\\left(x_{0}-a\\right)+y_{1} y_{0}=0. (2)\n\\end{aligned}\n$$\n\n将 (1)、(2) 相减, 得 $2 a\\left(x_{1}+x_{0}\\right)=0$, 即 $x_{1}=-x_{0}$, 将其代入 (1), 得 $-x_{0}^{2}+a^{2}+y_{1} y_{0}=0$,故 $y_{1}=\\frac{x_{0}^{2}-a^{2}}{y_{0}}$, 于是 $Q\\left(-x_{0}, \\frac{x_{0}^{2}-a^{2}}{y_{0}}\\right)$.\n\n根据 $R F_{1} \\perp P F_{1}, R F_{2} \\perp P F_{2}$, 同理可得 $R\\left(-x_{0}, \\frac{x_{0}^{2}-c^{2}}{y_{0}}\\right)$. 因此\n\n$$\n|Q R|=\\left|\\frac{x_{0}^{2}-a^{2}}{y_{0}}-\\frac{x_{0}^{2}-c^{2}}{y_{0}}\\right|=\\frac{b^{2}}{\\left|y_{0}\\right|}\n$$\n\n由于 $\\left|y_{0}\\right| \\in(0, b)$, 故 $|Q R| \\geqslant b$ (其中等号成立的充分必要条件是 $\\left|y_{0}\\right|=b$, 即点 $P$ 为 $(0, \\pm b))$.']
['$|Q R| \\geqslant b$']
false
null
Need_human_evaluate
null
End of preview.
README.md exists but content is empty. Use the Edit dataset card button to edit it.
Downloads last month
0